Sie sind auf Seite 1von 98

Solutions to

© SAP Group Pte Ltd. ALL RIGHTS RESERVED.


SOLUTIONs

LCM is 2 × 2 × 2 × 2 × 3 × 3 × 5 = 720
1 NUMBERS Using prime factorisation method
Factorise the numbers to express in their index
1 67, 71 notation
For the integer numbers denoted by p such that 48 = 24 × 31
61 < p ≤ 71, 61 is excluded and 71 is included. (48 = 2 × 2 × 2 × 2 × 3)
The integers are 62, 63, 64, 65, 66, 67, 68, 69, 72 = 23 × 32
70 and 71. (72 = 2 × 2 × 2 × 3 × 3)
Of these, all the even numbers can be excluded 90 = 21 × 32 × 51
leaving 63, 65, 67, 69 and 71 (the only even (90 = 2 × 3 × 3 × 5)
prime number is 2). Besides being divisible by For HCF take the common prime factors with
1 and themselves, 63, 65 and 69 are divisible the lowest power.
by 3, 5 and 3 respectively and therefore are not HCF of 48, 72, 90 is 21 × 31 = 6
prime numbers. Only 67 and 71 would then be 2 is a common prime factor and 1 is its lowest
prime numbers as they could only be divided power
by 1 and themselves only. 3 is a common prime factor and 1 is its lowest
power
2 100, 121, 144, 169, 196, 225, 256, 289, 324 5 is not a common prime factor and is not
and 361 taken
For the integer numbers denoted by n such For LCM take all prime factors in the numbers
that 100 ≤ n < 400, 100 is included and 400 is with the highest power.
excluded. The perfect square numbers in that LCM of 48, 72, 90 is 24 × 32 × 51 = 720
range are 100, 121, 144, 169, 196, 225, 256, 2 has a highest power of 4
289, 324 and 361 obtained from 102, 112, 122, 3 has a highest power of 2
132, 142, 152, 162,172, 182 and 192. 5 is only present in 90 and the power is 1
3 729 5 15
Perfect cube numbers in that range are 512 The maximum number is obtained by finding
and 729 obtained by 83 and 93. Of these, the the largest integer which could divide all the
only perfect square number is 729 which is 272. three numbers. That will give equal number of
Note that it is easier to find the perfect cube blocks of a particular colour in each object in
numbers and then narrow down for the perfect order that all the objects are identical. Hence,
square number. find HCF.
4 6 and 720 Using prime factorisation method
Using repeated division method Factorise the numbers to express in their index
2 48, 72, 90 notation
3 24, 36, 45 30 = 21 × 31 × 51
8, 12, 15 (no more common factors except 1) (30 = 2 × 3 × 5)
HCF is 2 × 3 = 6 75 = 31 × 52
2 48, 72, 90 (75 = 3 × 5 × 5)
2 24, 36, 45 90 = 21 × 32 × 51
2 12, 18, 45 (90 = 2 × 3 × 3 × 5)
2 6, 9, 45 For HCF take the common prime factors with
3 3, 9, 45 lowest power.
3 1, 3, 15 HCF is 31 × 51 = 15
5 1, 1, 5 2 is not a common prime factor and is not
1, 1, 1 taken.

S Success In Mathematics
© SAP Group Pte Ltd

Solution.indd 1 9/7/2011 2:43:37 PM


3 is a common prime factor and 1 is its lowest Power of 3 is not a multiple of 3 and has to be
power. multiplied by 3 to make it 33.
5 is a common prime factor and 1 is its lowest Power of 5 is not a multiple of 3 and has to be
power. multiplied 25 (52 = 25) to make it 53.
Therefore, 360 has to be multiplied by
6 432 3 × 25 = 75.
Note that the number of blocks in the length, 360 × 75 = 27000 is a perfect cube.
breadth and height of the cube is different
but all three dimensions have to be the same. 61
____
9 2 ​    ​ 
Therefore, the smallest possible multiple of 12, 495
9 and 8 has to be found which is the LCM. ··
​ ​
2.1​23​ = 2.12323232323…………
First, state the prime factors of the numbers in
index notation:
··
​ ​
10 × 2.1​23​ = 21.23232323………
12 = 22 × 31 ··
​ ​
1000 × 2.1​23​ = 2123.23232323………
9 = 32 ··
​ ​
··
​ ​
Since (1000 × 2.1​23​)  – (10 × 2.1​23​) 
8 = 23 ··
​ ​
= 990 × 2.1​23​, 
For LCM, take all prime factors in the numbers
with the highest power. ··
​ ​
990 × 2.1​23​ = 2102
LCM is 23 × 32 = 72 ··
​ ​ _____
2102
Hence 2.1​23​ = ​  990 ​ 
2 has a highest power of 3
3 has a highest power of 2 ____
61
= 2 ​ 495  ​ 
The dimensions of the cube formed will
be 72 cm with 6 cuboids along the length 11π ___
____ 22
10 ​   ​ , ​  7 ​ , 3.142, π and 3.14
(72 ÷ 12 = 6), 8 cuboids along the breadth 10
(72 ÷ 9 = 8) and 9 cuboids along the height Using the calculator,
(72 ÷ 8 = 9). ___
22
​  7 ​ (not the exact value of π) = 3.142857
The total number of cuboids required is
π = 3.14159
6 × 8 × 9 = 432.
____
11π
​ 10 ​ = 3.457143
7
28
____
11π
Prime factorise 21952 Therefore, the largest value is ​ 10 ​ followed
21952 = 2 × 10976 ___
22
by ​  7 ​ , 3.142, π and 3.14 in that order.
= 2 × 2 × 5488
= 2 × 2 × 2 × 2744
11 Saturday, Thursday
= 2 × 2 × 2 × 2 × 1372
15 April 2007 comes 365 days from 15 April
= 2 × 2 × 2 × 2 × 2 × 686
2006 (there is no 29 February in between).
= 2 × 2 × 2 × 2 × 2 × 2 × 343
Since 365 is a multiple of 7 plus 1 (365 ÷ 7 =
=2×2×2×2×2×2×7×7×7
52 remainder 1), every multiple of 7 days will
= 26 × 73
3
______ return to the same day of the week. Therefore,
​√  21952 ​ 
= 22 × 71 (dividing the powers by 3) 15 April 2007 will be one day of week after that
= 28 of 15 April 2006. Hence, 15 April 2006 must be
a Saturday.
8 75
15 April 2010 comes 1096 days (365 + 365
Prime factorise 360
+ 366) from 15 April 2007 (there is one 29
360 = 2 × 180
February 2008 in between). The remainder
= 2 × 2 × 90
after 1096 divided by 7 is 4. The day of the
= 2 × 2 × 2 × 45
week moves 4 days forward. Therefore 15
= 2 × 2 × 2 × 3 × 15
April 2010 was on a Thursday.
=2×2×2×3×3×5
= 23 × 32 × 51 12 (a) 14.0, (b) 14.026
For a perfect cube the powers must be multiples The three significant figures will be 14.0
of 3. counting from the first digit which is not a
Power of 2 is already a multiple of 3. zero. The fourth significant figure is 2 which is
dropped since it is less than 5.

S Success In Mathematics
© SAP Group Pte Ltd

Solution.indd 2 9/7/2011 2:43:38 PM


The fourth decimal digit counted after the the mixed number and vice versa. Hence, the
decimal point is 7 which requires the third answer cannot be obtained by simply keying in
decimal digit to be rounded up. the numbers into the calculator.

13 • • • • • 15 (a) 7.91 × 10–10, (b) 6.58 × 1012


–3 –2 –1 0 1 2 3 4 5 For numbers less than 1 the decimal moves to
the right by n number of steps and the number
Numerically,
becomes × 10–n.
___

(A) ​  3 ​ = 4.1 For numbers more than 1 the decimal moves to
___ the right by n number of steps and the number
(B) √
​ 15 ​ = 3.8
____ becomes × 10n.

3
(C) ​√  –20 ​ = –2.7 These has to make the numbers A × 10n where
____
–10 1 ≤ A <10 and n is an integer
(D) ​  7 ​  = –1.4 Therefore
(E) –0.6 0.000000000791 = 7.91 × 10–10 and
6580000000000 = 6.58 × 1012
Therefore, they could be positioned on the
number line accordingly 616
_____
16 ​    ​ 
2825
C D E B A

–3

–2

–1

0 1 2 3
• •
4 5
​ 
( (  ) (  ) ) ( 
__
3 2 __
___________
1 3
    ​  = ​ 
) ___ ____
​ ​​ ​ 5 ​   ​​ ​– ​​ ​ 5  ​  ​​ ​  ​ ​ ​  9  ​ – ​  1   ​  ​
25 125
__________
 ​ 
 
2641
_____
(  ) ( 
__
5 ___ 9
​ ​ 7 ​  + ​ 10  ​   ​ ) __
5 ___ 9
​ ​ 7 ​  + ​ 10  ​   ​
14 3 ​   ​  LCM of 25 and 125 is 125.
3528
First add the fractions within the brackets by LCM of 7 and 10 is 7 × 10 = 70 as there are no
changing them to common denominators using common factors.
the LCM of 3 and 7 which is 21.
​ 

___9 ____
) ( 
​ ​ 25  ​ – ​ 125
__________
1
) ____
45 ____
   ​  ​ ​ ​ 125  ​ – ​ 125
 ​    = ​ 
___________
1
   ​  ​
 ​  
(  ) ( 
__
2 __ 4 ___ 21 ___ 14 ___ 12
1 + ​ 3  ​– ​ 7 ​  = ​ 21 ​ + ​ 21 ​ – ​ 21 ​  __
5 ___ 9
​ ​ 7 ​  + ​ 10  ​   ​ ) ___
50 ___ 63
​ ​ 70 ​ + ​ 70 ​   ​


___
23
= ​ 21 ​  (  ) (  ) ____
44 ____
70
= ​ ​ 125  ​  ​× ​ ​ 113  ​  ​

Therefore ​ 
(  __
2 __
)
4 2
​​ 1+ ​ 3 ​  – ​ 7 ​   ​​ ​
__________ __________
 
(  )
 ​= ​ 
___
23 2
​​ ​ 21 ​   ​​ ​
   ​ 
_____
616
= ​ 2825  ​ 
(0.2 + 0.12) (0.2 + 0.12)
17 A = 2, B = 3, a = 15 and b = 5

= ​ 
___
23 2
(  )
​​ ​ 21 ​   ​​ ​
______
 
 ​ 
Prime factors of 24 are 23 × 31
(0.32) Therefore, 245 = (23 × 31) × (23 × 31) × (23 × 31)
× (23 × 31) × (23 × 31)


____
529
​ ​ 441 ​  ​
______
= ​ 

 ​ 
) = 23 × 23 × 23 × 23 × 23 × 31 × 31 × 31 × 31 × 31
____
32
​ ​ 100  ​  ​ (  )

= 215 × 35
A = 2, B = 3, a = 15 and b = 5


____
529
(  ____
32
) (  )
= ​ ​ 441 ​  ​÷ ​ ​ 100  ​  ​ 18 3500
A × B must give a value with all the powers of

____
529
(  ____
100
= ​ ​ 441 ​  ​× ​ ​  32 ​  ​) (  )
its prime factors as multiple of 5.
Therefore, B must be a value of minimum of
______
52900 22 × 53 × 71.
= ​ 14112 ​ 
A × B will then be (23 × 52 × 74) × (22 × 53 × 71)
______
13225 = 25 × 55 × 75
= ​  3528 ​ 
B = 22 × 53 × 71
_____
2641  = 3500
= 3 ​ 3528 ​ 
Note that although the calculator is allowed to 19 An easy way to approach this question is to
be used, the question requires the knowledge take a simple value of X such that 0 < X < 1.
of how improper fractions are converted to find __
1
Let X = ​   ​  .
2

S Success In Mathematics
© SAP Group Pte Ltd

Solution.indd 3 9/7/2011 2:43:38 PM


__
1
(A) X = ​ 2 ​  3 E, B, D, C, F and A.


__
1
(B) –X = –​ 2  ​ (  )
(A) –2x = –2​ –​ 2 ​   ​
__
1

__
1 =1
(C) –X 2 = –​ 4 ​ 
__
1
__
1 (B) x = –​ 2 ​ 
(D) (–X)2 = ​ 4  ​

__
1
(E) X 3 = ​ 8 ​ 
(  )__
1 3 __
1
(C) x3 = ​​ –​ 2 ​   ​​ ​= –​ 8 ​ 


__
1
(F) (–X)3 = –​ 8 ​ 

x _____
__ (  )__
1
​ –​ 2 ​   ​
(D) ​ 2  ​= ​  2 ​ 
__
1
 = –​ 4 ​ 
__
1
(  (  ) )
(G) –X  = –​ 8 ​ 
3
__
1 2
(E) –(2x)2 = –​​ 2 × ​ –​ 2 ​   ​  ​​ ​
The values could then be marked on the number
line as follows: = –(–1)2
B C F, G E D A
= –1

–1 0 1
(  )
__
1 2
(F) 2x2 = 2 × ​​ –​ 2 ​   ​​ ​
__
1
= 2 × ​ 4 ​ 
20 2, 3, 5, 7 and 11
__
1
Numerically, = ​ 2  ​
_____ __
1
3
​√  –900 ​ = –9.65.... The smallest value is –1 and followed by – ​ 2 ​ ,
____
15π __
1 __ 1 __ 1
​  4 ​  = 11.78... – ​ 4 ​ , – ​ 8 ​ , ​ 2 ​  and 1. Hence, the order is –(2x)2, x,
x
__
Prime numbers can only be integers greater ​ 2  ​, x3, 2x2, –2x.
than 1.
k
__
Therefore, the prime numbers here are 2, 3, 5, 7 4 ​   ​ 
3
and 11. Let the length of the rectangle be x.
x
__
The breadth of the rectangle is ​ 2  ​.
2 BASIC ALGEBRA The perimeter = 2 × length + 2 × breadth
x
__
= 2 × x + 2 × ​ 2  ​
1 $5(x + 4), $​ ​  ( 100
_____
x+4 )
  ​  ​ = 2x + x
(a) Each of the x members paid $5. = 3x
Total collected = $5 × (x + 4) = $5(x + 4) Since perimeter = k
(the $ must be included for completeness)
k = 3x
(b) Total payment = $100.
Each will have to pay an equal share of __k
x = 3​   ​ 

_____
100
$100 ÷ (x + 4) = $​ ​ x + 4  ​  ​ ) 5 –2x3 + 6xy2 + xy + 3x + y
3
__ Group the terms according to the like terms.
2 8 ​ 5 ​  –5x3 + 3xy – 2x3 + 3xy2 + 3y – 2xy + 5x3 + 3xy2
Substituting x = 2 and y = –3, + 3x – 2y
= –5x3 – 2x3 + 5x3 + 3xy2 + 3xy2 – 2xy + 3xy +
__
1
​ 5 ​ x3 + 3y2 + 3xy – 2 3x – 2y + 3y

(  )
__
1 = –2x3 + 6xy2 + xy + 3x + y
= ​ ​ 5 ​   ​(2)3 + 3(–3)2 + 3(2)(–3) – 2
–20x2
_____
6 ​   ​   
(  )
__
1
= ​ ​ 5 ​   ​(8) + 3(9) + 3(2)(–3) – 2

3
(–4xy) × 5x ÷ 3y = –4 × x × y × 5 × x ÷ (3 × y)
(  )
__
8
= ​ ​ 5 ​   ​+ 27 – 18 – 2 = –20x2y ÷ 3y
–20x2y
__
3 ______
= 28 ​ 5  ​– 18 – 2 = ​  3y   
​ 

__
3 _____
–20x2
= 8 ​ 5  ​ = ​  3 ​ 
 

S Success In Mathematics
© SAP Group Pte Ltd

Solution.indd 4 9/7/2011 2:43:39 PM


16
___ 13 x : y = 4 : 11
7 –​   ​ xy – 8xy2
3
__
2 __
2 x + 2x + 3x + 4x + .......... +10x = 20y
–2y(3x + 4xy) + ​ 3 ​  xy = –6xy – 8xy2 + ​ 3  ​xy
(1 + 2 + 3 + ......... + 10)x = 20y
__
2
= –6xy + ​ 3 ​  xy – 8xy2 55x = 20y
___
16 __x ___20
= –​  3 ​ xy – 8xy2 ​ y ​= ​ 55 ​ 
8 –48x + 30y + 4xy ___
4
= ​ 11  ​ 
–12(3x – 4y) – 6(2x + 3y) + 4xy
= –36x + 48y – 12x – 18y + 4xy x : y = 4 : 11
= –48x + 30y + 4xy
14 (a) x is doubled
9 2πx2 2(2a)
_____ 2(4a
2
_____)
2
__
1 If a and b are doubled ​   ​ 
= ​   ​ 
Area = ​ 2 ​  × π × (2x)2 (2b) (2b)
__
1 (4a
____)
2
= ​ 2 ​  × π × 4x2 = ​   ​   
b

(  )
= π × 2x2 ___2
2a
= 2πx2 = 2​ ​   ​   ​
b
= 2x
883v3
_____
10 ​   ​   cm3 x is doubled.
8
Each side of a red cube is v cm. __
1
__v (b) x becomes ​   ​ x
Each side of a blue cube is 2​   ​  cm. 2
Each side of a red cube is 3v cm.

__
1 2
_______ _______
If a and b are halved ​ 
(  )
 ​ =
  ​ 
__
1
(  )
​ ​ ​ 2  ​a  ​​ ​ 2​ ​ 4  ​​a2​ ​  ​
2
 ​ 


Volume of each red cube is v3 cm3.
__v 3
(  )
(v3)
____
Volume of each blue cube is ​ 2​   ​   ​ cm3 = ​  8 ​  cm3
__
1
(  )
​ ​ 2 ​  b  ​
__
1
(  )
​ ​ 2 ​  b  ​

4​( ​ 4  ​a  )​
__
1 2
Volume of each green cube is (3v)3 cm3 = 27v3 cm3 _______
= ​   ​ 
 
___
3v3 b
Total volume = 2v3 + ​  8 ​ + 4 (27v3) cm3
a__ 2
= ​   ​ 
___
3v3 b
= 2v3 + ​  8 ​ + 108v3 cm3
__
1 2a ___2
__
3 = ​ 2 ​  (​   ​ )
= 110​ 8 ​  v3 cm3 b
_____
883v3 __
1
= ​  8 ​ 
 cm3 = ​ 2 ​  x
__
1
x becomes ​ 2 ​  x.
11 x = 0
__
2 49
____
–2(3x + 4) + ​ 3 ​  x = –8 15 ​    ​ 
216
__
2
–6x – 8 + ​ 3 ​  x = –8
​ 

__
5 __ 2
) ( 
​ ​ 4 ​ x – ​ 3 ​x 
_________
___
15 ___
  ​ ​ ​   ​ x – ​    ​x 
 ​ 
12
___________
= ​ 
)8
12   ​
 ​ 
__
(  ) ( 
2
–6x + ​ 3 ​  x = 0 __
4 __
4
​ ​ 7 ​ x + 2x  ​ ​ ​ 7 ​ x + ​  7 ​x  )
___
14
  ​
__
1
–5 ​ 3 ​  x = 0
x=0
(  ___
7
)
​ ​ 12  ​ x  ​
______
= ​   ​ 
–9
____
(  ___
18
)
​ ​  7 ​ x  ​
12 x = ​    ​ 

256
Substitute y = –6
(  ___
7
)
​ ​ 12  ​   ​
_____
= ​   ​ 

__
2
(  )
–2(–6)(3x + 4x(–6)) + ​ ​ 3 ​   ​(–6) x = 9 (  ___
18
)
​ ​  7 ​   ​
12(3x – 24x) – 4x = 9 ___7 ___ 18
= ​ 12  ​ ÷ ​  7 ​ 
12(–21x) – 4x = 9
–252x – 4x = 9 ___7 ___ 7
= ​ 12  ​ × ​ 18  ​ 
–256x = 9
____
–9 ____
49
x = ​ 256  ​  = ​ 216  ​ 

S Success In Mathematics
© SAP Group Pte Ltd

Solution.indd 5 9/7/2011 2:43:39 PM


c2
___ Interest on loan $ 2100n (5% of $40 000
16 ​    ​ 
4π for n years)
c = 2πr Total loan amount $ 42 000 + $ 2100n
c
___ Repayment would be done by equal monthly
r = ​ 2π  ​ 
instalment for 12n months.

___
(  )
c 2
area = π​​ ​ 2π  ​   ​​ ​ $(42 000 + 2100n)
________________
Monthly repayment = ​  12n     ​ 
(  )

c2
___
= π​ ​  2  ​  ​

_____
3500
= $(​  n ​ 
 + 175)
c2
___
= ​ 4π  ​ 
3 $51 500
Cost price of car A = $50 000
Selling price of car A = $50 000 – 5% of $50 000
3 ARITHMETIC PROBLEMS = $47 500
Cost price of car A and B = $90 000
1 –11.8% (one decimal place) or a decrease of Profit of car A and B = 10% of $90 000
11.8% = $9000
2005 profit = 2005 income – 2005 expenditure Selling price of car A and B = $90 000 + $9000
= $150 000 – $122 000 = $99 000
= $28 000 Selling price of car B = $99 000 – $47 500
2006 income = $150 000 + 10% of $150 000 = $51 500
= $150 000 + $15 000
= $165 000 4 8 years, $12 184.03 (to the nearest cent)
2006 expenditure = $122 000 + 15% of $122 000 Interest earned = $12 000 – $10 000
= $122 000 + $18 300 = $2000
= $140 300 Here P = $10 000
2006 profit = 2006 income – 2006 expenditure R = 2.5
= $165 000 – $140 300 I = $2000
= $24 700 $10 000 × 2.5 × T
_______________
$2000 = ​  100 ​    
change in profit for 2006 as compared to 2005 _____________
100
= $24 700 – $28 000 rearranging T = $2000 × ​   
   ​
($10 000 × 2.5)
= –$3300 = 8 years
percentage change in profit for 2006 as ____
R
Using, Pn = P(1 + 100
​     ​) n
compared to 2005
____
2.5
change = $10 000(1 + ​ 100  ​) 8
_______
= ​    ​  × 100%
original = $10 000(1.025)8
–$3300
_______ = $12 184.03 (to the nearest cent)
= ​    ​ 
× 100%
$28 000
5 $20 301.88 (to the nearest cent)
= –11.8% (1 decimal place)
First find the monthly interest rate.
2 ( 
3500
_____
$​ ​  n ​ 
 + 175  ​ )
___
3
monthly interest rate = ​ 12  ​%  
Price of car $60 000 = 0.25 %
Down payment $18 000 (30% of $60 000) Modify the problem to count the interest in
Loan amount $42 000 terms of months instead of years and use the
The loan amount can be repaid in a certain number compound interest formula where n = 6.
of years taken as n. The interest on the loan will Here P = $20 000
be worked out on the basis of simple interest, R = 0.25
even though payments will be made monthly and T=6
not at the end of the loan period. ____
R
Pn = P(1 + 100
​     ​) n
____
PRT
I = ​ 100 ​  ____
0.25 6
= $20 000(1 + ​ 100 ​  )
$42 000 × 5 × n
_____________
= ​      
100 ​ = $20 000(1.0025)6
= $2100n = $20 301.88 (to the nearest cent)

S Success In Mathematics
© SAP Group Pte Ltd

Solution.indd 6 9/7/2011 2:43:40 PM


50 000 + 2750n < 12 000n
6 S$3825
______
15 000 50 000 < 9250n
Amount received by Alex in US$ = ​  1.50 ​ 
 
______
50 000
= US$10 000 Therefore n > ​  9250 ​ 
After spending 75%, he had 25% of the US$10 000. n > 5.4
Amount to be converted = 25% of US$10 000 Minimum number of years for paying less than
= US$2500
$1000 per month the number of years of loan
Value of US$1 after increase in value
= 1.50 × 1.02 should be 6.
= S$1.53 10 300
Amount received by Alex in S$ = 2500 × 1.53 Since the selling price of the shirt includes 32%
= S$3825
of cost price, so the selling price must be 132%
7 $4153.85 of the cost price.
From the table, given Bernard’s income tax of $33
____
$1410, he must have a $900 tax for first $40 000. If 132% is $33, 100% is ​ 132 ​ × 100 = $25 (cost
The remaining amount of $390 ($1410 – $900 price)
= $510) would have to be the amount of tax Reduced price during sales = $33 – 20% of $33
payable for the next $40 000 at a rate of 8.5 %. = $26.40
____
100 Profit made if sold at $26.40 = $26.40 – $25
If 8.5 % is $510, 100% is $510 × ​ 8.5 ​ = $6000
Bernard’s chargeable income = $40 000 + $6000 = $1.40
= $46 000 Gross profit for consignment = $420
Bernard’s income $420
_____
Number of shirts sold = ​   
 ​ 
= chargeable income + total reliefs $1.40
= $46 000 + $8000 = 300
= $54 000
$54 000
_______ 11 56
Average monthly salary = ​  13 ​    Using the ratio of the number of each computer
= $4153.85 (nearest cent) model is 3x, 7x and 4x for the models X, Y and
Z respectively.
8 $63.49 (nearest cent)
Danny’s electricity usage this month after The total commission received for model
reduction in usage X = $50 × 3x
= 310 – 5% of 310 = $150x
= 294.5 units The total commission received for model
Rate of payment for usage after increase Y = $60 × 7x
= $0.20 + 0.75% of $0.20 = $420x
= $0.2015 per unit The total commission received for model
Electricity charges to be paid Z = $80 × 4x
= 294.5 × $0.2015 = $320x
= $59.34 (nearest cent) Therefore $150x + $420x + $320x = $3560
Charges inclusive of GST = $59.34 + 7% of $59.34
$890x = $3560
= $63.49 (nearest cent)
_____
3560
9 6 x = ​  890 ​ 
Let the number of years for repayment be n. =4
Loan amount $50 000 Total number of units sold = 3x + 7x + 4x
Interest on loan $2750n (5.5% of $50 000 = 14x
for n years)
= 14 × 4
Total loan amount $50 000 + $ 2750n
Number of months of repayment = 12n = 56
$(50 000 + 2750n)
________________ 12 37.5 %
Monthly repayment ​  12n     ​  Using the ratio of the ticket prices, let each adult
Since the monthly repayment must be < $1000 ticket be $5x and each child ticket, be $2x.
(50 000 + 2750n)
_______________ Using the ratio of the number of adults’
​  12n       ​< 1000 attendance is 2y and children’s attendance is
(12n is a positive value, multiply 12n on both 3y.
sides) Total payment for adults = $5x × 2y = $10xy

S Success In Mathematics
© SAP Group Pte Ltd

Solution.indd 7 9/7/2011 2:43:40 PM


Total payment for children = $2x × 3y ___
14
Cross multiplying, x = 85 × ​ 17 ​ 
= $6xy
= 70
Total payment collected = $10xy + $6xy
= $16xy __
4
15 4 ​ 5 ​  days
Percentage payment for children’s tickets
$6xy
_____ Number of rooms painted by the worker per
= ​    ​ 
× 100% ___
13
$16xy day = ​  3 ​ rooms per day
___6
= ​ 16  ​ × 100% Number of rooms painted by the assistant per
= 37.5 % (3 sig. fig). ___
11
day = ​ 4 ​ rooms per day
13 870 km (nearest km) Total number of rooms painted by both working
Distance travelled per litre of petrol last month together per day
____
920
= ​  80 ​ km per litre ( 
___
13 ___
)
11
= ​ ​  3 ​ + ​ 4 ​   ​rooms per day
= 11.5 km ___
85
= ​ 12 ​ rooms per day
Cost of petrol last month = $1.55 × 80
= $124 Number of days taken to paint 34 rooms
Distance travelled per litre of petrol this month ___
85
= 34 ÷ ​ 12 ​ 
= 11.5 + 2% of 11.5
= 11.73 km per litre ___
12
= 34 × ​ 85 ​ 
Let Mike’s distance travelled this month be x. __
4
Price of petrol after increase = $1.55 + 5% of $1.55 = 4 ​ 5 ​  days
= $1.6725
x
_____ 16 $1152
Number of litres of petrol required = ​ 11.73    ​  Profit of the business is 12% of $72 000 = $8640
litre If they re-invested 70% of the profit, the
x
_____ percentage of profit available for sharing is
Cost of petrol this month = $1.6725 × ​ 11.73    ​ 
30%.

_______
1.6725x
)
= $​ ​  11.73 ​  
 ​ Amount of profit available for sharing
= 30% of $8640
If his petrol cost has to be the same as last month,
= $2592

_______
1.6725x
$​ ​  11.73 ​   )
 ​= $124
__
4
Jack’s share as a fraction is ​ 9 ​  of the business
_______
1.6725x (4 units out of a total of 4 + 5 = 9 units)
​  11.73 ​ 
 
= 124
__
4
x = 870 km (nearest km) Profit received by Jack = ​ 9 ​  × $2592

14 70 tonnes = $1152
The ratio, number of units of A used : number
of units of C produced, is 1:2. (The actual
units could be any fixed quantity like number 4 BASIC GEOMETRY
of bags, boxes, etc., with each unit having a
standard fixed mass.) 1 120º
Convert the ratio in units to a ratio of, Draw a line through ∠BEC, which is parallel to
mass of A used : mass of C produced AB and CD.
1 × 28 grams : 2 × 17 grams
A B
28 grams : 34 grams 70º
28 : 34
X Y
14 : 17 E
The ratio of 14 : 17 means 14 grams of A will
produce 17 grams of C or 14 kg of A will 130º
produce 17 kg of C, etc. D
C
Let the mass of A required be x tonnes.
14 : 17 must be equal to x : 85. ∠ABE = ∠BEY = 70º (alternate angles)

S Success In Mathematics
© SAP Group Pte Ltd

Solution.indd 8 9/7/2011 2:43:41 PM


∠YEC = 180º – ∠DCE (interior angles) ∠QDE = 80º – ∠BDQ
= 180º – 130º = 80º – 70º
= 50º = 10º
∠BEC = ∠BEY + ∠YEC ∠QDE = ∠DER = 10º (alternate angles)
= 50º + 70º ∠REG = 180º – ∠FGE (interior angles)
= 120º = 180º – 120º
= 60º
2 29º, 6º, 151º
∠DEG = ∠DER + ∠REG
(i) ∠FHB = ∠HID = 35º (corresponding angles)
= 10º + 60º
∠HIC = 180º – ∠HID
= 70º
= 180º – 35º
= 145º 5 26, 38
Since ∠HIC = 5x = 145º If the equal angles are (3x + 2)º each,
x = 29º 2(3x + 2)º + (x – 6)º = 180º
∠FCD = x = 29º 6x + 4 + x – 6 = 180
(ii) ∠CFI + ∠CIF + ∠ICF = 180º 7x = 182
∠CFI + 145º + 29º = 180º x = 26
∠CFI = 6º If the equal angles are (x – 6)º each,
(iii) ∠AGC = ∠GCI = 29º (alternate angles) (3x + 2)º + 2(x – 6)º = 180º
∠AGF = 180º – ∠AGC 3x + 2 + 2x – 12 = 180
= 180º – 29º 5x = 190
= 151º x = 38
3 105º Note that 3x + 2 = x – 6 will give a negative
Draw a line XY which is parallel to AB and DE value of x.
through point C. 6 x = 40, y = 25
D
B E (3x + 5)º = (5y)º (vertically opposite angles)
A
115º 3x + 5 = 5y

40º 3x – 5y = –5 (eq 1)
(5y)º + (x + y – 10)º = 180º (interior angles)
X Y 6y + x = 190
C x + 6y = 190 (eq 2)
∠DCY = 180º – ∠EDC (interior angles) 3 × (eq 2), 3x + 18y = 570 (eq 3)
= 180º – 115º (eq 3) – (eq 1), 23y = 575
= 65º y = 25
∠BCY = ∠DCY + ∠BCD Sub. in eq 2, x + 150 = 190
= 65º + 40º x = 40
= 105º
∠ABC = ∠BCY = xº (alternate angles) 7 073º
= 105º N

4 70º
X
A C
B B C
P 110º Q
D Y
R S 56º
E
F 120º H A
G By drawing a vertical line XY through B, ∠ABY
Draw lines PQ through D and RS through E = 56º (alternate angles)
which are both parallel to AC. ∠YBC = 90º (XY parallel to north and BC
∠BDQ = 180º – ∠DBC (interior angles) eastwards)
= 180º – 110º ∠ABC = 90º + 56º = 146º
= 70º ABC forms an isosceles triangle as AB = BC.

S Success In Mathematics
© SAP Group Pte Ltd

Solution.indd 9 9/7/2011 2:43:41 PM


(180º – 146º)
___________ –7x ≤ 5(13x + 9) (multiply 5 on both sides)
∠BAC = ​  2 ​     = 17º –7x ≤ 65x + 45
Therefore bearing of C from A –7x – 65x ≤ 45 (subtract 65x on both sides)
= 56º + 17º –72x ≤ 45
= 73º (written as 073º) _____
45
x ≥ ​     ​ (divide –72 on both sides and reverse
(–72)
8 120º the inequality sign)
∠AEC = ∠DEB (vertically opposite angles) __
5
5yº = 3xº x ≥ –​ 8 ​ 
___
3x __
5
y = ​ 5 ​  That means x can take any of the values x ≥ –​ 8  ​
From diagram, ∠FED = 90º for meeting the requirements of the inequality.
__
1
​ 3 ​  (4y + x + 22)º + 3xº = 90º 3 x = 0, x = 1
4y + x + 22 + 9x = 270 ______
4x – 3 _____ x+7
4y + 10x = 248 ​  2 ​   < ​  5 ​  
___
3x ___ 3x 5(4x – 3) < 2(x + 7) (multiplying 10 on both
Substitute y = ​ 5 ​ , 4(​ 5 ​ ) + 10x = 248
sides will remove the fractions and it will be
____
12x
​  5 ​  + 10x = 248 easier to work with integer values. 10 is used
12x + 50x = 1240 because of the denominators are 2 and 5 with
62x = 1240 LCM of 10).
x = 20 20x – 15 < 2x + 14
3x = 60 20x < 2x + 29 (adding 15 on both sides)
18x < 29 (subtracting 2x on both sides)
∠DEB = 3xº = 60º
___
29
∠AED = 180º – 60º (supplementary angles) x < ​ 18 ​ (dividing 18 on both sides)
= 120º x = 0 or 1
Since the x can take only whole number values
___
29
5 LINEAR INEQUALITIES and x < ​ 18 ​ will mean any whole number 1 or
lesser. Note that the answer has an equal sign
1 x ≤ 3 and not an inequality.

x≤3 __
2
• 4 –​ 5  ​< x < 1
Split the inequality into 7 < 15x + 13 and
–1 0 1 2 3
15x + 13 < 28 and solve them separately.
5x + 3 ≤ 18 7 < 15x + 13
5x + 3 – 3 ≤ 18 – 3 (subtract 3 on both sides) 15x + 13 > 7 (simply reversing the inequality)
5x ≤ 15 15x > –6 (subtracting 13 on both sides)
___
5x ___ 15 ___
6
​ 5 ​ ≤ ​  5 ​ (divide 5 on both sides) x > –​ 15  ​ (dividing 15 on both sides)
x≤3 __
2
Therefore any value which is equal to 3 or x > –​ 5  ​
less than 3 will satisfy the requirements of the __
2
The values x > –​ 5 ​  will satisfy only the first
inequality. The solution to the inequality is
inequality and we have to consider the other
represented on the number line as shown.
inequality for completeness.
Note that for x ≤ 3, a small darkened circle is
15x + 13 < 28
placed at x = 3 to indicate that it is included. If
15x < 15 (subtracting 13 on both sides)
x < 3, a small empty circle is placed at x = 3 to
x < 1 (dividing 15 on both sides)
indicate that it is not included.
The values x < 1 will satisfy only the second
5
__ inequality and we have to consider the other
2 x ≥ –​   ​ 
8
____
–7x inequality for completeness.
​  5 ​  ≤ 13x + 9

S10 Success In Mathematics


© SAP Group Pte Ltd

Solution.indd 10 9/7/2011 2:43:41 PM


To satisfy both inequalities, x values must be in Therefore a score of minimum 100 must be
__
2 obtained for an overall pass.
the range x > –​ 5 ​  and x < 1.
__
2 Marks received for 29 correct questions in
Therefore –​ 5 ​  < x < 1. Paper 1 = 29 × 2
= 58
5 2, 5 Taking an average of x marks for the 12 questions in
17 ≤ 13x + 2 < 79 will be split into Paper 2, the total marks for Paper 2 is 12x marks.
17 ≤ 13x + 2 and 13x + 2 < 79. For passing, the condition to be met is
Solve 17 ≤ 13x + 2 58 + 12x ≥ 100
13x + 2 ≥ 17 12x ≥ 42
13x ≥ 15 ___
42
___
15 x ≥ ​ 12 ​ 
x ≥ ​ 13 ​ 
x ≥ 3.5
Solve 13x + 2 < 79
For an integer value of x, a minimum of 4 marks
13x < 77
must be obtained for each of the structured
___
77
x < ​ 13 ​  questions.
___
15 ___
27
Therefore both x ≥ ​ 13 ​  and x < ​ 13 ​  must be 8
–2, 11 ___ ___
satisfied in order that the requirement for the
3 3
Split the inequality into x – 7 < ​√  70 ​ and ​√  70 ​ ≤
inequality be met. Hence x + 7 and solve
___ ___ ___separately.
15 77 3
​ 13 ​ ≤ x < ​ 13 ​  For x – 7 < ​√  70 ​ 
___2 ___
12 x – 7 < 4.1 (finding the cube root of 70 and just
1​ 13  ​ ≤ x < 5 ​ 13 ​  taking one decimal place)
The integer values which meet these conditions x < 11.1
___
are 2, 3, 4 and 5. Therefore, the smallest integer
3
For ​√  70 ​ ≤ x + 7
value is 2 and the largest integer value is 5. 3
___
x + 7 ≥ ​√  70 ​ 
5
___ x + 7 ≥ 4.1
6 – ​    ​ ≤ x < 6
13
x ≥ –2.9
7x – 9 < 5x + 3 ≤ 18x + 5 will be split into Combining both requirements –2.9 ≤ x < 11.1
7x – 9 < 5x + 3 and 5x + 3 ≤ 18x + 5. The smallest and the largest integer values
Solve 7x – 9 < 5x + 3 required are –2 and 11 respectively.
7x < 5x + 12
2x < 12 9 largest (in order): 18, 15, 165, 1089 and
x<6 smallest (in order): –38, –41, –264, 0.
Solve 5x + 3 ≤ 18x + 5 For the largest possible value of x + y, both
5x ≤ 18x + 2 x and y must be as large as possible. For the
–13x ≤ 2 smallest possible value of x + y, both x and y
___2 must be as small as possible.
x ≥ –​ 13  ​ 
So when x = 10 and y = 8 give x + y = 18
To satisfy both requirements, x must be such (largest) while x = –33 and y = –5 give x + y =
___2 –38 (smallest).
that – ​ 13  ​  ≤ x < 6. This is represented on a
number line as follows: For x – y to be the largest possible value, x
must be as large as possible and y as small as
possible. For x – y to be the smallest possible
value, x must be as small as possible and y as
–1 0 1 2 3 4 5 6 large as possible.
___2
–​ 13  ​  So when x = 10 and y = –5 give x – y = 15
(largest) while x = –33 and y = 8 give – 33 – 8
7 4 marks
= –41 (smallest).
Total marks for the two papers = 40 × 2 + 12 × 10
There are negative and positive values for both
= 200
x and y. For the largest possible value of xy,
50 % of 200 = 100
make x and y both positive or both negative

S11 Success In Mathematics


© SAP Group Pte Ltd

Solution.indd 11 9/7/2011 2:43:42 PM


so that the product will be positive. If they are ax × bx = (ab)x according to the standard rules of
of different signs the product will be negative indices.
which need not be considered for the largest
value.

21 x
___
(  )
Similarly 21x ÷ 84x = ​​ ​ 84 ​   ​​​also from the rules
of indices.


xy largest for both positive: 10 × 8 = 80.
xy largest for both negative: (–33) × (–5) = 165.
21 x
___
(  )
Hence (ab)x = ​​ ​ 84 ​   ​​​
Therefore, the largest value of xy = 165 ___
21
The value of ab = ​ 84 ​ 
Since there are positive and negative numbers
to consider for both x and y, take the smallest __
1
= ​ 4 ​ 
negative of one value and multiply with largest
positive of the other. Then make a comparison. 2a(2 + 5a2)
__________
2 ​  5 ​   
Therefore, comparing (–33) × 8 = –264 and 10
× (–5) = –50, the smallest value is –264. __
1 __
4
​ 5 ​ a3 × ​ 3 ​ a2 × 3a–4 + 2a3
For the largest possible value of x2, take the
largest positive and the smallest negative a__3 ___2
4a __
3
= ​ 5 ​  × ​  3 ​  × ​  4   ​ + 2a3 (follow the order of
values of x and compare their squares. The a
squares will be positive regardless of whether operations)
the x value is positive or negative. ___5 __
4a 3
= ​ 15 ​ × ​  4   ​ + 2a3 (reducing the powers of a in
When x is positive, largest x2 = 102 a
= 100 the division)
When x is negative, largest x2 = (–33)2 ___
4a
= ​  5 ​  + 2a3 (reducing the numerical factors in
= 1089
the first term)
Therefore the largest value of x2 = 1089
___ 10a
4a ____3
The smallest value of x2 = 0 when x = 0, since = ​  5 ​  + ​  5 ​   (change the denominator of the
both positive or negative values when squared second term to be common with the first)
give positive values which are all larger than 0.
(4a + 10a3)
_________
= ​  5 ​   (numerator has no like terms to
10 largest and smallest x2 are 64 and 1, largest
and smallest xy are 96 and –15. be added)
For the largest and the smallest values of x2, 2a(2 + 5a2)
_________
take the largest and the smallest positive values = ​  5 ​   
(numerator is factorised)
of x and find the squares. There are no negative
values of x to consider. 3 a = –6, b = 4
When x is positive, largest x2 = 82 Using 22 = 4, 47 = (22)7
= 64 = 214
smallest x2 = 12 = 1 Using 3 = 9, 9 = (32)2
2 2

Largest and smallest x2 are 64 and 1. = 34


For the largest possible value of xy, x and y are Using 6 = (2 × 3)3
3

to be both positive. Both being negative values = 23 × 33


do not occur here. If they are of different signs,

and 3 = 1
0

the product will be negative which will be


considered for the smallest value of xy.
__
(   ) (   ) (  ) (   ) (   )
35 __
25
9
__
30
6
___
2
35 __
25
3
_______
1
​ ​  7 ​   ​× ​ ​  2 ​   ​÷ ​ ​  3 ​   ​= ​ ​  14  ​   ​× ​ ​  4 ​   ​÷ ​  3   3  ​ 
4 (2 × 3 )
xy largest for both positive : 8 × 12 = 96. __
3 _______ 1
= ​  9  ​ ÷ ​  3   3  ​ 
2 (2 × 3 )
For the smallest value of xy, take the smallest
(multiply first two terms according to order of
negative of one value and multiply with largest
operations and reduce the powers)
positive of the other. __
3
–16 < xy ≤ 96 = ​  9  ​ × (23 × 33)
2
Smallest integer value of xy is –15. __
34
= ​  6 ​ 
2
= 2–6 × 34
6 INDICES Therefore a = –6 and b = 4.
__
1 767
____
1 ​   ​  4 – ​   ​ 
4 256

S12 Success In Mathematics


© SAP Group Pte Ltd

Solution.indd 12 9/7/2011 2:43:42 PM


__
5
​   ​  36 = 729
If (3 + a​)–​8​ = 32
The pattern of the last digits is 3, 9, 7, 1, 3, 9,
_______
1
​    __5   ​ = 32  (repeats in a cycle of 4)
​   ​ 
(3 + a​)​8​ Every power which is a multiple of 4 will have
__
5 ___
1 1 as the last digit.
(3 + a)​​ ​8 ​ ​= ​ 32  ​ 
Therefore, 312 will have the last digit 1 followed

___
3 + a = ​​ ​ 32  ​   ​​5​ (  )
1 __​ 8 ​  by 3, 9, 7 for subsequent powers of 3 with the
last digit of 315 being 7.

__
= ​​ ​  5 ​   ​​5​
2 (  )
15 __​ 8 ​  Note that attempts to find the answer by using
a calculator may not work if the power is large,

__
1 8
= ​​ ​ 2 ​   ​​ ​ (  ) as the number of digits may exceed the number
of digits the calculator could accommodate.
____
1
= ​ 256   ​ 
9
4
____
1 Taking 620 = 220 × 320
a = ​ 256   ​ – 3
620 ÷ 310 = 220 × 310 × 310 ÷ 310
____
767
= –​ 256 ​  ___________
220 × 310 × 310
= ​   ​
     (reducing by the
310
5 x14 factor 310 )
=2 ×320 10

_______

(x–2)(x7) –3 ___
​​ ​  9 ​  
x
 
x3 2
) (  )
​​ ​÷ ​​ ​  4 ​   ​​ ​
x


21 = 2
22 = 4

x
___ 3 2
= (x–4)–3 ÷ ​​ ​  4 ​   ​​ ​
x (  )

23 = 8
24 = 16
= x12 ÷ (x–1)2 25 = 32
= x12 ÷ x–2 26 = 64
= x14 The pattern of the last digits is 2, 4, 8, 6, 2, 4,
 (repeats in a cycle of 4)
6 16a14b9
Last digit of 220 is 6 (since 20 is a multiple of 4).
Since anything to the power 0 is 1, (a3b3)0 = 1 Working out a similar pattern for powers of 3,
(we can take a3b3 ≠ 0) we can obtain the pattern of the last digits as 3,
(2a3b3)4 × (b3a–2)–1 ÷ (a3b3)0 9, 7, 1, 3, 9,  (repeats in a cycle of 4)
= (2a3b3)4 × (b3a–2)–1÷ 1 Last digit of 310 is 9 (since 10 is 2 more than a
= (2a3b3)4 × (b3a–2)–1 multiple of 4).
= 24a12b12 × b–3a2 Therefore, the last digit of 220 × 310 is the last
__
a2 digit of 6 × 9, which is 4.
= 24a12b12 × ​  3  ​
b
= 16a14b9 13
___
10 – ​   ​ 
18
5
__ For this type of questions, it should be noted
7 ​   ​ 
6 that 27 and 9 could be written as bases of 3.
__
1 __
2 3
__ __
1 __
2 __
1
(​a​ ​2 ​ ​)(​a​ ​3 ​ ​) ÷ ​√  a ​  = (​a​ ​2 ​ ​)(​a​ ​3 ​ ​) ÷ a​ ​ ​3 ​ ​ 27 = 33 and 9 = 32
__
1 __
2 __
1
= ​a​ ​2 ​ ​+​​ ​3 ​ ​​– ​ ​3 ​ ​ Therefore, the equation can be re-written as,
__
1
__
5 ​   ​ 
= ​a​ ​6 ​ ​ (33x + 2)(33)1 + x = (3​2)​3​
__
2
__
5 (33x + 2)(33 + 3x) =__ ​3​ ​3 ​ ​
x = ​ 6 ​  2
33x + 2 + 3 + __3x = ​3​ ​3 ​ 
2
8 7 ​ 36x + 5 = ​3​ ​3 ​ ​
__
2
Note that: Therefore 6x + 5 = ​ 3 ​ 
31 = 3 18x + 15 = 2
32 = 9 18x = –13
33 = 27 ___
13
34 = 81 x = –​ 18 ​ 
35 = 343

S13 Success In Mathematics


© SAP Group Pte Ltd

Solution.indd 13 9/7/2011 2:43:43 PM


11 3 17 –1.76 × 1011 (in standard form and to 3
Taking 625 = 54 significant figures)
5x + 1 = (54)2x – 5 charge
______
​  mass ​  ratio of an electron
5x + 1 = 58x – 20
__________
–1.6 × 10–19
x + 1 = 8x – 20 = ​   ​ 
9.11 × 10–31
–7x = –21
7x = 21 (  )
____
–1.6
= ​ ​ 9.11 ​  ​× 1012
x=3 = –0.176 × 1012
___4 = –1.76 × 1011 (in standard form and to 3
12 k = –​    ​ , m = –2, n = 17
81 significant figures)
–64x12y3
_________ –64x12y3
___________
​  7 –7   2 ​ 
= ​    ​  18 5 × 102
(–36x y ) (1296x14y–14)
Change 1.25 × 108 to a convenient value with
_____
64 ___ y17
= (–​ 1296  ​) ​  2 ​  perfect cubes so that cube roots can be found
x without a calculator.
___
4 ___ y17 1.25 × 108 = 125 × 106
= (–​ 81  ​)  ​  2 ​ 
x 125 × 106 (it is convenient to have 125 than
___
4 –2 17
= (–​ 81  ​)  x y 1.25 as 125 is a perfect cube and 106 could be
___4 easily cube rooted without a calculator)
k = –​ 81  ​,  m = –2, n = 17 3
__________ 3 __________
​√  (1.25 × 108) ​ 
= ​√  (125 × 106) ​ 
(   ) (  ) (   )
3
__ a __​ 1 ​  __
__ a __​ 1 ​  __ 1 __​ 3 ​  3
____ 3 ____
13 ​a​​ 4 ​ ​+ ​​ ​ b ​   ​​2​+ ​​ ​ b ​   ​​4​+ ​​ ​ b ​   ​​4​ = ​√  125 ​ × ​√  106  ​ 

​( ​a​ ​+ b​ –​4​ )​​( ​a​ ​4 ​ ​+ b​–​2​ )​ = 5 × 102


__
1 __
1 __
1 __
1
​ 2 ​  ​   ​  ​   ​ 

= ​( ​a​ ​2 ​ ​ )(​​  ​a​4​ )​+ (​  ​a​ ​2 ​ ​ )(​​  ​b–​2​ )​+​( ​a​4​ )(​​  ​b–​4​ )​+ (​  ​b–​4​ )(​​  ​b–​2​ )​


__
1 __
1 __
1 __
1 __
1 __
1 __
1 __
1
​   ​  ​   ​  ​   ​  ​   ​  ​   ​  ​   ​ 

__
1 __
1
__
1 __ 1 ___​ 2 ​ ​ __
​ a​ ​a​ ​4 ​ ​ __ 1 7 VARIATIONS
= ​a​ ​2  ​+ ​ 4 ​ ​+ ​  __1  ​ + ​  __1  ​+ ​  __3  ​ 
​ 2 ​  ​ 4 ​  ​   ​ 
​b​ ​ ​b​ ​ ​b​4​
(  ) (  ) (  )
1 3.62
= (​  ​a​ ​4 ​ ​ )​+ ​​ ​    ​  ​​2​+ ​​ ​    ​  ​​4 ​+ ​​ ​    ​  ​​4​
__
3 __
a __​ 1 ​  __ a __​ 1 ​   __ 1 __​ 3 ​ 
y α x3
b b b
y = kx3
14 2 × 10–26 kg When x = 3, y = 4, 4 = k(33)
12 g = 0.012 kg ___
4
k = ​ 27  ​ 
_______
0.012 ___4
Mass of one atom = ​    ​  Subsequently, the value of k is fixed at ​ 27  ​ and
6 × 1023
= 2 × 10–26 kg ___
4x 3
y = ​ 27 ​ 
___
4x3
15 4.01 × 107 m When y = 7, 7 = ​ 27 ​ 
circumference = 2πr 189 = 4x3
= 2π(6.38 × 106) ____
189
x3 = ​  4 ​  
= 40100000 m
x = 3.62 (3 sig. fig.).
(3 significant figures)
= 4.01 × 107 m 2 44.27
___ 1
16 –1.26 × 106 a α ​ ​√__
x   ​​  
First find B – A, ___ k
​  __
a = ​√ x   ​ ​ 
B – A = 1.3 × 106 – 3.97 × 107 ____ k
When x = 49, a = 6, 6 = ​  ___    ​ 
= 1.3 × 106 – 39.7 × 106 ​ 49 ​ 

= –38.4 × 106 __k
6 = 7​   ​ 
B(B – A) ____________________
________ (1.3 × 106) (–38.4 × 106)
​  A     
​ = ​     ​
   k = 42
3.97 × 107
= –1.26 × 106 (to 3 significant figures) Subsequently, the value of k is fixed at 42 and
___
42__
a = ​ √
​ x   ​ ​ 

S14 Success In Mathematics


© SAP Group Pte Ltd

Solution.indd 14 9/7/2011 2:43:43 PM


___9 ______
42 k
______
When x = ​ 10  ​,  a = ​  _____    ​     ​ 
Since ​ 24000 = 101000,
√ (  )
 ​   ​ ___9
​ 10  ​   ​ ​ 
___1
____
42 P1 = ​ 0.8   ​ (101000)
= ​  ___  ​ 
​ 0.9 ​ 

= 126250 Pa
= 44.27 (2 decimal places)
6
768 grams
3 7.47 (2 decimal places)
m α r3
__1
A α ​  3  ​  m = kr3
x
__k When r = 5, m = 187.5
A = ​  3  ​ 
x 187.5 = k(5)3
__
5 __5 __ k
When x = 2, A = ​ 8 ​ , ​ 8 ​  = ​  3  ​  187.5 = k(125)
2
__
5 __k
​ 8  ​= 8​   ​  _____
187.5
k = ​  125 ​ 
k=5
= 1.5
Subsequently, the value of k is fixed at 5 and
__
5 m = 1.5r3
A = ​  3  ​  When r = 8,
x
____
3 ____ 3 __
5 m = 1.5(8)3
When A = ​ 250   ​, ​ 250   ​ = ​  3  ​ 
x
3x3 = 1250 = 768 grams
_____
1250 7 300%
x3 = ​  3 ​   
__1
x = 7.47 (2 decimal places) R α L and R α ​ A ​  
__
L
4 60 Combining the two relationships, R α A
​   ​  
y α (x – 4)2 ___
kL
R = ​ A ​ 
y = k(x – 4)2
kL1
___ kL2
___
When x = 9, y = 15 Let R1 = ​ A  ​ and R2 = ​ A  ​ 
15 = k(9 – 4)2 1 2

15 = k(5)2 When L2 = 2L1 (length doubled) and


15 = k(25) __
1
___
15 __ 3 A1 = ​ 2 ​  A2 (area halved)
k = ​ 25 ​ = ​ 5 ​ 
__
3 k(2L1)
______
Subsequently, the value of k is fixed at ​ 5 ​  and R2 = ​  __
1  ​ 
__
3 (​ 2 ​  A1)
y = ​ 5 ​ (x – 4)2
__
3 4kL1
____
When x = 14, y = ​ 5  ​(14 – 4)2 = ​  A  ​   
__
3 1
= ​ 5 ​ (100)
= 4 R1
= 60 Resistance becomes 4 times.
5 126250 Pa (4 R1 – R1)
_________
Percentage increase = ​  R  ​   
× 100%
__1
P α ​ V ​   1

(3 R1)
_____
__k = ​  R  ​ 
 × 100%
P=V ​   ​   1
When P = 101000, V = 24000 = 3 × 100%
______
k = 300%
101000 = 24000
​     ​ 
When V is reduced by 20%, V becomes 0.8V 8 –18.4%
and P becomes P1 ___
1
F α ​  2  ​ 
d 

____k
P1 = ​ 0.8V
    
___

1 ______ k
​= ​ 0.8   ​ ​ 24000 )
​     ​  ​.
k
___
F = ​  2  ​ 
d 

S15 Success In Mathematics


© SAP Group Pte Ltd

Solution.indd 15 9/7/2011 2:43:44 PM




F changes to F1 when d changes to d1
F1 = F + 0.5F (increase of 50%)

__
1
) 2
__
1
as ​​ ​ 5 ​ x + 3y  ​​ ​, where a = ​ 5  ​x and b = 3y.

= 1.5F Using the identity (a + b)2 = a2 + 2ab + b2,


___
k
F1 = ​  2  ​ (equation 1)
d1
__
1
(  )
2
___1 __
6
​​ ​ 5 ​ x + 3y  ​ ​= ​ 25  ​x  2 + ​ 5 ​  xy + 9y2

___k 3
497
F = ​  2  ​ (equation 2)
d  Using the identity (a – b)2 = a2 – 2ab + b2,
F1 ___
__ d 2
Dividing eq1 by eq 2, ​ F ​ = ​  2  ​  (x – 3)2 = x2 – 6x + 9,
d1
____
1.5F
​  F    (  )
__d 2
 ​= ​​ ​    ​   ​​ ​
d1


If 247 009 = x2 – 6x + 9,
(Note that the nearest obvious perfect square is
1.5 = ​​( ​    ​  )​​ ​
__d 2
250 000 = 5002)

d 1 247 000 = x2 – 6x

Therefore ​​( ​   ​  )​​ ​= ​ 1.5   ​ 


d
__ 1
2
___1 Since 247 000 = 25 0000 – 3000

d = 5002 – 6(500)
__
2 Comparing 5002 – 6(500) with x2 – 6x, x = 500
= ​ 3 ​ 
d1
__ If 247 009 = x2 – 6x + 9 = (x – 3)2,
_______
 ​   ​ = 0.816
d ​√247 009 ​ =x–3
_______
d1 = 0.816d ​√247 009 ​ = 500 – 3
_______
(0.816d – d)
___________ Therefore √ ​ 247 009 ​ 
= 497
Percentage change = ​   ​   
× 100%
d
= –18.4% 4 9 × 1014
30 000 002 = 30 000 000 + 2
29 999 998 = 30 000 000 – 2
8 EXPANSION AND FACTORISATION Therefore 30 000 002 × 29 999 998
= (30 000 000 + 2 )(30 000 000 – 2)
1 2(25x2 + 16y2) Using the identity (a + b)(a – b) = a2 – b2
(5x + 4y)2 + (5x – 4y)2 (30 000 000 + 2)(30 000 000 – 2) = 30 000 0002 – 4
(remember that (a + b)2 is not a2 + b2) Therefore if 30 000 002 × 29 999 998 = N – 4,
= (5x + 4y) (5x + 4y) + (5x – 4y) (5x – 4y) N = 30 000 0002
(using the expansion for (a + b)(c + d) = ac + = (3 × 107)2
ad + bc + bd taking the sign before the term = 9 × 1014
as part of the term) 5 49x2 – 9y2
= (25x2 + 20xy + 20xy + 16y2) + (25x2 – 20xy While the expansion (a + b)(c + d) = ac +
– 20xy + 16y2) ad + bc + bd could be applied to obtain the
= (25x2 + 40xy + 16y2) + (25x2 – 40xy + 16y2) expansion, the identity (a + b)(a – b) = a2 – b2
(note that the first bracket could be obtained would be easier to use.
Using the identity (a + b)(a – b) = a2 – b2
by using the identity for (a + b)2 = a2 + 2ab +
Where a = –7x and b = 3y,
b2 and the second bracket could be obtained (–7x – 3y)(–7x + 3y) = (–7x)2 – (3y)2
by the identity (a – b)2 = a2 – 2ab + b2) = 49x2 – 9y2
= 25x2 + 40xy + 16y2 + 25x2 – 40xy + 16y2
6
2y(5x + 4z)(5x – 4z)
= 50x2 + 32y2
The highest common factor of the terms 50x2y and
= 2(25x2 + 16y2) –32yz2 is 2y found by the method as follows:
(where possible, answers should be in the 2 50x2y and –32yz2
factorised form unless required otherwise) y 25x2y and –16yz2 (divided by 2 which is a
____
1 6
__ common factor)
2 ​   2 ​ + ​ 5 ​  xy + 9y2
25x 25x2 and –16z2 (divided by y which is a
For expansion, the expression should be seen common factor)

S16 Success In Mathematics


© SAP Group Pte Ltd

Solution.indd 16 9/7/2011 2:43:44 PM


HCF = 2y
11 x(x – 1)(2x + 3)
Dividing by 2y, 50x2y – 32yz2 = 2y(25x2 – 16z2)
First factorise the common factor of the three
Using the identity (a + b)(a – b) = a2 – b2,
terms.
25x2 – 16z2 = (5x + 4z)(5x – 4z)
Therefore 50x2y – 32yz2 = 2y(5x + 4z)(5x – 4z) 2x3 + x2 – 3x = x(2x2 + x – 3)

7 10x2y + 20xy3 – 6x – 12y2 Next factorise 2x2 + x – 3


Using the expansion (a + b)(c + d) = ac + ad + (You may want to confirm whether it can
bc + bd, be factorised by finding the b2 – 4ac in the
(5xy – 3)(2x + 4y2) = 10x2y + 20xy3 – 6x – 12y2 expression. b2 – 4ac = 12 – 4(2)(–3) = 25. Since
The expansion cannot be simplified further and 25 is a perfect square, the expression can be
could only be factorised back to the original factorised.)
form. factor pairs for 2x2 is x and 2x (take only the
positive forms)
8 3(2x – 3y)(2x – 3y) factor pairs for –3 are +1 and –3, –3 and +1
First note that there is a common factor of 3 in (here the positive and negative factors have to
all the three terms. be taken)
12x2 – 36xy + 27y2 = 3(4x2 – 12xy + 9y2) Try using the factor pairs x and 2x with +1 and
The first term of 4x2 – 12xy + 9y2 is 4x2, a perfect –3
square of 2x and the third term is 9y2, a perfect Factor pair Factor pair Cross-multiplied
square of 3y. The middle term is –12xy is such of 2x2 of –3 products
that –12xy = –2(2x)(3y).
Therefore, the expression 4x2 – 12xy + 9y2 can cross
x multiply
be factorised using the identity (a – b)2 = a2 +1 +2x
– 2ab + b2 as (2x – 3y)(2x – 3y)
Therefore 12x2 – 36xy + 27y2 = 3(2x – 3y)(2x – 3y) 2x –3 –3x

9 (ax – b)(x – 1) Product: 2x2 Product : –3 Sum : –x


Try factorising the first pair of terms and then Therefore (x + 1)(2x – 3) is not the required
the second pair of terms. factorisation. This process has to be repeated
ax2 – bx – ax + b = x(ax – b) – ax + b until the sum of the cross products is the middle
(the second pair of terms cannot be factorised) term ie. +x in this case.
Then try rearranging the terms such that the Using the factors x and 2x with –1 and +3.
first and the second pair of terms could be
Factor pair Factor pair Cross-multiplied
factorised. of 2x2 of –3 products
Rearranging the expression,
ax2 – b – ax + b cross
x multiply
= ax2 – ax – bx + b –1 –2x
= ax(x – 1) – b(x – 1)
Taking the expression as two terms of ax(x 2x +3 +3x
– 1) and –b(x – 1), it can be seen that there is a
Product: 2x 2
Product : –3 Sum : x
common factor (x – 1).
Factorising again, we obtain the factorisation Since the sum of the cross products is x, the
of the expression which is (ax – b)(x – 1). factorisation is
2x2 + x – 3 = (x – 1)(2x + 3)
10 25a2 – 4b2
Therefore 2x3 + x2 – 3x = x(x – 1)(2x + 3).
625a4 – 16b4
Using the identity (a + b)(a – b) = a2 – b2 12 p = a2, q = 5a–1
(5a + 2b)(5a – 2b) = 25a2 – 4b2
From indices it should be noted that am + 2 = am ×
Using the identity (a + b)(a – b) = a2 – b2, to
simplify the given expression, a2 and am – 1 = am × a–1
(25a2 + 4b2)(5a + 2b)(5a – 2b). Therefore am + 2 + 5am – 1 = am × a2 + 5 × am × a–1
(25a2 + 4b2)(5a + 2b)(5a – 2b) Here the common factor in the two terms is am.
= (25a2 + 4b2)(25a2 – 4b2) am + 2 + 5am – 1 = am (a2 + 5a–1)
= 625a4 – 16b4 __
5
p = a2, q = ​ a  ​= 5a–1

S17 Success In Mathematics


© SAP Group Pte Ltd

Solution.indd 17 9/7/2011 2:43:45 PM


__
1
2 (  ) 2
13 ​​ ​   ​ x + 2  ​​ ​ 3
__ __
4
2 ​    ​or –​   ​ 
2 3
__
1 __
1
The first term ​ 4 ​ x2 is a perfect square of ​ 2 ​ x and 9 __
__ 5
​   ​  or ​   ​ 
2 3
the third term 4 is a perfect square of 2.
__
1 Factor pairs of 6x2 are x and 6x,
Using the identity (a + b)2 = a2 + 2ab + b2, a = ​ 2 ​ x 2x and 3x (taking only the
and b = 2. positive factors).
(  )
__
1
The middle term is 2x = 2​ ​ 2  ​x ​(2) is the 2ab Factor pairs of –12 are 1 and –12,
required in the identity. –1 and 12,

__
1

__
1
)(  __
1
)
Therefore ​ 4 ​ x2 + 2x + 4 = ​ ​ 2 ​ x + 2  ​​ ​ 2 ​ x + 2  ​

2 and –6,
–2 and 6,

__
1
)
2
= ​​ ​ 2 ​ x + 2  ​​ ​ 3 and –4,
14 k = 3, n = 2 –3 and 4.
Curved area of a hemisphere Choose the appropriate factor pairs for both the
__
1 terms by trial and error. The correct factor pairs
= ​ 2 ​  (curved area of a sphere) are as shown:
__
1 Factor pair Factor pair Cross-multiplied
= ​ 2 ​  (4πr2) = 2πr2
of 6x2 of –12 products
Flat surface of a hemisphere = πr2 (circular area
cross
of radius r)
2x multiply –3
Total surface area = 2πr2 + πr2 –9x
= 3πr2
3x 4 +8x
k = 3 and n = 2
15 7(x2 + 1)(x + 1) Product: 6x 2
Product: –12 Sum: –x
First factorise the common factor of all the Therefore, 6x – x – 12 = (2x – 3)(3x + 4)
2

terms, For (2x – 3)(3x + 4) = 0, the product of the


7x3 + 7x2 + 7x + 7 = 7(x3 + x2 + x + 1) factors (2x – 3) and (3x + 4) is 0.
Try factorising the first pair of terms and then So 2x – 3 = 0 or 3x + 4 = 0. (two linear equations)
the second pair of terms in the expression x3 + __
3 __
4
For 2x – 3 = 0, x = ​ 2 ​  and for 3x + 4 = 0, x = –​ 3  ​
x2 + x + 1.
__
3 __
4
x3 + x2 + x + 1= x2(x + 1) + (x + 1) x = ​ 2  ​or – ​ 3 ​ (this means either of these values of
Taking the expression as two terms x2(x + 1) x when substituted into the quadratic expression
and (x + 1), the common factor is (x + 1). will give a value 0).
The expression x3 + x2 + x + 1 then becomes To solve the second part of the question, one
(x2 + 1)(x + 1). might proceed to expand and simplify the
Therefore 7x3 + 7x2 + 7x + 7 = 7(x2 + 1)(x + 1). expression to obtain the solutions. However,
a comparison will show that the expression is
similar to that in the first part, with the x replaced
9 QUADRATIC EQUATIONS by y – 3. The phrase ‘hence or otherwise’
suggests strongly that the second part should
1 8 be seen as a continuation of the question and
First step is to factorise x2 – 16x + 64. not to be solved as a stand-alone question.
The first term is x2, a perfect square of x and Therefore taking x = y – 3,
third term is 64, also a perfect square of 8. 6x2 – x – 12 = 6(y – 3)2 – (y – 3) – 12
Taking a = x and b = 8 and using (a – b)2 = a2 The solutions for 6x2 – x – 12 = 0 could then
– 2ab + b2 be modified for the equation 6(y – 3)2 – (y – 3)
The middle term is –2ab = –2(x)(8) – 12 = 0.
= –16x __
3 __
4 __
3 __
4
Since x = ​ 2 ​  or –​ 3 ​ , y – 3 = ​ 2 ​  or –​ 3 ​ .
Therefore, the expression can be factorised as __
3 __
4
x2 – 16x + 64 = (x – 8)2. y = ​ 2 ​  + 3 or –​ 3 ​  + 3
(x – 8)2 = 0 __
9 __ 5
= ​ 2 ​  or ​ 3  ​
x–8=0
x=8

S18 Success In Mathematics


© SAP Group Pte Ltd

Solution.indd 18 9/7/2011 2:43:46 PM


7
__ 6 a = 4, b = –1
3 –1 or ​   ​ 
3
Using a2 – b2 = (a + b)(a – b)
Multiply both sides of the equation by 4 to
15 = 3(a – b)
remove the fractions as it would be easier to
___
15
work with integers. a – b = ​  3 ​ 
__
3 __
7 =5
​ 4 ​ x2 = x + ​ 4 ​  becomes 3x2 = 4x + 7
If a + b = 3 and a – b = 5,
Re-arrange the equation 3x2 = 4x + 7 to 3x2 – 4x (a + b) + (a – b) = 3 + 5 (according to simultaneous
– 7 = 0. equations)
The expression 3x2 – 4x – 7 is factorisable as 2a = 8
the value b2 – 4ac = 100, a perfect square. a=4
Factorise 3x2 – 4x – 7 into (x + 1)(3x – 7) using If a = 4, using a + b = 3, b = –1.
the trial error or ‘cross method’.
As (x + 1)(3x – 7) = 0 7 2.77 or 5.90
x + 1 = 0 or 3x – 7 = 0 Expanding and simplifying the equation into a
__
7 quadratic equation as follows:
x = –1 or ​ 3  ​ 3(x – 4)2 + 5(x + 2) – 7x = 9
3
__ 3(x – 4)(x – 4) + 5(x + 2) – 7x = 9
4 0 or –​   ​  3(x2 – 8x + 16) + 5(x + 2) – 7x – 9 = 0
2
First factorise 8x3 + 24x2 + 18x into 2x(4x2 + 3x2 – 24x + 48 + 5x + 10 – 7x – 9 = 0
12x + 9) to take out the obvious common factor 3x2 – 26x + 49 = 0
of the three terms. This equation does not conform to any of the
Next it can be seen that the expression (4x2 + identities and also could not be factorised as the
12x + 9) conforms to the identity (a + b)2 = a2 + value of b2 – 4ac = 88, which is not a perfect
2ab + b2 where a = 2x and b = 3. square. Hence, the quadratic formula has to be
The 2ab term is 2(2x)(3) = 12x. used. The question already indicates that the
Therefore 8x3 + 24x2 + 18x = 2x(4x2 + 12x + 9) answer has to be given correct to two decimal
= 2x(2x + 3)(2x + 3) places which can be taken as a hint that the
So if 2x(2x + 3)(2x + 3) = 0, equation may not be factorised.
x (2x + 3)2 = 0 Using the quadratic formula where a = 3,
__
3 b = –26 and c = 49,
_______________
x = 0 or –​ 2 ​  (repeated roots)  


–(–26)
x = ​ 
√    
±​ (–26) 2
– 4 (3)  
________________________
 
 ​   
(49) ​
49
___ ___
5 ​    ​or – ​    ​ 
1 ___ 2(3) ___ ___
40 40 26±​ √  88 ​  _________
________ 26 +  ​√  88 ​  ________
26 – ​ √  88 ​ 
= ​  6 ​   = ​  6  ​    or ​  6  ​  
 
The solution to this equation need not be found
(  __
)
3 2
by expanding ​​ x – ​ 5  ​  ​​ ​, although the answer
= 2.77 or 5.90 (2 decimal places).
7
__
obtained would be the same. 8 –2 or ​   ​ 
3
It could be found in a much easier and shorter In this question, it is not the value of x or y that
(  )
__
3 2
way by isolating and finding the root of ​​ x – ​ 5 ​   ​​ ​. is to be found. Instead, it is the ratio of ​ y ​.
__x

(  ) __
3 2 ___ 25
2​​ x – ​ 5 ​   ​​ ​= ​ 32 ​  The equation can be divided by y2 to give an
expression,
​​( x – ​ 5 ​  )​ ​ = 64
__
3 2
___
25
___
25
​   ​ 
__
5
(  )
__x 2 __x
(  ) __x
3​​ ​ y ​  ​ ​ – y​  ​ – 14 = 0 ​ let ​ y ​= z  ​
​ 64 ​ is a perfect square of ​ 8 ​ . 3z2 – z – 14 = 0 (this is now a quadratic equation
__
3 __ 5 of z which needs to be found)
Therefore x – ​ 5 ​ = ±​ 8 ​ (both positive and negative
The expression 3z2 – z – 14 can be factorised as
values have to be considered as the squaring of the b2 – 4ac value is 169, a perfect square.
___
25 Using the trial-and-error method,
both values will give ​ 64 ​ )
__
5 __ 3 __
5 __ 3 3z2 – z – 14 = (z + 2)(3z – 7)
x = ​ 8 ​  + ​ 5 ​  or –​ 8 ​  + ​ 5 ​ 
For (z + 2)(3z – 7) = 0
___
49 ___
1 __
7
= ​ 40 ​ or –​ 40  ​  z = –2 or ​ 3 ​ 

S19 Success In Mathematics


© SAP Group Pte Ltd

Solution.indd 19 9/7/2011 2:43:46 PM


9
__ 12 8.83 or –2.83
9 ​ 7 ​ 
For completing the square, it is a requirement
147x2 – 243y2 = 0 that the equation must be such that the coefficient
147x2 = 243y2 of x2 must be 1, which in this case is met.
x2 ____
__ 243 x2 – 6x – 25 = 0
​  2 ​  = ​ 147 ​ 
y x2 – 6x = 25
(  )
__x 2 ___ 81
​ ​ y ​  ​ = ​ 49 ​  For the next step, take the positive value of the
coefficient of x, half it and then square it.
__x __
9
​ y ​= ± ​ 7 ​  Coefficient of x is –6.
__x Positive value is 6 (disregard the negative sign)
Since x and y are both greater than 0, ​ y ​must be
__x __ 9 Halving it will give 3
positive, ​ y ​= ​ 7 ​ . Squaring it will give 9
Add 9 to both sides of the equation,
10 –4.86 or 12.36 x2 – 6x + 9 = 25 + 9
Multiplying throughout by 10 (LCM of 2 and 5, x2 – 6x + 9 = 34
the denominators of the fractions) the quadratic The expression x2 – 6x + 9 conforms to the
equation is written with integer coefficients identity (a – b)2 = a2 – 2ab + b2
which makes it easier to be solved. where a = x and b = 3 with –2ab = –6x.
__
1 __
3 Therefore,___(x – 3)2 = 34
​ 5 ​ x2 – ​ 2 ​ x – 12 = 0 becomes 2x2 – 15x – 120 = 0
Solving 2x2 – 15x – 120 = 0 gives the same answer x – 3 = ±​√34 ​ (both positive and negative values
__
1 __
3 have to be___taken) ___
as solving the equation ​ 5 ​ x2 – ​ 2 ​ x – 12 = 0.
x=3+√ ​ 34 ​ or 3 – ​√34 ​ 
2x2 – 15x – 120 = 0 x = 8.83 or –2.83
The equation cannot be solved by factorisation The same values will be obtained using the
and the quadratic formula has to be used. quadratic formula. Actually the quadratic
Using the quadratic formula where a = 2, formula is a one-step process of completing the
b = –15 and c = –120, square.

_________________
x = ​ 
–(–15) ± √(–15)
​ 2
– 4(2) (–120) ​
__________________________
 
     ​
  
   13 0.622 or –5.62
2(2) __
5
_____ x = – ​ 2  ​

​ 1185 ​ 
15 ± √
__________
= ​   ​ 
4 _____  
_____

__
5 ___ 39
)
Minimum point ​ – ​ 2 ​  , – ​  4 ​   ​
​ 1185 ​  __________
15 + √
__________ ​ 1185 ​ 
15 – √ For completing the square, the equation must
= ​  4 ​   
or ​  4 ​   
be such that the coefficient of x2 must be 1,
= –4.86 or 12.36 (2 decimal places) which in this case is not met.
Dividing the equation throughout by 2 to make
11 60 cm
the coefficient of x2 1,
Let the breadth be x cm and the length be __
7
(x + 4) cm. 2x2 + 10x – 7 = 0 becomes x2 + 5x – ​ 2 ​  = 0
Area of the rectangle = x(x + 4) cm2 __
7
x2 + 5x – ​ 2 ​  = 0
Since x(x + 4) = 221 __
7
x2 + 4x – 221 = 0 x2 + 5x = ​ 2 ​ 
Since x2 + 4x – 221 can be factorised (b2 – 4ac For the next step, take the positive value of the
= 900, a perfect square), obtain the factors by coefficient of x, half it and square it.
trial-and-error. Coefficient of x is 5.
x2 + 4x – 221 = (x – 13)(x + 17) Positive value is 5 (already positive 5)
Therefore (x – 13)(x + 17) = 0 __
5
x = 13 or –17 (–17 is rejected since side of a Halving it will give ​ 2 ​ 
rectangle cannot be negative) ___
25
Squaring it will give ​  4 ​ 
breadth = 13 cm, length = 13 + 4 ___
25
= 17 cm Add ​  4 ​ to both sides of the equation.
Perimeter = 2(13 + 17) ___
25 __ 7 ___ 25
x2 + 5x + ​  4 ​ = ​ 2 ​  + ​  4 ​ 
= 60 cm

S20 Success In Mathematics


© SAP Group Pte Ltd

Solution.indd 20 9/7/2011 2:43:47 PM


___
25 16 (i) 4 m
The expression x2 + 5x + ​  4 ​  conforms to the
From the graph, the maximum height
identity (a + b)2 = a2 + 2ab + b2, where a = x reached is about 4 m.
__
5
and b = ​ 2 ​  with 2ab = 5x. t 0 0.5 1.0 1.5 1.6
__
5 ___
39 h 1.5 3.75 3.5 0.75 –0.1
Therefore, (x + ​ 2  ​)2 = ​  4 ​ 
___ (Refer to Graph 2, page S22)

__
5
√ ___
39
x + ​ 2 ​ = ±​ ​  4 ​ ​  (both positive and negative values (ii) 1.0 s or 0.4 s
Answer can be obtained by using the graph
have to be taken)
___ ___
or by calculation.

__
5
√ ___
39
√ __
5 ___
39
x = –​ 2 ​  + ​ ​  4 ​ ​  or –​ 2 ​  – ​ ​  4 ​ ​   3.5 = 7t – 5t2 + 1.5
5t2 – 7t + 2 = 0
x = 0.622 or –5.62
__
5 Factorising 5t2 – 7t + 2,
The quadratic curve can be written as y = (x + ​ 2 ​ )2
(t – 1)(5t – 2) = 0
___
39 __
2
– ​  4 ​ and compared with y = a(x – h)2 + k from t = 1 or ​ 5 ​ 
the topic on Functions and Graphs. Stone is 3.5 m above ground when t = 1s or
__
5 0.4 s.
The line of symmetry is x = –​ 2 ​ and the minimum

(  __
5 ___
) 39
point is ​ –​ 2  ​, –​  4 ​   ​.
(iii) 1.4 s
Answer can be obtained by using the graph
The graph of the expression y = 2x2 + 10x – 7 is or by calculation.
as follows. From the expression h = 7t – 5t2 + 1.5,
(Refer to Graph 1, page S22) when t = 0, h = 1.5 m.
So 1.5 = 7t – 5t2 + 1.5,
14 4
___
AB ___ BC 7t – 5t2 = 0
Since ∆ABC is similar to ∆PQR, ​ PQ   ​= ​ QR  ​ t(7 – 5t) = 0
______
2x – 5 ______x+3 __
7
​ 2x – 2 ​ = ​ 3x + 2   ​  t = 0 or t = ​ 5 ​ 
Therefore stone is again at h = 1.5 when
(2x – 5)(3x + 2) = (2x – 2)(x + 3) (by cross
t = 1.4 s.
multiplication)
6x2 + 4x – 15x – 10 = 2x2 + 6x – 2x – 6 (iv) 1.59 s
4x2 – 15x – 4 = 0 Answer can be obtained by using the graph
Solving 4x2 – 15x – 4 = 0, or by calculation.
(x – 4)(4x + 1) = 0 When stone hits the ground h = 0,
__
1 __ 1 7t – 5t2 + 1.5 = 0
x = 4 or –​ 4 ​ (–​ 4 ​ is rejected as sides of the triangle Re-arranging by multiplying throughout –1,
cannot be negative) 5t2 – 7t – 1.5 = 0
Solving by quadratic formula, t = 1.59 or
15 17, 19 –0.19 (reject negative value of t as time
Let the numbers be x and x + 2 (the smaller must be positive).
number is x and the next odd number must be 2 t = 1.59 s
more than that).
The product of the two numbers is x(x + 2) 17 1.5 m
algebraically. xm
If x(x + 2) = 323,
x2 + 2x = 323 9m
xm xm
x2 + 2x – 323 = 0
Factorising x2 + 2x – 323 by trial-and-error 14 m
method and solving, xm
(x – 17)(x + 19) = 0
x = 17 or –19 (–19 is rejected) Since the garden forms the inner rectangle, the
The other number is 19. The answer could also garden and the path form the outer rectangle.
be obtained by straightforward trial-and-error Length of the outer rectangle = (14 + 2x) m
without systematically applying the quadratic Breadth of the outer rectangle = (9 + 2x) m
equation. Area of the outer rectangle = (14 + 2x) (9 + 2x) m2

S21 Success In Mathematics


© SAP Group Pte Ltd

Solution.indd 21 9/7/2011 2:43:47 PM


Graph 1
Topic 9, question 13.

20

10

(0.622, 0)
(–5.62, 0)
x
–5 –4 –3 –2 –1 1 2
(0, –3.5)
y-intercept

–10
minimum point ​
( –__​ 52 ​, –​ __394  ​ )​
x = –2.5 (line of symmetry)

Graph 2
Topic 9, question 16(i). maximum height
h

h = 3.5

t (sec)
0 0.4 0.5 1.0 1.4 1.5 1.6

S22 Success In Mathematics


© SAP Group Pte Ltd

Solution.indd 22 9/7/2011 2:43:48 PM


Therefore (14 + 2x) (9 + 2x) = 204
126 + 28x + 18x + 4x2 = 204 10 ALGEBRAIC MANIPULATIONS
Re-arranging to form a quadratic equation, 3a2
______
4x2 + 46x – 78 = 0 1 ​    ​ 
5a + 1
2x2 + 23x – 39 = 0 _________
15a 3
+ 3a2
Factorising 2x2 + 23x – 39 by trial-and-error ​   ​ 
(5a + 1)2 
method and solving, 3a 2
(5a + 1)
__________
(x + 13)(2x – 3) = 0 = ​   ​ 
 
(factorising the numerator)
(5a + 1)2
__
3
x = –13 or ​ 2 ​  (–13 is rejected as length cannot 3a2(5a + 1)
__________
= ​   ​ 
 (reducing the common factors)
be a negative value) x = 1.5 (5a + 1)2
(5a + 1)
______
3a2
18 18 = ​ 5a +  1 
 ​
Cost of each brand A shirt = $x
Cost of each brand B shirt = $(x + 6) y (x2 + y2)
_________
2 ​  2  ​ 
For $360, the number of each brand that could 2x – 3y2
be bought: __x __y x2 + y2
______
____
360 ​ 
y x 

______ + ​   
​ ​ 
________xy   
​ 
Number of brand A shirts = ​  x   ​  ​ ___
2x __  ​ =
3 2x – 3y 
    ​  2 2  ​ 
_____
360 ​  2 ​ – ​ x ​ ​ ________ 2 ​   
Number of brand B shirts = ​ x + 6  ​  y xy
__x __y __ y2
x2 __
More brand A shirts could be bought for the For the numerator: ​ y ​+ ​ x ​= ​ xy ​ + ​ xy ​ 
same amount of money as the price is lower. (make the denominators the
____
360 _____ 360 same by taking LCM of y, x
Therefore ​  x   ​ – ​ x + 6  ​ = 5 (first write the as xy)
algebraic expression as a single fraction) 3y2
___
2x2 ___
360(x + 6) _______ For the denominator: ​  2 ​ – ​  2 ​ 
_________ 360x xy xy
​   ​ 
 
– ​    ​ 
=5
x(x + 6) x(x + 6) (make the denominators the
360(x + 6) – 360x
_______________ same by taking LCM of y2,
​    
 ​  =5 x as xy2)
x(x + 6)
(when the denominator of a fraction is
_________________
360x + 2160 – 360x
​       = 5
 ​ multiplied by a factor, the numerator also has
x(x + 6)
to multiplied by the same factor for the fraction
_______
2160 value to be unchanged)
​    ​ 
=5
x(x + 6)
x2 + y2
______
2160 = 5x(x + 6) (cross multiplying) ​  xy   
________ ​  ______
x2 + y2 ________
2x2 – 3y2
​  2   
 ​ = ​  xy   
​ ÷ ​   ​  
2160 = 5x2 + 30x 2x – 3y2
________ xy2
​   ​  
 
Re-arranging 5x2 + 30x – 2160 = 0 xy2
x2 + y2 ________
______ xy2
y
x2 + 6x – 432 = 0 = ​  xy   ​ × ​  2   2 ​ 
(dividing throughout by 6 which 2x – 3y
is a common factor) y(x2 + y2)
________
= ​  2  ​ 
x2 + 6x – 432 can be factorised as b2 – 4ac value 2x – 3y2
is 1764, a perfect square.
Using the trial-and-error method, ______
2
3 ​  13  2 ​ 
3a b
x2 + 6x – 432


= (x – 18)(x + 24).
For (x – 18)(x + 24) = 0
____
2a
​  3 
3ab
2
b
(  )
___
a3 2 2a ____
2
 ​÷ ​​ ​  –4   ​  ​​ ​= ​  3 
a 3ab
b
 ​÷ (a7)2 (using indices a3 ÷
a–4 = a7)
x = 18 or –24 (–24 is rejected since the price
____
2a 2
b a___ 14
cannot be a negative value). = ​  3   ​÷ ​  1 ​ 
3ab
For those who consider the trial-and-error
2a2b ___
____ 1
process lengthy may use the quadratic formula = ​  3 ​ × ​  14  ​ 
3ab a
and the same answer will be obtained. _____
2
= ​  13  2  ​
3a b

S23 Success In Mathematics


© SAP Group Pte Ltd

Solution.indd 23 9/7/2011 2:43:48 PM


8 (b + c)
________ (b – a)2 _____
_______ a + b _____
b–a a + b _____
_____ b–a
4 – ​   ​    ​   ​ 
÷ ​    
​× ​    ​= 1 ÷ ​    
​× ​   
 ​
3a (a – b)2 a – b b + a a–b b+a
8 (b + c)
–16a3 (b + c)3 –8 (b + c) (Since (b – a)2 = (a – b)2)
​  _________ ​= ​ _______
   3a   
 ​
6a (b + c)
4 2
a – b _____
_____ b–a
= ​   
 ​× ​   
 ​
3a
a+b a+b
5 a+1 a – b _____
_____ a–b
= – ​   
 ​× ​   
 ​
Factorise the numerator 2a2 + 3a + 1 into a+b a+b
(a + 1)(2a + 1) by using the trial and error cross
method shown in the topic Expansion and

a–b
= – ​​ ​ 
a+b( 
_____
  )
2
 ​  ​​ ​

Factorisation. The order of operations has to be followed by


(a + 1)(2a + 1) carrying out the division first.
​ ___________
2a + 3a + 1 ____________
2

2a + 1   ​ 
= ​  2a + 1  
   ​ (reduce the common
factors) (x + 5)(x – y)
___________
=a+1 9 ​      ​ 
4(x + 4)
___
1 The numerator and denominator of the first
6 ​   5 ​ 
8x term can be factorised.
The value xa + 2 is xa × x2 using indices x2 + 9x + 20 = (x + 4)(x + 5)
The value xa + 7 is xa × x7 using indices x2 + 8x + 16 = (x + 4)(x + 4)
Therefore, The denominator of the second term is also

1
5(xa)(x2)
factorised.
______
5xa + 2
​ 40xa + 7 ​= ________
    ​ (reducing the common
​ 40(xa)(x7)  x2 – y2 = (x – y) (x + y)
8 x5 factors)
4(x + y) (x
x2 + 9x + 20 ______
_________ + 4)(x + 5) _________4(x + y)

___
1
= ​   5 ​  ​ x2 + 8x + 16 
 ​÷ ​  2 2  
x –y
​= ​  __________  ​÷ ​ (x – y)(x + 
(x + 4)(x + 4) 
  ​
y)  
8x
x + 5 ____
_____ 4
 ​÷ ​ x – y  
= ​ x + 4  ​
3b2(a – b)
_________
7 ​  2   ​  x + 5 ____
_____ x–y
4a (a + b)2 = ​ x + 4  ​× ​  4 ​ 
 
Factorise and reduce each fraction where
possible to make the problem easier. Then (x + 5) (x – y)
____________
= ​    
 ​ 
follow the order of operations. 4 (x + 4)
_______
3a2 + 6a _______ ab3 2a3b
_____ 7a2 + 8ab + b2 + 4a + b
___________________
​   ​ 
× ​     ​ 
÷ ​   
 ​  10 ​      
 ​
4a + 2a2 (a + b)2 a – b (a – b)(a + b)
3a(a + 2) _______
________ 2a3b (reducing the common
​× ​  ab    ​  _____
3
​     ÷ ​   ​
   7a + b ___________
______ 4a + b
2a(2 + a) (a + b)2 a – b factors of the first term) ​   ​ 
+ ​     ​
  
a – b (a – b)(a + b)
__
3 _______ ab 3
2a b
_____ 3
(7a + b)(a + b) ___________
= ​ 2 ​ × ​     ​ 
÷ ​    ​ (multiply first and second _____________ 4a + b
(a + b)2 a – b = ​    
   ​+ ​     ​
  
terms as they cannot be (a – b)(a + b) (a – b)(a + b)
reduced further) 7a2 + 7ab + ab + b2 ___________
________________ 4a + b
= ​    
   ​+ ​     ​
  
________
3ab3 2a3b
_____ (a – b)(a + b) (a – b)(a + b)
= ​   2 ​ 
÷ ​   
 ​ 
2(a + b) a – b 7a2 + 7ab + ab + b2 + 4a + b
_______________________
= ​       ​
  
b2 (a – b)(a + b)
________
3ab3 a–b
_____
= ​ 2(a + b) 2 ​  2a b
​ (reducing the common factors)
× ​  3    7a2 + 8ab + b2 + 4a + b
___________________
= ​       ​
a2 (a – b)(a + b)
3b2(a – b)
_________
= ​  2  
 ​  7x + 41
_______
4a (a + b)2 11 ​   ​  
10
8 (  a–b 2
_____
)
–​​ a​  + b ​  ​​ ​
______
3x + 7 ______
​  2 ​ 
5(3x + 7) ________
4x – 3 ________
 – ​  5 ​ 
 = ​  10 ​   
2(4x – 3)
– ​  10 ​   
For any value of a and b, it is to be noted that 5 (3x + 7) – 2 (4x – 3)
__________________
a – b = –(b – a) (this can be easily verified by = ​  10 ​     
using numerical values for a and b) _______________
15x + 35 – 8x + 6
Since x2 = (–x)2 = ​  10 ​    
(a – b)2 = [–(b – a)]2 _______
7x + 41
= ​  10 ​  
∴(a – b)2 = (b – a)2

S24 Success In Mathematics


© SAP Group Pte Ltd

Solution.indd 24 9/7/2011 2:43:49 PM


(x + 1)(8x – 7)
_____________ 3
__
12 – ​      ​ 15 x = –3 or ​   ​ 
(x + 3)(2x + 1) 4
______
4x + 5 _____ 7
__________
3x + 7 _____
4x ____________
3x + 7 _____
4x ​  2 ​   = ​ x + 1 ​ (cross multiplying)
​  2 – ​    ​ = ​ 
  ​     ​– ​    ​ 
  
2x + 7x + 3 x + 3 (x + 3)(2x +1) x + 3
(4x + 5)(x + 1) = 14
____________
3x + 7 4x2 + 4x + 5x + 5 = 14
= ​    
   ​–
(x + 3)(2x + 1)
4x2 + 9x – 9 = 0 (equation can be factorised)
4x(2x + 1)
____________ (x + 3)(4x – 3) = 0
​        ​
(x + 3)(2x + 1) __
3
x = –3 or ​ 4  ​
3x + 7 – 4x (2x + 1)
________________
= ​    
    ​
(x + 3)(2x + 1) 16 x = –9
______________
3x + 7 – 8x – 4x
2 __________
2 _________
4
= ​    
   ​ ​  2    ​ – ​     ​ =0
(x + 3)(2x + 1) x + 7x + 12 x2 + 2x – 3
(both denominators can be factorised)
–8x2 – x + 7
____________
= ​    
   ​ ___________
2 ___________
4
(x + 3)(2x + 1) ​      ​– ​      ​= 0
(x + 3)(x + 4) (x + 3)(x – 1)
–(8x2 + x – 7)
____________ (the fractions can be combined with a common
= ​    
   ​
(x +3)(2x + 1) denominator)
–(x + 1)(8x – 7)
_____________ 2(x – 1) – 4(x + 4)
_________________
= ​    
   ​ ​    
   ​= 0
(x + 3)(2x + 1) (x + 3)(x + 4)(x – 1)
5x – 4ax – 4ay 2(x – 1) – 4(x + 4) = 0
_____________
13 ​    
   ​ (when an algebraic fraction is equal to zero,
(x – y)(x + y)
just take the numerator to be zero and disregard
______
5x ____
4a ___________
5x ____
4a the denominator)
​  2  2 ​ 
+ ​ y –  x ​ = ​     ​  + ​    ​ 
x –y (x – y)(x + y) y – x 2x – 2 – 4x – 16 = 0
___________
5x _______
4a –2x – 18 = 0
= ​     ​  + ​     ​ 
(x – y)(x + y) –(x – y) 2x + 18 = 0
___________
5x ____
4a x=–9
= ​     ​  – ​    ​ 
(x – y)(x + y) x – y 7
__
17 c = ​    ​
___________
5x 4a(x +
___________y) 3
= ​     ​  – ​    ​  _____
3 ______
5
(x – y)(x + y) (x – y)(x + y) ​ c + 4 ​ + ​  2    ​  = 0 (factorising the denominator)
c – 16
5x – 4a(x + y)
____________ _____
3 ___________
5
= ​    
   ​ ​ c + 4 ​ + ​      ​= 0
(x – y)(x + y) (c + 4)(c – 4)
5x – 4ax – 4ay 3(c – 4)
___________ ___________
5
____________ ​       ​+ ​      ​= 0
= ​    
   ​ (c + 4)(c – 4) (c + 4)(c – 4)
(x – y)(x + y)
3(c –
___________4) + 5
​       ​= 0
10(a – b)
________ (c + 4)(c – 4)
14 ​  x    ​ 
3(c – 4) + 5 = 0 (when a fractional value is 0, the
__
1 denominator could be dropped and numerator
Area = ​ 2 ​  × base × height
equated to 0)
__
1
35x(b – a)2 = ​ 2 ​  × base × 7x2(a – b) 3c – 12 + 5 = 0
70x(b – a)2 = base × 7x2(a – b) 3c – 7 = 0
__
7
70x(b – a)2 c = ​ 3 ​ 
_________
base = ​  2  ​ 
7x (a – b) S
____
18 ​     ​ – r
10 (a – b) 2�r
70x(a – b) 2

= ​ ________
7x2(a – b)
  ​ Note: (b – a)2 = (a – b)2 S = 2�rh + 2�r2
1x = 2�r(h + r)
10(a – b) S
____
________ ​ 2�r   ​ = h + r
= ​  x    ​ 
S
____
h = ​ 2�r   ​ – r

S25 Success In Mathematics


© SAP Group Pte Ltd

Solution.indd 25 9/7/2011 2:43:50 PM


_______
_____
4+2
19 ±​√v2 – 2as ​  gradient = ​ 4 + 4 ​ 
v2 = u2 + 2as
u2 = v2 _______
– 2as __
6
= ​ 8  ​
u = ±​√v2 – 2as ​ (Since there is no reason in this __
3
question to say that u must be positive, the = ​ 4  ​
negative value of u cannot be rejected) (Note there are no units for gradient.)
___ 3
__
(iii) y = ​   ​  x + 1

3V
3 ___
20 ​  ​   ​ ​   4
4�
Let the equation of the line AB be y = mx + c.
__
4
V = ​ 3  ​ �r3 where m = gradient of the line and c =
y-intercept of the line (where the line cuts
3V = 4�r3
the y-axis)
___
3V Using the gradient found in (ii) substitute
r3 = ​ 4� ​ 
___ __
3
for m, y = ​ 4 ​ x + c

3 ___3V
r = ​  ​ 4� ​ ​  
To find the value of c, substitute the co-
ordinates of any known pair of co-ordinates
(usually taking the most convenient for
11 CO-ORDINATE GEOMETRY
calculations).
Substituting the co-ordinates of point 2,
1 (i) 10 units
__
3
Using the distance formula, d 4 = ​ 4 ​  (4) + c
_________________
= ​√ (x2 – x1)2 + ( y  
2
– y1)2 ​ 4 = 3 + c
where, x1 = –4, y1 = –2, x2 = 4 and y2 = 4. c = 1
(Either of the points can be taken as point 1 Therefore the equation of the line is
__
3
and the other as point 2 for this formula, but y = ​ 4  ​x + 1.
as a matter of norm the point on the left, or This means that the line would have a slope
the one with the smaller x value is taken as __
3
point _______________
1.) of ​ 4 ​  and would cut the y-axis at 1.
d=√ ​ (4 + 4)2 + (4   + 2)2 ​
2 –4
(Note that the sign changes when subtracting Using the distance formula, d
negative co-ordinates.) _________________
_______ = ​√(x2 – x1)2 + ( y   – y1)2 ​
2
d=√ ​ (8 + 62) ​ 
2
________ where, x1 = –7, y1 = 5, x2 = 5, y2 = k and the
= ​√(64 + 36) ​ 
____ distance is 15 units,
_________________
= ​√100 ​ 
d=√ ​ (x2 – x1)2 + (y2  
– y1)2 ​
= 10 _______________
Therefore, the distance between the points 15 = √​ (5 + 7)2 + (k  
– 5)2 ​
is 10 units. (The units refer to the graph unit 225 = (5 + 7)2 + (k – 5)2 (squaring both sides to
which is used to draw the x and y axes.) remove the root sign)
= 122 + (k – 5)2
3
__
(ii) ​   ​  = 144 + (k – 5)2
4
y2 – y1
______ (k – 5)2 = 225 – 144
Using the gradient formula = ​ x – x    ​ = 81
2 1
where, x1 = –4, y1 = –2, x2 = 4 and y2 = 4. k – 5 = ±9
(Here too either of the points can be taken k = –4 or 14
as point 1 and the other as point 2 for this Since the point must be below x-axis, k = –4.
formula, but as a matter norm the point on 3 4.3
the left, or with the smaller x value is taken If A, B and C are on the same line, gradient of
as point 1.) AC = gradient of AB

S26 Success In Mathematics


© SAP Group Pte Ltd

Solution.indd 26 9/7/2011 2:43:50 PM


y2 – y1
______ vertical move from A to B (or the rise) is the
Using the gradient formula = ​ x – x  ​ 
and taking
2 1 same as the vertical move from D to C.
A as point 1 and C as point 2. run from A to B = run from D to C
That makes x1 = –6, y1 = 7, x2 = 4 and y2 = –2 8 – (–5) = 17 – p
p=4
______
–2 – 7
Gradient of AC = ​ 4 + 6 ​  rise from A to B = rise from D to C
___
9 9 – 4 = –2 – q
= – ​ 10  ​  q = –7
y2 – y1
______ Co-ordinates of point D: (4, –7).
Using the gradient formula = ​ x – x  ​ 
and taking
2 1
3
__
A as point 1 and B as point 2. 6 (i) y = – ​   ​  x + 7
2

n–7
______ y
Gradient of AB = ​ –3 + 6  ​ 
n–7
_____ •
= ​  3 ​ 
 
A(–2,10)
___9
Since gradient of AC = gradient of AB = – ​ 10  ​,  P•
n–7
_____ ___
9
​  3 ​  = – ​ 10  ​ 
10(n – 7) = –27 x
O Q
10n – 70 = –27
10n = 43 • B(8, –5)
___
43
n = ​ 10 ​ or 4.3 First find equation of line AB.
_______
–5 – 10 __
3
7
__ Gradient of AB is ​  8 + 2 ​ 
 
= –​ 2 ​ 
4 – ​   ​ x + 7
3
__
3
x co-ordinate of point A is 0 since it lies on the Equation of line: y = –​ 2 ​ x + c
y-axis.
For y-intercept of AB, substitute co-ordinates
y co-ordinate of point B and C are 0 since they
of point A.
lie on the x-axis.
To find x co-ordinate of B, substitute y = 0 in
3y – 7x = 21.
(  )
__
3
10 = ​ –​ 2 ​   ​(–2) + c

3(0) – 7x = 21 c=7
__
3
– 7x = 21 Equation of line: y = – ​ 2 ​ x + 7
x = –3
49
___
If ABC is an isosceles triangle, BO = OC since (ii) ​   ​ square units
3
OA is the line of symmetry for the triangle.
Co-ordinates of P(0, 7)
Co-ordinates of C: (3, 0)
To find co-ordinates of Q, substitute y = 0
Co-ordinates of A: (0, 7)
__
3
_____
0 – 7 into y = – ​ 2 ​ x + 7.
Gradient of AC = ​ 3 – 0 ​ 
__
3
__
7 0 = – ​ 2  ​x + 7
= – ​ 3 ​ 
___
14
y-intercept of AC is 7. x = ​  3 ​ 
__
7
Therefore, equation of line AC: y = – ​ 3 ​ x + 7
(  )
Co-ordinates of Q: ​ ​  3 ​ , 0  ​
___
14

(  )
5 (4, –7) __
1 ___ 14
Let the co-ordinates of point D be (p, q). Area of ∆OPQ = ​ 2 ​  ​ ​  3 ​   ​(7)
If ABCD is a parallelogram, AB is parallel to DC. ___
49
The horizontal move from A to B (run from A = ​  3 ​ square units
to B) is the same as the horizontal move from 7 (8, –12.32)
D to C (run from D to C). Similarly, for the

S27 Success In Mathematics


© SAP Group Pte Ltd

Solution.indd 27 9/7/2011 2:43:51 PM


To find c, substitute the x and y values of the
y co-ordinates (5, –7).
(  ) __
4
–7 = ​ ​ 5  ​  ​(5) + c
A(–2, 5) • • C(18, 5) c = –11
X
__
4
x y = ​ 5 ​ x – 11 (multiply throughout by 5 to remove
the fraction)
5y = 4x – 55
B• 5y – 4x = –55

Points A and C lie on the same horizontal line 51


___
9 (i) y = –3x + ​  5 ​ 
as can be seen from the sketch above. Point B
must be on a line which is the line of symmetry x-intercept is when y = 0,
of the ∆ABC which divides AC into two. The 0 = 5x – 17
line of symmetry of the ∆ABC cuts AC at X. ___
17
x = ​  5 ​ 
Let point X be located on AC such that AX =
XC   with a y co-ordinate k.
___
17
(  )
Co-ordinates of x-intercept: ​ ​  5 ​ , 0  ​
18 – k = k – (–2) (since AX = XC) Line with gradient –3 is y = –3x + c
k=8
Co-ordinates of X: (8, 5). ( 
___
17
)
To find c, substitute ​ ​  5 ​ , 0  ​ for x and y.
Let co-ordinates of B be (8, m)
Length of AC = 18 – (–2) = 20 units Since the line passes through the point, the
Length of AB = 20 (since ∆ABC is equilateral) co-ordinates of the point must conform to
Using the distance formula, the equation of the line.

(  )
_________________
___
17
AB = √
​ (x2 – x1)2 + (y2  
– y1)2 ​ 0 = –3​ ​  5 ​   ​+ c
where, x1 = –2, y1 = 5, x2 = 8 and y2 = m. ___
51
_______________ c = ​  5 ​ 
20 = √
​ (8 + 2)2 + (m  – 5)2 ​
___
51
400 = 10 + (m – 5)
2 2 Equation of line: y = –3x + ​  5 ​ 
400 = 100 + (m – 5)2
17
___
(m – 5)2 = 300____ (ii) y = –3x – ​   ​ 
2
m–5=±√ ​ 300 ​
  17
___
____ ____ y-intercept is when x = 0, y = – ​   ​ 
2
m = 5 + ​√300 ​ or 5 – ​√300 ​ 
(  )
____ ___
17
since point B is below the x axis, m = 5 – ​√ 300 ​.  Co-ordinates of y-intercept: ​ 0, – ​  2 ​   ​
m = –12.32 (2 decimal places)
___
17
Co-ordinates of B: (8, –12.32) \ c is just the co-ordinate – ​  2 ​ which need
8 5y – 4x = –55 not be found by substitution.
First write equation of the line in y = mx + c Equation of line with gradient –3 is
form. ___
17
5y – 4x = 9 y = –3x – ​  2 ​ 
5y = 4x + 9
10 5.53 units
__
4 __
9
y = ​ 5 ​ x + ​ 5 ​ 
__
4
The required line must have a gradient of ​ 5  ​ to O B(9,0)

be parallel to the given line. It must pass through
x
the point (5, –7).
__
4
y = ​ 5 ​ x + c
A(0,–7) •

S28 Success In Mathematics


© SAP Group Pte Ltd

Solution.indd 28 9/7/2011 2:43:51 PM


Length of OB = 9 units X
Length of OA = 7 units
∆AOB is a right-angled triangle.
12 cm
__
1
Area of ∆AOB = ​ 2 ​ (9)(7)
___
63
= ​  2 ​ square units
_______________ x cm
Length of AB = √
​ (0 + 7) + (9  
– 0)  ​
2 2
________
A B C
=√
​ (49 + 81) ​ 
____ Since an isosceles triangle is symmetrical
=
​√130 ​ 
about a perpendicular line drawn through the
Taking AB as the base of ∆AOB and x as the midpoint at the base, two right-angled triangles
height, will be formed with the equal sides as the
1 ____
__ hypotenuses.   Since B is the midpoint of the
Area of ∆AOB = ​ 2 ​  (​√130 ​) (x)
side AC, AB = 4 cm. Let the height be x cm.
___
63 __ 1 ____
​  2 ​ = ​ 2  ​(​√130 ​) (x) Using Pythagoras’ Theorem, 122 = x2 + 42
____
63 = (​√130 ​) (x) x2 = 122 – 42
= 144 – 16
_____
63
x = ​  ____
   ​  = 128
____
​ 130 ​ 

x=√ ​ 128 ​ 
= 5.53 units (3 sig. fig.)
= 11.31 (2 decimal places)
4 11.3 cm
12 PYTHAGORAS’ THEOREM

1 40 cm
Let length of AC = x cm. x 16 cm
Using Pythagoras’ Theorem, a2 = b2 + c2
(It has to be noted here that a is the hypotenuse
while b and c are the other two sides. The a, b x
and c could refer to any of the three sides but it Let the length of the sides of the square be x cm.
must be the square of the hypotenuse side equal Using Pythagoras’ Theorem, 162 = x2 + x2
to sum of the square of the other two sides.) 256 = 2x2
412 = x2 + 92 x2 = 128
____
1681 = x2 + 81 x=√ ​ 128 ​ 
x2 = 1681 – 81 = 11.3 (3 sig. fig.)
= 1600 5 7.86 m
x = 40
wall
2 18.0 cm
Let PR = x cm Y
Using Pythagoras’ Theorem, a2 = b2 + c2
x2 = 102 + 152 ladder
8m
= 100 + 225
= 325
____
x=√ ​ 325 ​ 
floor X 1.5 m O
=18.0 (3 sig. fig.)
The angle at O between the floor and the wall is
3 11.31 cm 90º (as usually should be). The ladder touches
the point X on the floor and the point Y on the
wall as shown. Let OY = x m.

S29 Success In Mathematics


© SAP Group Pte Ltd

Solution.indd 29 9/7/2011 2:43:52 PM


OX = 1.5 m Equating for y2, –x2 – 14x + 351 = 225 – x2
Using Pythagoras’ Theorem, 82 = 1.52 + x2 –14x = –126
64 = 2.25 + x2 14x = 126
x2 = 64 – 2.25 x=9
= 61.75 CD = 9 cm
x = 7.86 (3 sig. fig.)
9 12.3 cm
6 8 Taking the base rectangle ABCD, ∆ACD is a
Using Pythagoras’ Theorem, right-angled triangle. Let length of AC be x cm.
(2x + 1)2 = (2x – 1)2 + x2 Using Pythagoras’ Theorem for ∆ACD,
(2x + 1)(2x + 1) = (2x – 1)(2x – 1) + x2 x2 = 102 + 62
4x2 + 4x + 1 = 4x2 – 4x + 1 + x2 = 100 + 36
4x2 + 4x + 1 = 5x2 – 4x + 1 = 136
x2 – 8x = 0 Taking the right-angled triangle, ∆AHC, let
Factorising x(x – 8) = 0 length of AH be y cm.
x = 0 or 8 (x = 0 is rejected as Using Pythagoras’ Theorem for ∆AHC,
it cannot be the y2 = 42 + x2
side of a triangle) = 16 + 136
x=8 = 152
____
BC = 8 cm y=√ ​ 152 ​ 
= 12.3 (3 sig. fig.)
7 13 cm
Note that the value of x need not be found as
Drop a perpendicular line from B to the side
the value would have to be squared again.
DC. The foot of the perpendicular meets DC at
P. A right-angled triangle PBC is formed with 10 52 cm
the sides as shown in the diagram. Diagonals of rhombuses bisect each other
B at right angles. Therefore, in this case, four
right-angled triangles would be formed each
with perpendicular sides 12 cm and 5 cm. The
hypotenuse would be the side of the rhombus.
12 cm Let the sides be x cm.
x cm
Using Pythagoras’ Theorem x2 = 122 + 52
= 144 + 25
= 169
P 5 cm C x = 13
Therefore, perimeter of rhombus = 4 × 13
Since ABPD is a rectangle, PC = 23 – 18 = 5 cm. = 52
The length PB = AD = 12 cm. Let BC be x cm.
Pythagoras’ Theorem, x2 = 52 + 122
= 25 + 144 13 TRIANGLES,
= 169____ QUADRILATERALS AND
x=√ ​ 169 ​  POLYGONS
= 13
BC = 13 cm 1 (346 – 12x)º
8 9 cm Total of Ðs ABC, BAD and ADC
Let CD = x cm and let AD = y cm = (2x + 24)º + (3x – 18)º + (7x + 8)º
Using Pythagoras’ Theorem for ∆ABD, = (2x + 24 + 3x – 18 + 7x + 8)º
202 = (7 + x)2 + y2 = (12x + 14)º
400 = 49 + 14x + x2 + y2 Total of the angles of a quadrilateral = 360º
y2 = –x2 – 14x + 351 ∠BCD = 360º – (12x + 14)º
Using Pythagoras’ Theorem for ∆ACD, = (346 – 12x)º
152 = x2 + y2
y2 = 225 – x2

S30 Success In Mathematics


© SAP Group Pte Ltd

Solution.indd 30 9/7/2011 2:43:52 PM


2 78º (n – 2)180 = 140n
Opposite angles of a parallelogram are equal. 180n – 360 = 140n
∠PQR = ∠PSR 40n = 360
(3x – 27)º = (2x + 16)º n = 9 (polygon is 9-sided)
3x – 27 = 2x + 16
x = 43 6 24º
∠QRS + ∠PQR = 180º (interior angles)
pentagon Y
∠QRS = 180º – ∠PQR hexagon
= 180º – (3x – 27)º
= (180 – 3x + 27)º A X
= (180 – 129 + 27)º
= 78º
B
3 75º
Total interior angles of a 7-sided polygon Since AX = XB, triangle XAB is an isosceles
= (n – 2)180º triangle.
(n – 2)180º
__________
= (7 – 2)180º Using interior angles formula ​  n ​  of a
 
= 900º regular polygon,
Total interior angles of the 7-sided polygon
(5 – 2)180º
__________
given ∠AXY = ​  5 ​   
(n = 5 for pentagons)
= 108º + 162º + 5xº + 7xº + 3(6xº) (there are 3
= 108º
angles which are 6xº)
(6 – 2)180º
__________
= 270º + 30xº ∠BXY = ​ 
Therefore 270º + 30xº = 900º 6 ​   
(n = 6 for hexagons)
= 120º
30xº = 630º
∠BXY + ∠AXY + ∠AXB = 360º (total angles at
x = 21
a point)
From the angles given, 5x = 105, 7x = 147 and
120º + 108º + ∠AXB = 360º
6x = 126.
∠AXB = 132º
The largest exterior angle will be supplementary
_________
180 – 132
to that of the smallest interior angle. ∠XAB = ​  2 ​   
Since the smallest interior angle is 105º, the
= 24º
largest exterior angle
= 180º – 105º 7 104 cm2
= 75º A regular 9-sided polygon consists of nine
isosceles triangles as shown. The equal sides
4 101º
of each triangle are the radii of the circle in
The figure is a quadrilateral.
which the polygon is inscribed. Finding the
Total of the angles of a quadrilateral = 360º
area of one triangle and multiplying would give
Total of Ðs ABC, BAD and DCB
the area of the polygon. One of the triangles is
= 56º + 21º + 34º
shown below.
= 101º
The reflex ÐADC = 360º – 101º (note that
it is the reflex ÐADC which is part of the 40º
quadrilateral).
= 259º 6 cm 6 cm
The obtuse ÐADC = 360º – 259º
= 101º
5 9 ___
12
Radius of circle = ​  2 ​ = 6 cm
Each interior angle of a regular polygon
____
360º
(n – 2)180º
__________ Apex angle of the triangle = ​  9 ​   = 40º
= ​  n ​   
__
1
Area of triangle = ​ 2  ​(6)(6) sin 40º
(n – 2)180º
__________
If  ​  n ​  = 140º
 
= 18 sin 40º

S31 Success In Mathematics


© SAP Group Pte Ltd

Solution.indd 31 9/7/2011 2:43:52 PM


Area of polygon = 9 × 18sin 40º other five sides formed as a reflection of
= 162sin 40º ABCDEF.
= 104 cm2 (3 sig. fig.) Triangle BAC forms an isosceles triangle
(It is easier to use this formula than to find the with ÐABC being the interior angle of a
height and the base using trigonometry and/or regular decagon.
Pythagoras’ Theorem for finding the area of the (n – 2)180º
__________
triangle.) ​  n ​   = 144º where n = 10
(180º – 144º)
___________
8 5 ÐBAC = ​  2 ​     = 18º
(ii) 54º
polygon 1
ABCDE is a pentagon with three of the
polygon 2 angles, Ðs ABC, BCD and CDE, being 144º
xº each (interior angles of a regular decagon).
xº By symmetry, the other two Ðs BAE and

DEA are equal.
Total angles of a pentagon
= (n – 2)180º (where n = 5)
The two polygons are identical and when = 540º
placed side by side as shown, three angles will Total of ∠ABC, ∠BCD and ∠CDE = 3 × 144º
be formed. Angle x is the interior angle of the = 432º
regular polygon made by the tile. Angle y is (540º – 432º)
___________
the angle made by the regular decagon space ∠BAE = ​  2 ​    
enclosed by the tiles. = 54º
Therefore, 2x + y = 360º
Angle y can be found using the formula (iii) 108º
By symmetry, BC and AD are parallel. This
(n – 2)180º
__________
​  n ​   where n = 10. makes Ðs ABC and BAD interior angles of
a trapezium.
(10 – 2)180º
___________
y = ​  Since ÐABC = 144º, ÐBAD = 180º – 144º
10 ​   
= 36º
= 144º ∠DAE = ∠BAE – ∠BAD
2x + 144º = 360º = 54º – 36º
x = 108 = 18º
To find the number of sides of the tiles use Using the sum of angles of triangle ADE,
(n – 2)180º
__________ ∠ADE = 180º – (∠DAE + ∠DEA)
​  n ​   = 108º as x is the interior angle of a
∠ADE = 180º – (18º + 54º)
regular polygon. = 108º
(n – 2)180º
__________ 10 11 sides
​  n ​   = 108º
Ratio of exterior angle : interior angle of a
180n – 360 = 108n
regular polygon in general is
72n = 360
n=5 ____ (n – 2)180º
360º __________
​  n ​   : ​  n ​   
Each tile is a 5-sided polygon (pentagon).
360 : (n – 2)180
9 (i) 18º 2:n–2
Partial sketch of the decagon showing five Since 2 : n – 2 is 2 : 9, n – 2 = 9
of the ten sides. n = 11.

C D
B E 14 BASIC TRIGONOMETRY

F 1 (i) 33.1º, 0.58 rad


A
In this situation, to decide which trigonometric
Since it is a regular pentagon it must be ratio to use, consider what sides and angles
symmetrical about the line AF with the are involved (ie. known or need to find out).

S32 Success In Mathematics


© SAP Group Pte Ltd

Solution.indd 32 9/7/2011 2:43:53 PM


First ÐC needs to be found with AB and AC
known. 25º
With respect to ÐC, AB is the opposite
x cm
side and AC is the hypotenuse. Of the three
trigonometric ratios, sin ratio applies to
opposite and hypotenuse.
6 cm
opposite
__________
sin C = ​     ​  For Ð 25º, the sides involved are the opposite
hypotenuse
___6 which is 6 cm and the adjacent which is
= ​ 11  ​  unknown.
Using a calculator, opposite
_______
Therefore using tan 25º = ​   ​ 
adjacent
(  ) ___
6
ÐC = sin–1 ​ ​ 11  ​   ​

__
6
= ​ x ​
= 33.1º (angles in degrees are usually ______6
given in 1 decimal place) x = ​ tan 25º
   ​ 
To find the angle in radians, = 12.9 cm (3 sig. fig.)
(  ) ____
π
33.1º = 33.1​ ​ 180   ​  ​radians
3 21.8º
= 0.58 radians (2 decimal places) P
(ii) 56.9º, 0.99 rad
To find ÐBAC, we do not have to use
trigonometry as it could be easily found by 2x cm
the sum of angles in a triangle.
90º + 33.1º + ∠BAC = 180º q
∠BAC = 56.9º Q 5x cm R
To find the angle in radians,
The smallest angle must be ÐPRQ as it faces
(  )
____
π
56.9º = 56.9​ ​ 180   ​  ​radians the shortest side. Referring to ÐPRQ, the sides
= 0.99 radians (2 decimal places) involved in this question are the opposite and
the adjacent. So use the tan ratio.
(iii) 9.22 cm opposite
_______
To find the side BC, we do not have to use tan θ = ​   ​ 
adjacent
trigonometry as it could be easily found by ___
2x
using Pythagoras’ Theorem. Let the length = ​ 5x ​
of BC be x cm. __
2
= ​ 5  ​(simplify by cancelling x)
112 = 62 + x2
x = 9.22 cm (3 sig. fig.) (  ) __
2
θ = tan–1​ ​ 5 ​   ​
2 12.9 cm = 21.8º
A
4 78.6º
First find the height AD. Let AD be h cm. For
x cm
Ð35º the sides involved are the opposite which
B C is the height and the adjacent which is 7 cm.
12 cm
opposite
_______
Draw a line to represent the height of the Using tan 35º = ​   ​ 
adjacent
triangle and let its length be x cm. The height
line separates the triangle ABC into 2 right- __
h
= ​ 7  ​
angled triangles by bisecting the side BC and
h = 7 tan 35º
ÐBAC. Each right-angled triangle will have a
= 4.901 cm
base of 6 cm and an angle of 25º as shown.
Let ÐABD be θ. For θ, the sides involved are
the opposite which has been found and the
hypotenuse which is 5 cm.

S33 Success In Mathematics


© SAP Group Pte Ltd

Solution.indd 33 9/7/2011 5:10:39 PM


opposite
__________ The perpendicular line from C to AB meets AB
Using sin θ = ​    ​  at X and a right-angled triangle XBC could be
hypotenuse
seen. Let CX be x cm
_____
4.901
= ​  5 ​    For triangle ABC, ∠CBA = 180º – 90º – θ


= 180º – 90º – 34.8º

_____
4.901
)
θ = sin–1​ ​  5 ​  
 ​ = 55.2º
= 78.6º ∠CBA of triangle ABC is ∠CBX of triangle
XBC, as shown.
5 16.1 cm2 C
To find the area, height AB and base BC of the
triangle need to be known.
Let AB = x cm and BC = y cm.
From the 20º angle, x is the opposite, y is the
adjacent and AC is the hypotenuse. x cm 4 cm
opposite
__________
Using sin 20º = ​    ​ 
hypotenuse
___x 55.2º
= ​ 10  ​ 
X B
x = 10 sin 20º Taking CB as the hypotenuse and CX as the
= 3.42 cm opposite, use sin ratio for ∠CBX.
adjacent
__________ opposite
Using cos 20º = ​     ​  __________
hypotenuse sin 55.2º = ​    ​ 
hypotenuse
y
___ x
__
= ​ 10  ​  = ​ 4  ​
y = 10 cos 20º x = 4 sin 55.2º
= 9.40 cm = 3.28 cm (3 sig. fig.)
Note that BC could also be found by Pythagoras’ The shortest distance of C from the side AB is
Theorem. In fact to find the area only the side 3.28 cm.
BC needs to be known and the formula for area
7 36.9º
of triangle could be found in the later topic on
A 8 cm B
Trigonometry.
__
1
Area = ​ 2 ​ (3.42)(9.40)
3 cm
= 16.1 cm2 (3 sig. fig.)
6 34.8º, 3.28 cm D 8 cm E 4 cm C
To find angle θ, use the sin ratio as the opposite Isolate a right-angled triangle by drawing a
and the hypotenuse are known. vertical line through B which is also parallel
__
4 to AD. The vertical line will intersect DC at E
sin θ = ​ 7 ​  where there will be a right angle.
__
4
θ = sin–1 (​ 7 ​ ) B
= 34.8º
To find the shortest distance of C from the side
AB, draw a perpendicular line from C to AB and
3 cm
find its length.

A
X q
q E 4 cm C
7 cm
BE = AD = 3 cm
EC = DC – DE
= 12 – 8
C 4 cm B = 4 cm
Let the unknown angle be θ.

S34 Success In Mathematics


© SAP Group Pte Ltd

Solution.indd 34 9/7/2011 2:43:53 PM


From the angle at C, the opposite and the 90º – 27.5° = 62.5º. Half the longer diagonal of
adjacent sides are known. So use the tan ratio. the rhombus forms the side of the right-angled
opposite
_______ triangle which must be opposite Ð 62.5º (longer
tan θ = ​   ​ 
adjacent side faces the larger angle). After finding the
__
3 perpendicular side of the triangle, doubling it
= ​ 4  ​
will give the length of the other diagonal.
θ = tan–1 0.75
= 36.9º
62.5º
8 97.4 cm2 x cm

27.5º
y cm 15 cm 5 cm

30º Let the unknown side be x cm


x cm For Ð 27.5º, opposite = x cm, adjacent = 5 cm
Let the length be x cm and the breadth be y cm. __x
tan 27.5º = ​ 5  ​
To find x, use the cos ratio as the known side
is the hypotenuse and the unknown side to be x = 5 tan 27.5º
found is the adjacent. Length of diagonal = 2(5 tan 27.5º)
x
___ = 5.21 cm (3 sig. fig.)
cos 30º = ​ 15  ​ 
x = 15 cos 30º 10 40.9º
= 12.99 cm (2 decimal places) First find the height of the building using the
To find the area, the breadth of the rectangle has 200 m and 60º. Using half the building height
to be found. However, it could also be found and 200 m find the angle of the triangle.
by other methods subsequently covered later building
in trigonometry. Finding the breadth could also
be done using the Pythagoras’ Theorem. To
find y use the sin ratio taking the breadth as the
opposite and the 15 cm side as the hypotenuse.
Note that tan ratio could also be used by taking 60º
the length already found as the adjacent. 200 m
y
___
sin 30º = ​ 15  ​  Using the 60º as the reference, the sides
y = 15sin 30º involved here are the opposite (need to find)
= 7.5 cm and adjacent (known). So use tan ratio taking
Area of rectangle = (7.5 × 12.99) cm2 the height of building as x m.
= 97.4 cm2 (3 sig. fig.) opposite
_______
tan 60º = ​   ​ 
adjacent
9 5.21 cm x
____
Diagonals of a rhombus bisect each other at    ​ 
= ​ 200
right angles which will result in four congruent x = 200 tan 60º
right-angled triangles. __________
200 tan 60º
Half the building height = ​  2 ​   
= 100 tan 60º
Form another triangle,

half
The triangles would have the sides of the building
rhombus as the hypotenuses. The diagonals also height
bisect the angles at the vertices. The 55° angle q
gives an Ð 27.5° in the right-angled triangle. 200 m
The other angle of the right-angled triangle is

S35 Success In Mathematics


© SAP Group Pte Ltd

Solution.indd 35 9/7/2011 5:14:10 PM


Using θ as the reference, the sides involved 2 (–5, 4), x = –5, x-intercepts are (–3.85, 0) and
here are the opposite (half building height
(–6.15, 0), y-intercept is –71
= 100 tan 60º) and adjacent (200 m).
The quadratic expression is in the form a(x – h)2
__________
100 tan 60º + k where a = –3, h = –5 and k = 4.
tan θ = ​  200 ​   
For a < 0, the graph is downward curving.
= 0.8660
For h = –5, the curve has a line of symmetry at
θ = tan–1 θ x = –5.
= 40.9º For k = 4, the curve has a maximum point at
y = 4 (since a < 0).
The maximum point is at (–5, 4).
15 FUNCTIONS AND GRAPHS The x-intercepts are the roots found as

1
5
__
4 ( 5 ___
__ 81
)
x = –​    ​, ​ –​   ​ , –​   ​   ​, x-intercepts are (–3.5, 0)
4 8
follows:
–3(x + 5)2 + 4 = 0
and (1, 0), y-intercept is (0, –7) __
4
(x + 5)2 = ​ 3 ​ 
The expression 2x2 + 5x – 7 can be factorised __
__
as the b2 – 4ac value is 81, which is a perfect
square.

__
√ 4
x + 5 = ±​ ​ 3 ​ ​  
__
Factorising 2x2 + 5x – 7 = (x – 1)(2x + 7) __ __
Sketch the graph of y = (x – 1)(2x + 7)
√ 4
√ 4
x = ​ ​ 3  ​ ​ – 5 or – ​ ​ 3 ​ ​  – 5
__
7 = –3.85 or –6.15 (approx)
(the roots are x = 1 and x = –​ 2 ​ )
The y-intercept is the y value when the value of
Draw the x and y axes and mark the roots on the x is 0 which is –71.
x-axis (roots are where the function value is 0). y
The a value in this quadratic expression is 2 (>
0), the graph should curve upwards. The line of
symmetry should be mid-way between the two
roots.

(–5, 4)

__
1 __ 7 __
5
x co-ordinate of the mid-point = ​ 2 ​ ​  –​ 2  ​+ 1  ​= –​ 4 ​ . ) (–6.15, 0) (–3.85, 0)
The minimum point occurs at the line of x
symmetry. The x co-ordinate of the minimum
__
5
point is –​ 4 ​ . The y co-ordinate of the minimum
__
5
point is found by substituting x = –​ 4 ​  into the
function.
(  __
5
)(  (  ) )
__
5 ___
81
y = ​ –​ 4 ​  – 1  ​​ 2​ –​ 4 ​    ​+ 7  ​= –​  8 ​ 
x = –5 (0, –71)


__
5 ___
Therefore, the minimum point is ​ –​ 4 ​ , –​  8 ​   ​.
81
(  ) 3 (i) y = x and y = –x lines
The x-intercepts are the roots and the y-intercept
y
is the y value when the value of x is 0 which is
–7.
The graph can be sketched as follows.
5
x = –__
​ 4 ​ y
x
y = (x – 1)(2x + 7)

 
​( –_​ 72 ​,  0 )​ (1, 0)
x
The lines of symmetry are shown by the
(0, –7)
 
​( –__​ 54 ​, –__
​ 818  ​ )​
dotted lines which are y = x and y = –x
lines.

S36 Success In Mathematics


© SAP Group Pte Ltd

Solution.indd 36 9/7/2011 2:43:54 PM


(ii) –2b x –2 –1.5 –1 –0.5 0 0.5 1 1.5 2
Since y = 2b when x = a, from the graph by ​ 1x ​ –2.5 –2.17 –2 –2.5 undefined 2.5 2 2.17 2.5
y = x + __
symmetry, y = –2b when x = –a.
(iii) y = p Take values of x with intervals of 0.5 as there
Since y = q when x = p, from the graph by must be sufficient values to draw the curve.
symmetry, y = p when x = q. After plotting the curves, it could be seen that
when value of x is close to 0 the graph would
(iv) 1 and –1
give a very large positive or negative value.
__
1 __
1
Since y = ​ x ​, when x and y are equal, x = ​ x ​ When x = 0, the y value is undefined.
or x2 = 1. This happens when x = 1 or –1. (Refer to Graph 3, page S38)
__
1
__
2 For solving the equation 3x – 1 = ​ x ​, the equation
(v) drawing the line y = –​   ​ x can be written as,
3
__
3 __
3 __
1
2x + ​ x ​= 0 can be re-written as ​ x ​= –2x x + 3x – 1 = x + ​ x ​
__
3 __
1 __
2 __
1
​ x ​= –2x can be re-written as ​ x ​= (–​ 3 ​ )x 4x – 1 = x + ​ x ​
Therefore, drawing the graph of y = ​ –​ 3 ​   ​x (  )
__
2
So a graph of y = 4x – 1 should be drawn on the
__
1 __
1
on the graph y = ​ x ​will give the solution to graph of y = x + ​ x ​to solve the equation.
__
3
the equation 2x + ​ x ​= 0 if the line intersects 6 x = –0.75 (approx) and x = 2
with curve. However, it can be seen that the
x –3 –2 –1 0 1 2 3
line will not intersect with the curve.
y = x2 9 4 1 0 1 4 9
4 (i) A = 4, n = –2 y = 2x 0.125 0.25 0.5 1 2 4 8

When x = 2 , y = 1, so 1 = A(2)n Eq 1 (Refer to Graph 4, page S39)


__
1 __
1 Plotting the graph, the intersection points are at
When x = 4 , y = ​ 4 ​ , so ​ 4 ​  = A(4)n Eq 2 x = –0.75 (approx) and x = 2.
Dividing equation 1 by equation 2,
7 –2, 0.5 (approx) and 3, –1.35, –0.45 and
4 = (2)n ÷ (4)n (A cancels off)
3.35 (approx)
1 n
__
(  )
= ​​ ​ 2 ​   ​​ ​ For y = 2x3 – 3x2 – 11x + 6
22 = (2–1)n (by indices) x –3 –2 –1 0 1 2 3 4
= 2–n y –42 0 12 6 –6 –12 0 42
Therefore n = –2. Plotting the graph, the roots of the equation are
Using Eq 1 to substitute n, 1 = A(2)–2 found by locating the x values where the curve
__
A cuts the x axis.
= ​  2  ​ 
2 (Refer to Graph 5, page S40)
A=4 The roots are –2, 0.5 (approx) and 3. For finding
(ii) y = –2x – 2 the roots of the equation 2x3 – 3x2 – 11x + 6 =
y = 4x–2 10 draw a horizontal line y = 10 on the same
__
4 graph and locate the intersection points. The x
= ​  2  ​ 
x values of the intersection points are the roots.
__
2 __
2 The approximate roots are –1.35, –0.45 and
​  2  ​ + x + 1 = 0 can be re-written as ​  2  ​ = –x – 1
x x 3.35.
__
2
​  2  ​ = –x – 1 can be multiplied by 2 throughout to
x 8 26.7 m (3 sig. fig.)
__
4 Parabolic curve means a quadratic curve
give ​  2  ​ = –2x – 2
x y = A(x – h)2 + k.
Therefore, a straight line y = –2x – 2 has to be Since the highest point reached should be at the
__
2 line of symmetry, x = 50 is the line of symmetry.
drawn to solve the equation ​  2  ​ + x + 1 = 0.
x The two roots of the equation are at sea level
5 Maximum pt: (–1, –2) which is the y = 0 line. One root is at x = 150.
Minimum pt: (1, 2) There is one root which is at a point which is
y = 4x – 1 at a negative value obtained by extending the

S37 Success In Mathematics


© SAP Group Pte Ltd

Solution.indd 37 9/7/2011 2:43:55 PM


Graph 3
Topic 15, question 5.
y

2
​ 1x ​
y = x + __

x
–3 –2 –1 0 1 2 3

–1

–2

S38 Success In Mathematics


© SAP Group Pte Ltd

Solution.indd 38 9/7/2011 4:14:32 PM


Graph 4
Topic 15, question 6.

y = x2
6

y = 2x
x
–3 –2 –1 0 1 2 3

S39 Success In Mathematics


© SAP Group Pte Ltd

Solution.indd 39 9/7/2011 2:43:56 PM


Graph 5
Topic 15, question 7.
y

40
y = 2x3 – 3x2 – 11x + 6

20

y = 10
x
–3 –2 –1 0 1 2 3 4

–20

–40
are the roots of equation
2x3 – 3x2 – 11x + 6 = 10

S40 Success In Mathematics


© SAP Group Pte Ltd

Solution.indd 40 9/7/2011 2:43:56 PM


graph backwards but is not applicable to the x –3 –2 –1 0 1 2 3 4
physical problem here. y –27 –6 1 0 –3 –2 9 36
The quadratic curve applicable can be drawn (Refer to Graph 7, page S43)
with x and y axes as follows: From the graph, the roots of the function are
–1.3 (approx), 0 and 2.3 (approx).
y
For solving x3 – x2 – 4x + 13 = 0, it can be re-
(50, k) written as
x3 – x2 – 3x = x – 13
(0, 20) Therefore, the straight line y = x – 13 needs to
(150, 0) be drawn. From the graph, there is only one
x intersection point at x = –2.55 (approx).
x = 50

16 SIMULTANEOUS EQUATIONS
Taking h = 50 1 (i) x = 3, y = 5
When x = 0, y = 20. The process of solving these types of
20 = A(0 – 50)2 + k simultaneous equations by elimination is
20 = 2500A + k Eq 1 to make one of the variables ‘disappear’
When x = 150, y = 0. by adding or subtracting the two
0 = A(150 – 50)2 + k equations. This is possible in the case of
0 = 10000A + k Eq 2 (1i). Both equations contain the term 2x
Multiply Eq 1 by 4, and subtracting one from the other will
80 = 10000A + 4k Eq 3 eliminate the x variable.
Subtracting: Eq 3 – Eq 2, 2x + 3y = 21 Eq 1
80 – 0 = (10000A + 4k) – (10000A + k) 2x + 7y = 41 Eq 2
80 = 3k Eq 2 – Eq 1, (It could also be Eq 1 – Eq 2,
___
80 but it is easier to have positive numbers in
k = ​  3 ​ 
the calculations.)
k represents the highest point reached by the
___ (2x + 7y) – (2x + 3y) = 41 – 21
80
projectile above sea level which is ​  3 ​ = 26.7 m 4y = 20
(3 sig. fig.) y=5
To find the x value which was eliminated
9 x = 5.1 cm, h = 7.7 cm (approx) earlier, substitute y = 5 into equation 1 or 2,
Volume of component, V = hx2 choosing the one where the computations
200 = hx2 are minimal. In this case, computations for
____
200 both equations are about the same.
h = ​  2 ​ 
x
Using eq 1, 2x + 3(5) = 21
Taking 1 ≤ x ≤ 6, 2x + 15 = 21
x 1 2 3 4 5 6 2x = 6
h 200 50 22.2 12.5 8 5.6 x=3
Draw the graph. Solutions to the equations: x = 3, y = 5
(Refer to Graph 6, page S42) For confirmation, the values could be
__
h substituted in either of the equations and
Using ​ x ​= 1.5 verified.
h = 1.5x
Draw a straight line h = 1.5x on the same graph 19
___ 28
___
(ii) x = ​   ​ , y = ​   ​ 
13 13
and find the point of intersection.
For these equations, make the coefficients
The approximate x value at the intersection is
of one of the variables the same or
5.1 and the h value is 7.7.
the negative of the other. Then, make
10 –1.3 (approx), 0 and 2.3 (approx), that variable ‘disappear’ by adding or
x = –2.55 (approx) subtracting the two equations. In (1ii), it

S41 Success In Mathematics


© SAP Group Pte Ltd

Solution.indd 41 9/7/2011 2:43:56 PM


Graph 6
Topic 15, question 9.
h

200
h = ___
​ x2   ​
40

30

20

10

h = 1.5x

0 1 2 3 4 5 6 x

S42 Success In Mathematics


© SAP Group Pte Ltd

Solution.indd 42 9/7/2011 2:43:57 PM


Graph 7
Topic 15, question 10.
y

30
y = x3 – x2 – 3x

20

10

x
–3 –2 –1 0 1 2 3 4

–10
y = x – 13

–20

S43 Success In Mathematics


© SAP Group Pte Ltd

Solution.indd 43 9/7/2011 2:43:57 PM


is better to eliminate y as only the second For these equations, make the coefficients
equation needs to be multiplied while both to be integers as it would be easier to work
equations need to be multiplied to eliminate with integers. However, the equations could
x. still be evaluated with fractions.
3x + 4y = 13 Eq 1 __
1 __ 1 __
7
​ 4 ​ x – ​ 3 ​ y = –​ 8 ​   Eq 1
5x – 2y = 3 Eq 2
Multiply eq 2 by 2 throughout, __
3 __
1 ___
27
​ 8 ​ x + ​ 2  ​y = ​ 16 ​   Eq 2
10x – 4y = 6 Eq 3
Add Eq 1 and Eq 3 (note that when the Multiply eq 1 by 24 throughout,
terms containing the variables are both of 6x – 8y = –21 Eq 3
the same sign they have to be subtracted (24 is the LCM of 3, 4 and 8 which are
while they have to be added when they denominators in the equation)
have different signs.) Multiply eq 2 by 16 throughout,
(3x + 4y) + (10x – 4y) = 13 + 6 6x + 8y = 27 Eq 4
13x = 19 (16 is the LCM of 2, 8 and 16 which are
___
19 denominators in the equation)
x = ​ 13 ​  Now the equations can be solved as
To find the y value which was eliminated 6x – 8y = –21 Eq 3
___
19 6x + 8y = 27 Eq 4
earlier, substitute x = ​ 13 ​ into equation 1, 2 Add eq 3 and eq 4 to eliminate y,
or 3. (6x – 8y) + (6x + 8y) = –21 + 27
___
19
(  )
Using Eq 2, 5​ ​ 13 ​   ​– 2y = 3 12x = 6
__
1
___
95 x = ​ 2 ​ 
​ 13 ​ – 2y = 3 __
1
Substitute x = ​ 2 ​  into eq 4,
___
28
(  )
y = ​ 13 ​  __
1
6​ ​ 2 ​   ​+ 8y = 27
___
19 ___
28
Solution to the equations: x = ​ 13 ​ , y = ​ 13  ​ 3 + 8y = 27
(Solutions need not be neat integers and y=3
could be cumbersome fractions!) __
1
Solution to the equations: x = ​ 2 ​ , y = 3
(iii) x = –5, y = 0 38
___ 3
___
For these equations, make the terms of one (v) x = ​ 75  ​ , y = ​    ​ 
25
of the variables the same or the negative For this type of equations, the expressions
of the other. In (1iii), for eliminating x or have to be written in the form which
y, about same steps are required. Here we conforms to the simultaneous equations
would eliminate y. format. Then the usual steps are applied.
3x – 5y = –15 Eq 1 Eq 1: 15(2x + y) =17 becomes 30x + 15y
7x + 2y = –35 Eq 2 =  17
Multiply eq 1 by 2 throughout, x_____– 2y ___ 1
6x – 10y = –30  Eq 3 Eq 2: ​  4 ​   = ​ 15  ​ becomes 15x – 30y = 4
Multiply eq 2 by 5 throughout, (multiplying throughout by 60)
35x + 10y = –175  Eq 4 After that, it is the usual process of finding
Since the y terms to be eliminated have the unknowns.
different signs, add eq 3 and eq 4. 30x + 15y = 17 Eq 1
(6x – 10y) + (35x + 10y) = –30 – 175 15x – 30y = 4 Eq 2
41x = –205 Multiplying eq 1 by 2 throughout,
x = –5 60x + 30y = 34 Eq 3
Substitute x = –5 into eq 1, 3(–5) – 5y = –15 Add eq 2 and eq 3,
y=0 (15x – 30y) + (60x + 30y) = 4 + 34
Solution to the equations: x = –5, y = 0 75x = 38
__
1 ___
38
(iv) x = ​   ​ , y = 3 x = ​ 75 ​ 
2

S44 Success In Mathematics


© SAP Group Pte Ltd

Solution.indd 44 9/7/2011 2:43:58 PM


___
38 35(2y) – 10y – 24 = 0
To find y, substitute x = ​ 75  ​into equation 1,
70y – 10y – 24 = 0
___
38
(  )
30​ ​ 75 ​   ​+ 15y = 17 60y – 24 = 0
___
24
___
3 y = ​ 60 ​ 
y = ​ 25  ​ 
___
38 ___
3 __
2
Solution to the equations: x = ​ 75 ​ , y = ​ 25  ​  = ​ 5 ​ 


13
___ ___
11 ​
(vi) x = 1​ 24 ​,  y = ​ 12
__
2
(  )__
4
x = 2​ ​ 5 ​   ​= ​ 5 ​ 

For this type of equations, write the __


2 ___
21 __
4
3 y = –​    ​ x + ​  5 ​ , y = ​   ​  x + 1, x = 1.6 (approx),
3 3
expressions in the form which conforms to y = 3.1 (approx)
the simultaneous equations format. __
3 __
3
Eq 1: 6x + 27y – 34 = 0 becomes 6x + 27y 10x + 15y = 63 and ​ 4 ​ y – x = ​ 4 ​ 
= 34 15y = –10x + 63
Eq 2: 3y – 18x + 25 = 0 becomes –18x + 3y __
2 ___
21
= –25 y = –​ 3 ​ x + ​  5 ​ 
6x + 27y = 34 Eq 1 __
3 __
3
​ 4 ​ y – x = ​ 4 ​ 
–18x + 3y = –25 Eq 2
Multiplying eq 1 by 3 throughout 3y – 4x = 3
18x + 81y = 102 Eq 3 3y = 4x + 3
__
4
Add eq 2 and eq 3, y = ​ 3 ​ x + 1
(–18x + 3y) + (18x + 81y) = –25 + 102 These are two straight line equations which can
84y = 77 be plotted on the graph paper as shown.
___
77 __
2 ___
21
y = ​ 84 ​  For y = –​ 3 ​ x + ​  5 ​ , when x = 0, y = 4.2 and when
___
11 x = 3, y = 2.2
= ​ 12  ​
__
4
___
11 For y = ​ 3  ​x + 1, when x = 0, y = 1 and when
Substitute y = ​ 12  ​into eq 2,
x = 3, y = 5
___
11
(  )
–18x + 3​ ​ 12  ​  ​= –25 (Refer to Graph 8, page S46)
___
11 Note that for straight lines just two points need
–18x + ​ 4 ​ = –25 to be taken on the graph to draw the line.
___
13 The solution is the x and y co-ordinates of the
x = 1​ 24 ​ 
___
13 ___
11 point of intersection. The intersection point is
Solution to the equations: 1​ 24 ​ , ​ 12  ​ approximately (1.6, 3.1) as shown on the graph.
2 (i) x = 3, y = –2 4 (x – y) = 4, x = 5, y = 1
Substitution could be carried out for either Using the difference of squares identity,
x or y, but in this case y would be an easier x2 – y2 = (x + y)(x – y)
choice as it could be more easily obtained. 24 = 6(x – y)
Using y – 5x = –17, y = 5x – 17 x–y=4
Substitute y value into 3x – 7y = 23,
So we have two equations, x + y = 6 and x – y = 4.
3x – 7(5x – 17) = 23
Adding the two equations,
3x – 35x + 119 = 23
(x + y) + (x – y) = 6 + 4.
–32x = –96
x=3 2x = 10
Since y = 5x – 17 (substituting x = 3) x=5
= 15 – 17 Using x + y = 6 to substitute x = 5, y = 1.
= –2 5 x = 27, y = 31
__
4 __
2 For a right-angled isosceles triangle the two
(ii) x = ​ 5 ​ , y = ​ 5 ​  angles which are not right angles are both 45º.
35x – 10y – 24 = 0, x = 2y Therefore (3x – y – 5)º = (x + y – 13)º = 45º
Since x is given as 2y, Using (3x – y – 5)º = (x + y – 13)º,

S45 Success In Mathematics


© SAP Group Pte Ltd

Solution.indd 45 9/7/2011 2:43:58 PM


Graph 8
Topic 16, question 3.

6 ​ 43 ​x + 1
y = __

3.1 (1.6, 3.1)

2
​ 23 ​x + __
y = –__ ​ 215  ​

x
–6 –4 –2 0 1.6 2 4 6

–2

S46 Success In Mathematics


© SAP Group Pte Ltd

Solution.indd 46 9/7/2011 2:43:58 PM


3x – y – 5 = x + y – 13 (15x + 35y) – (15x + 9y) = 3075 – 1983
2x – 2y = –8 26y = 1092
x – y = –4 Eq 1 _____
1092
y = ​  26 ​ 
 
Using (x + y – 13)º = 45º
x + y = 58 Eq 2 = 42
Add Eq 1 and Eq 2, Substituting y = 42 into 3x + 7y = 615,
(x – y) + (x + y) = –4 + 58 3x + 7(42) = 615
2x = 54 3x + 294 = 615
x = 27 3x = 321
Using eq 2, 27 + y = 58 x = 107
y = 31 Prices of a table and a chair are $107 and $42
Note that when equations are formed they respectively.
must be independently obtained so that they
can be solved. For example, equation 2 above 19
___ 37
___
8 ​   ​ and ​   ​ 
8 8
was obtained by (x + y – 13)º = 45º. The other Let the numbers be x and y.
equation cannot be obtained by relying on the From the sum of the numbers,
fact that double that angle is 90º and therefore x + y = 7 Eq 1
2(x + y – 13)º = 90º. When simplified, both If x is multiplied by 3, it will be 2.5 more than
equations would be the same and therefore not the other number. So,
independent. 3x = y + 2.5
6
204 cm2 6x = 2y + 5
Since opposite sides of a rectangle must be 6x – 2y = 5 Eq 2
equal, Eq 1 is multiplied by 2 throughout by 2
2x + 2y = 14 Eq 3
2x + 3 = x + 2y + 5 and 3x – 4y + 1 = 2x + 2y
Adding eq 2 and eq 3 to eliminate the y term,
– 7.
(6x – 2y) + (2x + 2y) = 5 + 14
2x + 3 = x + 2y + 5
8x = 19
x – 2y = 2 Eq 1
___
19
3x – 4y + 1 = 2x + 2y – 7 x = ​  8 ​ 
x – 6y = – 8 Eq 2 ___
19
Using eq 1, ​  8 ​ + y = 7
Subtracting eq 2 from eq 1 to eliminate x,
(x – 2y) – (x – 6y) = 2 – (– 8) ___
37
y = ​  8 ​ 
4y = 10 ___
19 ___
37
__
5 The two numbers are ​  8 ​  and ​  8 ​  (numbers
y = ​ 2  ​
could be also fractions).
__
5
Substitute y = ​ 2  ​into x – 2y = 2, x = 7.
9 $25
Using the x and y values, Let the number of $5 notes be x and the number
length = 2x + 3 of $10 notes be y.
= 17 cm From the total number of notes,
breadth = 2x + 2y – 7 x + y = 18 Eq 1
= 14 + 5 – 7 From the total value of the notes,
= 12 cm 5x + 10y = 155 Eq 2
area = 17 × 12 = 204 cm2 To solve by elimination, we can make the x
term or the y term the same (or negatives of
7 $107, $42 each other).
Let the price of a table be x and the price of a Here it can be seen that instead of multiplying
chair be y. we can divide eq 2 by 5 throughout to obtain,
5x + 3y = 661 Eq 1 x + 2y = 31 Eq 3
3x + 7y = 615 Eq 2 Eq 3 – Eq 1, (x + 2y) – (x + y) = 31 – 18
Multiply eq 1 by 3 throughout, y = 13
15x + 9y = 1983 Eq 3 Substitute in eq 1, x + 13 = 18
Multiply eq 2 by 5 throughout, x=5
15x + 35y = 3075 Eq 4 Total value of $5 notes = 5 × $5
Subtracting eq 3 from eq 4, = $25

S47 Success In Mathematics


© SAP Group Pte Ltd

Solution.indd 47 9/7/2011 2:43:59 PM


10 48 years vertical axis. The value indicated on the
Let John’s present age be x years and his vertical axis is the conversion of 100ºC
daughter’s present age be y years. into ºF. It should be about 212ºF.
12 years ago,
(ii) –18ºC
John’s age = x – 12 years
For 0º F, the ºC equivalent temperature is
His daughter’s age = y – 12 years
where the graph cuts the ºC axis which is
Here x – 12 = 9(y – 12)
x – 12 = 9y – 108 about –18ºC.
x – 9y = –96 Eq 1 (iii) F = 1.8C + 32
16 years later, (212 – 32)
_________
slope of graph = ​   ​ 
John’s age = x + 16 years (100 – 0)
His daughter’s age = y + 16 years = 1.8
Here x + 16 = 2(y + 16) intercept at the ºF axis is 32º F
x + 16 = 2y + 32 Equation of line : F = 1.8C + 32
x – 2y = 16 Eq 2
Eq 1 – Eq 2, (x – 9y) – (x – 2y) = –96 – 16 (iv) –40
x – 9y – x + 2y = –96 – 16 When both scales have the same value let
–7y = –112 that value be x.
y = 16 x = 1.8x + 32
Using eq 2, x – 2(16) = 16 0.8x = –32
x = 48 (John’s present age) ___
32
x = –​ 0.8  ​ 
= –40
17 PRACTICAL GRAPHS __
1
3 (i) 1​    ​hour
2
1 11.5 cm __
1
1​ 2 ​ hour. (The graph is flat for Alan one hour
(Refer to Graph 9, page S49)
A graph is drawn with inches in the vertical after the start for one and a half hour.)
axis and centimetres in the horizontal axis (it
(ii) 30 km/h
could be the other way also). The inches should
have 10 units and the centimetres about 26 ______________
distance covered
Speed = ​    
   ​
time taken
units. Mark a point which corresponds to 10
inches = 25.4 cm on the graph. This is the point = 30 km/1 h
with co-ordinates (25.4, 10). Join the origin and = 30 km/h
the point (25.4, 10) and that is the conversion or
(30 – 0)
_______
graph of inches to centimetres since this is a slope of graph for first hour = ​   ​ 
(1 – 0)
linear correlation.
= 30 km/h
For finding the conversion in centimetres for
4.5 inches, mark the 4.5 inches on the vertical (iii) 10 km/h
axis. Draw a horizontal line to the graph line. Speed during Alan’s last hour of journey =
Draw a vertical line from the graph line to the slope of graph during the fourth hour
horizontal axis. The value indicated on the Taking two points on the graph, point 1 is
horizontal axis is the conversion of 4.5 inches (3, 40) and point 2 is (4, 60)
into cm. It should be about 11.5 cm.
(60 – 40)
________
slope = ​   ​ 
 
2 (i) 212º F (4 – 3)
The graph is shown with degrees Celcius in ___
20
the horizontal axis and degrees Fahrenheit = ​  1 ​ 
in the vertical axis. It is the conversion = 20 km/h
graph of ºC to ºF, and is a linear correlation 30 – 20 = 10 km/h
with an intercept.
For finding the conversion in ºF, mark (iv) 2 hr 48 min after starting
the 100ºC on the horizontal axis. Draw When the two graphs intersect, they are at
a vertical line to the graph line. Draw a the same distance from Town A (assuming,
horizontal line from the graph line to the of course, to be on the same road). The

S48 Success In Mathematics


© SAP Group Pte Ltd

Solution.indd 48 9/7/2011 2:43:59 PM


Graph 9
Topic 17, question 1.
inches

10 (25.4, 10)

4.5
4

cm
0 5 10 11.5 15 20 25 30

S49 Success In Mathematics


© SAP Group Pte Ltd

Solution.indd 49 9/7/2011 2:44:00 PM


time at the point is obtained by drawing a Since this is a graphical question requiring
vertical line from the intersection down to accurate drawing, take a point as far as possible
the time axis which is 2.8 h approximately. for better accuracy. He would have covered 60
2.8 h = 2 hr 48 min after starting km in 5 hours, so take the point representing
__
1 60 km at 1.00 pm. Draw a straight line and
(v) Ben reached ​   ​  an hour earlier
2 that would be Danny’s distance-time graph as
Alan’s destination is Town B and Ben’s shown.
destination is Town A. Alan reached his For Elise, who started at 9.00 am, draw a line
destination 4 hr after starting while Ben starting at the point representing 9.00 am and
reached his destination 3.5 hr after starting. 0 km to the point 10.30 am and 75 km. She
__
1 __
1
So Ben reached his destination first, ​ 2 ​  an would take 1​ 2 ​  hr to travel 75 km at 50 km/h.
hour earlier. For the one hour break, draw a horizontal line
from 10.30 am to 11.30 am. Again travelling
4 (i) 9.4 N
at the same speed she will complete the return
Original length = 12 cm (length when 0 N
journey at 1.00 pm. Draw another straight line
force was applied)
to represent that and the graph as shown would
Double the original length = 24 cm
be obtained.
From the graph, 24 cm length of spring is
(Refer to Graph 12, page S53)
obtained with a force of 9.4 N.
The two points in time when they met were
(Refer to Graph 10, page S51)
when Elise overtook Danny when going from
(ii) 4 cm A to B and when she passed by Danny from
From the graph, lengths at 6 N and 9 N the opposite direction later. The two times are
are 19.5 cm and 23.5 cm respectively. given approximately by the intersection points
Difference in length = 23.5 – 19.5 = 4 cm. on the graph. The times when Elise overtook
(iii) 10.2 N Danny was 1.3 hours after 8.00 am or about
The graph is a straight line up to the point 9.18 am and she met Danny on the return
(10.2, 25) approximately. Therefore, the journey around 12 noon.
extension is not proportional beyond a
force of 10.2 N. 6 25 m/s
At t = 20, the distance covered is 250 m and
(iv) Due to the same original length the graph
should start at the same point at (0, 12). let the speed at that time be v. The distance
The length for the first spring for 8 N is travelled is given by the area under the graph
about 22 cm (approx) as obtained from the as shown.
graph.
v Speed (m/s)
Extension = 22 – 12
= 10 cm
For the second spring the extension for 8 N Area = dist. travelled
__
1 0
should be ​ 2 ​  the original, ie. 5 cm. 20 Time(s)
Length of spring at 8 N = 12 + 5 __
1
Geometrically the triangle area = ​ 2 ​  (20)v
= 17 cm = 10v
So a straight line graph from the point Therefore, 10v = 250
(0, 12) to the point (8, 17) should be drawn. v = 25 m/s
(Refer to Graph 11, page S51)
7 (i) 5.5 s
5 9.18 am, 12 noon
Danny cycles at a steady speed of 12 km/h
Speed (m/s)
until he reaches town B. It will be a straight
line. Draw a straight line starting at the point 22
representing 0 km at 8.00 am. The line should
have a slope of 12. This can be obtained by
taking another point according to the distance
covered over the period of time. t
0 T Time(s)

S50 Success In Mathematics


© SAP Group Pte Ltd

Solution.indd 50 9/7/2011 2:44:00 PM


Graph 10
Topic 17, question 4(i).

30
Length of
spring (cm)

20

10

0 2 4 6 8 10 Force (N)

Graph 11
Topic 17, question 4(iv).

Length of
spring (cm)
20

10

0 2 4 6 8 10 Force (N)

S51 Success In Mathematics


© SAP Group Pte Ltd

Solution.indd 51 9/7/2011 2:44:00 PM


Taking that the vehicle reached 22 m/s at ______
338.25
Average speed = ​  19.75 ​ 
time t. change in speed
_____________ = 17.1 m/s (3 sig. fig.)
Using acceleration = ​    
  
 ​
time taken
(or slope) 8 (ii) 84T m
______
22 – 0
4 = ​  t – 0 ​  Total time taken = 3T s
___
22 (Refer to Graph 13, page S53)
= ​  t ​ 
Total distance travelled
4t = 22 = area under graph (a trapezium)
t = 5.5 s __
1
= ​ 2 ​  (T + 3T)(42)
(ii) 11 s
Taking that the vehicle started decelerating = 84T m
at time T. (iii) 28 m/s
Distance travelled is area under graph from _______________
distance travelled
Average speed = ​    
   ​
time t to time T, which is a rectangle. total time taken
242 = 22(T – t) ____
84T
= ​  3T ​ 
T – t = 11
The vehicle was travelling at a constant = 28 m/s
speed for 11 s.
9 (i) 34.4 m
(iii) 2.75 s
If the deceleration is at twice the rate, Speed (m/s)
deceleration is 8 m/s2 and the deceleration 15
took x seconds.
Taking deceleration as negative acceleration 10
and using,
change in speed
_____________
acceleration = ​    
   ​(or slope)
time taken
______
0 – 22
–8 = ​  x    ​  0 2.5 5 12 Time(s)
___
22 Mid-way through the deceleration means
= –​  x ​ 
when the car had decelerated for 2.5 s.
x = 2.75 s To find the distance travelled note that
(iv) 17.1 m/s it is given by the area under the graph
___________________
total distance travelled (trapezium) for that portion of the time.
Average speed = ​    
   ​
total time taken The parallel sides of the trapezium is given
Find the total distance travelled by finding by the speed at t = 0 and speed at t = 2.5 s.
the area under the graph. Note that the Since the car decelerated from 15 m/s to
entire figure is a trapezium and hence 10 m/s in 5 s. The speed at 2.5 s must be
the area could be found using the area of 12.5 m/s by proportion. The speed at 0 s
trapezium formula rather than by splitting is 15 m/s.
up the area into rectangles and triangles. Area of trapezium
T – t = 11 (the parallel side of the trapezium __
1
= ​ 2 ​  (sum of parallel sides)(height)
at the top)
Time taken for the whole distance __
1
= ​ 2 ​  (15 + 12.5)(2.5)
= 5.5 + 11 + 2.75
= 19.75 s = 34.4 m (3 sig. fig.)
(the parallel side of the trapezium at the
bottom) (ii) 2.5 s
acceleration during first 5 s
Area of trapezium
change in speed
_____________
__
1 = ​    
  
 ​(or slope)
= ​ 2 ​  (sum of parallel sides)(height) time taken
__
1 _______
10 – 15
= ​ 2 ​  (11 + 19.75)(22) = ​  5 ​   
= 338.25 (distance travelled in metres) = –1 m/s 2

S52 Success In Mathematics


© SAP Group Pte Ltd

Solution.indd 52 9/7/2011 2:44:01 PM


Graph 12
Topic 17, question 5.

Distance
from A
(km)
60
Danny

40

20

Elsie

0
8.00 am 10.00 am 12.00 noon Time

Graph 13
Topic 17, question 8.

Speed
(m/s)

40

30

20

10

0
T 2T 3T Time (s)

S53 Success In Mathematics


© SAP Group Pte Ltd

Solution.indd 53 9/7/2011 2:44:01 PM


deceleration during first 5 s = 1 m/s2 __
1
750 = ​ 2 ​  (2.5T)(20)
Since acceleration after 12 s = 2 m/s2
Taking the time taken for acceleration as t, = 25T
change in speed
_____________ ____
750
acceleration = ​      
 ​ T = ​  25 ​ 
time
_______
15 – 10 = 30 s
2 = ​  t ​   
__
5
= ​ t ​ 
__
5 18 CONGRUENCY AND SIMILARITY
t = ​ 2 ​ 
= 2.5 s 1 ∆BOC ≡ ∆DOC, ∆BOA ≡ ∆DOA (SSS) and
(iii) 11.3 m/s ∆BAC ≡ ∆DAC (SSS), 90º
For the average speed find the area under the Taking ∆BOC and ∆DOC,
graph. The area could be found more easily BO = DO (by symmetry along AC)
by taking the rectangle and subtracting the BC = DC (by symmetry along AC)
trapezium X as shown.
Side OC is common
The entire process took 12 + 2.5 = 14.5 s.
Therefore, ∆BOC is congruent to ∆DOC or stated

mathematically as ∆BOC ≡ ∆DOC (SSS).
15 Note that the triangles must be stated in the same
X
order of their corresponding points and hence
10 it would be incorrect to state ∆OCB ≡ ∆DOC.
Even though the two triangles referred to are
congruent, the point O does not correspond to
0 5 12 14.5 Time(s) point D in the congruence of the triangles. The
side OC does not correspond to DO either.
Area of rectangle = (15)(14.5)
= 217.5 By similar reasoning, ∆BOA ≡ ∆DOA (SSS)
__
1 and ∆BAC ≡ ∆DAC (SSS). ∠BOC = 90º
Area of trapezium = ​ 2 ​  (14.5 + 7)(5)
2 AX and BY are equal since ∆AOX ≡ ∆BOY
= 53.75
(SAS) as proven
Distance travelled = 217.5 – 53.75
Draw a line to join the points X and Y.
= 163.75
•Y
______
163.75
Average speed = ​  14.5 ​   
= 11.3 m/s (3 sig. fig.) A O B
road • • •
10 30 s
Time taken for the whole journey is T + 1.5 T •
X
=  2.5 T
The maximum speed reached was 72 km/h AO = OB (O is the mid-point of AB)
which has to be converted into m/s to make XO = OY (both are 100 m away from O)
sure that calculation units tally. ÐAOX = ÐBOY (vertically opposite angles)
72 km/h = 72000 m in 3600 s Therefore, ∆AOX ≡ ∆BOY (SAS)
________
72000 Once the triangles are proven to be congruent all
= ​    ​ 
3600 m/s corresponding sides and angles in the triangles
= 20 m/s will be equal based on the congruency. Hence,
The acceleration and deceleration were constant AX = BY
and hence the graph consists of straight lines
3 10 cm
which form a triangle of height 20 (the maximum
For easier reference, the two triangles could be
speed) and a base 2.5 T (total time taken).
drawn separately as shown.
Using distance travelled = area of triangle
__
1
= ​ 2  ​(base)(height)

S54 Success In Mathematics


© SAP Group Pte Ltd

Solution.indd 54 9/7/2011 2:44:02 PM


B Therefore, the ratio QY : QR is 5 : 7.
(iii) 25:49
9 cm 15 cm
If the corresponding length ratio QY : QR is
5 : 7, the corresponding area ratio is 52 : 72
or 25 : 49.
A C
Q Therefore, the ratio area of ∆QXY : area of
the ∆QPR is 25 : 49.
6 cm
5 (i) A
P x cm C

Since triangle ABC is similar to triangle QPC,


the corresponding sides could be paired to P Q
equate their ratios. Hence,
___
AB ___ BC ___ AC
​ QP  ​ = ​ PC  ​= ​ QC  ​ 
B R C
Using the known values, the ratio could be ∆ABC and ∆PQR are the first and second
___
AB __ 9 triangles respectively.
determined as ​ QP  ​ = ​ 6  ​
___
BA
__
3 ​ PA ​ = 2 (P is the mid-point of BA)
 = ​ 2 ​ 
AC
___
___
AB ___ BC ___ AC __ 3 ​ AQ  ​= 2 (Q is the mid-point of AC)
Therefore ​ QP  ​ = ​ PC  ​= ​ QC  ​ = ​ 2 
ÐBAC is common to both ∆BAC and
 ​ Let length of PC = x cm. ∆PAQ.
___
15 __
3 ∆BAC and ∆PAQ are similar (when two
To find x use BC = 15 and the ratio, ​  x ​ = ​ 2 ​ 
pairs corresponding sides of the two
Cross multiplying 3x = 30 triangles are of the same ratio and the
x = 10 cm corresponding pair of angles between the
two sides are equal).
4 (i) QX and QP, XY and PR, QY and QR Therefore, ∆PAQ is an equilateral triangle.
First prove that ∆QXY is similar to ∆QPR. Similarly, ∆BPR and ∆RQC are also
ÐQXY is common to both ∆QXY and equilateral and are congruent to ∆PAQ.
∆QPR Hence ∆PQR is an equilateral triangle
ÐQXY = ÐQPR (corresponding angles) which is similar to ∆ABC.
Therefore, ∆QXY is similar to ∆QPR
(ii) 256 : 1
(AAA). The 3 pairs of sides with the same Since the corresponding length ratio is 2 : 1
length ratio are (a) QX and QP, the corresponding area ratio is 22 : 12 or 4 : 1.
(b) XY and PR, Therefore, ∆PAQ is an equilateral triangle
(c) QY and QR. __
1
which is ​ 4 ​  the area of ∆BAC. Similarly,
These pairs are the corresponding sides of
∆QXY and ∆QPR as they are proven to be ∆BPR and ∆RQC are also equilateral
__
1
similar. triangles each ​ 4 ​  the area of ∆BAC. Hence,
Note that the third pair of angles need not __
1
∆PQR is also an equilateral triangle with ​ 4 ​ 
be proven to be equal as it will be equal if
the area of ∆BAC.
the other two pairs are equal due to sum of
(It should be noted at this stage that any
angles in a triangle.
two equilateral triangles will be a pair of
(ii) 5 : 7 similar figures and for that matter any two
If PX to XQ is 2 : 5, PQ to XQ is 7 : 5. regular polygons with the same number of
sides will be a pair of similar figures.)
QX ___
___ QY __ 5
Based on similarity ​ QP  ​= ​ QR  ​= ​ 7 ​ 

S55 Success In Mathematics


© SAP Group Pte Ltd

Solution.indd 55 9/7/2011 2:44:02 PM


__
1 Draw a line XY through R which will be
The second triangle is ​ 4 ​ the area of the first.
perpendicular to both PQ and ST since PQ
By the same reasoning, the third triangle and ST are parallel.
__
1
(  ) __
1 2
is ​ 4 ​  the area of the second or ​​ ​ 4 ​   ​​ ​the area P 10 cm X
of the first. The fifth triangle by extension Q

(  )
__
1 4 ____ 1
should be ​​ ​ 4  ​  ​​ ​ or ​ 256   ​ the area of the first R
as they are all similar equilateral triangles.
Ratio of the area of first triangle : area of
S
fifth triangle = 256 : 1.
Y 15 cm
T
(iii) 16 : 1
Since the area ratio of first triangle : fifth XR will be the height of ∆PQR taking PQ
triangle is 256____ : 1, corresponding length as the base.
__
ratio must be √ ​ 256 ​ : √
​ 1 ​ or 16 : 1. YR will be the height of ∆RST taking ST as
Ratio of the perimeter of first triangle : the base
perimeter of fifth triangle = 16 : 1 Find XR using the ∆PQR which has an area
(The ratio also could be obtained by noting of 40 cm2. Let XR be h.
that the sides of the second triangle would __
1
Area of ∆PQR = ​ 2 ​  bh
__
1
be ​ 2 ​  the first and the perimeter would also __
1
40 = ​ 2 ​  (10) h
__
1
be ​ 2 ​  the first. By extension, perimeter of the h = 8 cm
(  )
__
1 4 ___ 1
fifth triangle would be ​​ ​ 2 ​   ​​​or ​ 16  ​ of the first.) YR could be found using the area of ∆RST
or by similarity of ∆PQR and ∆RST.
6 (i) 12 cm Using corresponding lengths of ∆PQR and
Let length of RT = x cm. Length of PR will 3(8)
____
be (20 – x) cm. ∆TSR being 2: 3, height of ∆TSR is ​  2 ​   =
∠PRQ = ∠SRT (vertically opposite angles) 12 cm.
∠QPR = ∠RTS (alternate angles) Length of XY = 12 + 8
∆PQR and ∆TSR are similar (AAA) = 20 cm
Using ratio of corresponding sides of (distance between parallel
similar triangles,  lines of the trapezium SPQT )
PQ ___
___ PR ___
10 __
1
​ TS ​ = ​ TR  ​= ​ 15 ​  Area of trapezium SPQT = ​ 2 ​ (10 + 15)(20)

___
10 ______20 – x = 250 cm2
​ 15 ​ = ​  x   ​ 
(iv) 8.66 cm
__
2 ______20 – x
​ 3  ​= ​  x    ​ P 10 cm
2x = 3(20 – x) Q
2x = 60 – 3x
R
x = 12 cm
M x
(ii) 90 cm2
S N
corresponding length ratio of ∆PQR and
___
10 15 cm
∆TSR is ​ 15 ​ or 2 : 3. T
Let length of MN = x
corresponding area ratio of ∆PQR and
ÐSRT is common to both ∆RST and
∆TSR is 22 : 32 or 4 : 9. ∆RMN
If area of ∆PQR is 40 cm2, the area of ∆RST ÐRMN = ÐRST (corresponding angles)
9(40)
_____ ∆RST and ∆RMN are similar (AAA)
is ​  4 ​ 
 = 90 cm2.
__
1
If ∆RMN is ​ 3 ​  the area of ∆RST,
(iii) 250 cm2

S56 Success In Mathematics


© SAP Group Pte Ltd

Solution.indd 56 9/7/2011 2:44:03 PM


area ratio of ∆RMN and ∆RST __ __
is 1 : __3, or 343 : 512.
corresponding length__ratio is √
​ 1 ​ : √
​ 3 ​ or 1 : √
​ 3 ​ . If the volume of the larger cylinder is 200 cm3,

​ 
Length of
____________ ___
Length of MN 1
ST
  
​ 3 ​ 

   ​= ​   ​ 
____
343
(  )
the volume of the smaller cylinder is 200​ ​ 512 ​  ​
__ = 134.0 cm3 (1 decimal place)
___
15
​  x ​ = ​√3 ​ 
___
15 10 45 cm2
x = ​  __  ​ 
​ 3 ​ 
√ If two similar figures have a volume ratio__8 : 27, ___
= 8.66 cm 3 3
their corresponding length ratio must be ​√  8 ​ : ​√  27 ​ 
7 10 km, 6 km2 or 2 : 3.
Using similar figures, length ratio is 1 : 200 000 Their corresponding area ratio must be 22 : 32
The length ratio applies to perimeter. or 4 : 9.
The forested area on the map shown with If the base area of the smaller cone is 20 cm2, the
a perimeter of 5 cm would have an actual
perimeter:
__
9
(  )
base area of the larger cone is 20​ ​ 4 ​   ​= 45 cm2.

5 × 200 000 cm 11 14.9 cm


= 1 000 000 cm Since the displayed figure and the scale model
= 10 km are similar figures, let their corresponding
Using the length ratio 1 : 2 × 105 (in standard length ratio be x : y.
form) The corresponding volume ratio is x3 : y3.
The area ratio is 12 : (2 × 105)2 (taking the square x3 : y3 = 80 : 3
of the length ratio) x3 ___
__ 80
​  3 ​  = ​  3 ​ 
= 1 : 4 × 1010 y
A 4 × 1010 cm2 area on the ground will appear
as 1 cm2 area on the map.
(  )
__x 3 ___ 80
​​ ​ y ​  ​​ ​= ​  3 ​ 
___
Since the area appears as 1.5 cm2 on the map,
the actual area must be

__x 3 ___ 80
​ y ​= ​  ​  3 ​ ​  
___


3 ___
80
1.5 × 4 × 1010 cm2 The length ratio x : y becomes ​  ​  3 ​ ​  : 1.
= 6 × 1010 cm2
1 km2 = 100 0002 cm2 = 1010 cm2 If the length of the sword of the model is 5 cm,
Area of forest = 6 km2 y = 5, the length of the sword of the displayed
figure___ will be x cm.
8 8:5
Since two circles are similar plane figures (a √
__x 3 ___ 80
​ y ​ = ​  ​  3 ​ ​  
___
pair of regular polygons of infinite number of
sides), three-quarter circles are also similar
x 3 ___
__
√ 80
​ 5  ​= ​  ​  3 ​ ​  
___
( √ )
as they are formed by cutting off the same 3 ___
80
fractional portions (quadrants) of both figures. x = 5​ ​  ​  3 ​ ​    ​
___
( √ )
Area of larger figure : Area of smaller figure 3 ___
80
= 320 : 125 = 5​ ​  ​  3 ​ ​    ​
= 64 : 25 ___ ___ = 14.9 cm (3 sig. fig.)
Corresponding length ratio is ​√ 64 ​ : ​√ 25 ​ or 8 : 5.
Perimeters of the two figures are considered as __
1
12 10 cm, ​    ​
corresponding lengths. 4
The ratio,
Perimeter of larger figure : perimeter of smaller
figure is 8 : 5.
20 cm
9 134.0 cm3 (1 decimal place)
If two similar figures have an area ratio___49 : 64,___ B
A
their corresponding length ratio must be √
​ 49 ​ : √
​ 64 ​ 
or 7 : 8. We can take the shape of the water in the cone
Their corresponding volume ratio must be 73 : 83 to be a cone (referred as cone A) and it also is

S57 Success In Mathematics


© SAP Group Pte Ltd

Solution.indd 57 9/7/2011 2:44:03 PM


similar to the container cone (referred as cone (i) The 5 sets are birds, insects, mammals,
B). reptiles, and vertebrates.
Volume ratio of cone A: cone B is 1 : 8.__ __ The 9 members are ants, bears, crocodiles,
3 3
corresponding length ratio must be ​√  1 ​  : ​√  8 ​  eagles, elephants, grasshoppers, snakes,
or 1: 2. sparrows and vultures.
corresponding area ratio must be 12 : 22 or 1 : 4. (ii) The set {birds} is a subset of {vertebrates}.
Using corresponding length ratio of 1 : 2, if the It could also be the {mammals} or {reptiles}
height of the container cone B is 20 cm, height being a subset of the {vertebrates}.
of cone A (volume of water) is 10 cm. (iii) a suitable name for the universal set will be
Using corresponding area ratio of 1 : 4, the animals.
curved area of cone B is the portion of the
(iv) the Venn Diagram will be as follows:
curved area of A which is in contact with the
e
water. Therefore, the fraction of the container vertebrates (animals)
__
1 snakes
in contact with the water is ​ 4  ​. E crocodiles Key:
E - eagles
S reptiles S - sparrows
V V - vultures
grasshoppers bears
birds Sn - snakes
19 SET LANGUAGE ants mammals elephants C - crocodiles
B - bears
insects El - elephants
G - grasshopper
1 (i) 11 A - ants
Here ε = {whole numbers less than 20} is
(v) an animal which is an invertebrate which
quite a short list, and could be written as
ε = {0, 1, 2, 3, 4, 5, 6, 7, 8, 9, 10, 11, 12, 13, falls outside the classification of insects
14, 15, 16, 17, 18, 19}. would be suitable for this purpose, eg.
A = {2, 3, 5, 7, 11, 13, 17, 19} according to earthworm, snail, jellyfish, etc.
the definition of prime numbers which are 3 (i) Venn Diagram as shown:
less than 20. e
B = {1, 3, 5, 7, 9, 11, 13, 15, 17, 19} T Pa P
according to the definition of odd numbers Re Rh (plane figures)
which are less than 20
S
A  B is the set which lists out ALL the
members of both sets without repetition.
Q
A  B = {1, 2, 3, 5, 7, 9, 11, 13, 15, 17, 19}.
n (A  B) = 11 (ii) A  B means parallelograms which possess
(ii) 7 the features of a rectangle as well as that of
A  B is the set which lists out the members a rhombus. The parallelograms which fall
which are found in both sets under this category are squares.
A  B = {3, 5, 7, 11, 13, 17, 19} (iii) C  D will be the set D = {quadrilaterals}
n (A  B) = 7 as set C is a proper subset of set D.
(iii) 12 (iv) E  F = f there are no members common to
A’ is the set which lists out the members of set E and set F.
ε NOT in set A 4 (i) 33, 13
A’ = {0, 1, 4, 6, 8, 9, 10, 12, 14, 15, 16, 18} Set A = {x: x is an integer divisible by 3}
n(A’) = 12
= {3, 6, 9, 12, , 99}
(iv) 1 n(A) = 33 (as there are 33 members)
B’ = {0, 2, 4, 6, 8, 10, 12, 14, 16, 18}. Set B = {x: x is an integer divisible by 7}
(actually the even integers < 20) = {7, 14, 21, 28, , 91}
A  B’ ={2} (the only even number which n(B) = 13 (as there are 13 members)
is also a prime number) Note that both the sets have to comply with
n(A  B’) = 1 the requirements of the universal set ie. the
members must be < 100.
2 (as follows)

S58 Success In Mathematics


© SAP Group Pte Ltd

Solution.indd 58 9/7/2011 2:44:03 PM


(ii) 4 (ii) 10, 22
A  B = {21, 42, 63, 84} (these are the n(A  B) is minimum when A  B covers
members which fall in both sets A and B. the entire class such that set A and set B are
n(A  B) = 4 as wide apart as possible with minimum
overlap. Therefore,
(iii) 29 when n(A  B) = n(ε) = 37
A  B’ means a member of set A which is n(A  B) = x.
not a member of set B. For this, we have 22 + (25 – x) = 37
to take the list of members in set A and 22 + 25 – x = 37
exclude all those from set B. Alternatively, x = 10
we could also take the list of members in n(A  B) is a maximum when A is a proper
set B’ and pick all those from set A to be subset of B. In which case n(A  B)
the members of A  B’. The second option = n(A)
would be more tedious but would yield the = 22
same outcome.
(iii) 13
Hence A  B’ = {x : x is an integer <100
If 3 students play neither of the games, let
divisible by 3, x ≠ 21, 42, 63, 84}
n(A  B) = x.
n(A  B’) = n(A) – 4
n[(A  B)’] = 3
= 29
n(A  B) = 34
(iv) 57
The situation can be shown on a Venn
(A  B) = {x : x is a positive integer <100
Diagram as follows,
which divisible by 3 or 7 }

n(A  B) = n(A) + n(B) – n(A  B )
If n(A) and n(B) are added as there will be A B
some double counting due to n(A  B) as 22 – x x 25 – x
there are integers which are divisible by
3
both 3 and 7.
Since n(A  B) = 42, n[(A  B)’]
22 + (25 – x) = 34
= n(ε) – 42 22 + 25 – x = 34
= 99 – 42 x = 13
= 57
(iv) as shown
5 (i) 25, 37 (A’  B’) means the intersection set of
The minimum value of n(A  B) is when those who do not play soccer and the set
set A is a proper subset of B or A  B. A of those who do not play basketball. First
could be a proper subset of B since n(A) < shade A’ and then B’. It is shown on a Venn
n(B). In that situation, all students who play Diagram as follows.
basketball also play soccer and therefore, It can also be (A  B)’.
A  B = B.

A
B
6 (i) as shown
The five sets of triangles are shown on a
n(A  B) = n(B) = 25 Venn Diagram as follows:
The maximum value of n(A  B) is when e
A  B = ε when set A is as wide apart as Obtuse all triangles
possible. This is possible as n(A) + n(B) ≤ Right
n(ε). In that situation,
A  B = ε. E
Acute
n(A  B) = n(ε) = 37
Isosceles

S59 Success In Mathematics


© SAP Group Pte Ltd

Solution.indd 59 9/7/2011 5:21:10 PM


(ii) {equilateral triangles}, {isosceles triangles} is a set of positive integer values which are
If X is a subset of Y and X is also a the solutions of 2x – 3 > 17. A set of positive
subset of {acute-angled triangles}, X is integer values which is the solutions of
the set {equilateral triangles}, Y is the set both inequalities will be (A  B).
{isosceles triangles} as shown in the Venn
(ii) as shown
Diagram.
The sets A and B drawn in a Venn Diagram
(iii) 6.83 cm (3 sig. fig.) are as follows.
(I  R) = {all triangles with angles 90º, 45º,
e
45º}, (must be right-angled isosceles).
A 17 18 B
19 20
1 2 3 11
21 22
4 5 6 12 15
23 24
13 16
y 7 8 9 25 26
x 14
10 27 28
29

x (iii) 6
__
1 From the Venn Diagram, n(A  B) = 6
​ 2 ​ x2 = 2 (given)
(the integers 11, 12, 13, 14, 15 and 16)
x2 = 4
x=2 (iv) {all odd integers < 30}
y2 = x2 + x2 (Pythagoras’ theorem) It can be seen that (A  B) is the entire

= 8__ universal set. Therefore (A  B)  C = C.
y=√ ​ 8 ​  Hence (A  B)  C = {all odd integers < 30}
__
P = 2 + 2 + ​√8 ​  9 (i) To find A  B, first shade the area
= 6.83 cm (3 sig. fig.) representing set A and then shade the area
7 (i) 17 representing set B. The entire shaded area
A  C means the members of the universal is the set A  B as shown.
set which are either a multiple of 3 OR a
A B
multiple of 7 (it is not to be interpreted that
a member must be a multiple of 3 and a
multiple of 7). A  C = {3, 6, 7, 9, 12, 14,
15, 18, 21, 24, 27, 28, 30, 33, 35, 36, 39}
n(A  C) = 17
(ii) 37
A  B = {15, 30}, n(A ∩ B) = 2. (ii) To find A’  B, first shade the area
n{(A  B)’} = n(ε) – 2 representing set A’ (outside of A) and then
= 39 – 2 shade the area representing set B. Use
= 37 different shades for the two shadings. The
shaded area containing both shades is the
(iii) 4 set A  B as shown.
The set C  B’ is actually the multiples of 7
with the multiples of 5 removed.
A B A B A’  B
C = {7, 14, 21, 28, 35}
As set B’ contains the non-multiples of 5,
C  B’ will contain the elements common
to both C and B’, C  B’ = {7, 14, 21, 28}. (iii) For (A’  B)’ simply refer to (A’  B) and
n(C  B’) = 4 take whatever portion of the Venn Diagram
not covered and that will be the (A’  B)’ as
8 (i) A  B, shown.
Set A is a set of positive integer values
which are the solutions of x + 2 < 19. Set B

S60 Success In Mathematics


© SAP Group Pte Ltd

Solution.indd 60 9/7/2011 2:44:04 PM


To represent A’, shade the area outside set
A in the universal set. To represent B’, shade

A B the area outside set B in the universal set. You
will find that the area left unshaded is the area
(A    B). This means that (A’  B’) = (A  B)’
(this is actually a simplification process for any
two sets A and B in a universal set).
(iv) For (A  B)  B’, shade the area (A  B) as Therefore (A’  B’) = (A  B)’ = {all the letters
found earlier. Then shade the area outside except g, h, j, m, n}.
set B to represent B’ using a different shade
as this involves an intersection set. The
area which contains both shadings is the 20 MATRICES
(A  B)  B’.
1 a = 7, b = –3, c = 2 and d = 1.
Adding the corresponding terms,
a – 3 = 4, a = 7
–b – (–4) = 7, b = –3
A B 2 – c = 0, c = 2
7 – 6 = d, d = 1
10 (i) (A  B) = {c, d, f, g, h, i, j, k, l, m, n, o, p}
__
2
(the union set is formed by taking the 2 a = –17, b = –19, c = –​    ​, k = –3
3
members of both sets A and B and listing First find k, using the known corresponding
the combined set without repetitions) elements in the three matrices.
(A  B)’ = {a, b, e, q, r, s, t, u, v, w, x, y, z}
Using element (1,1) in the three matrices,
(the complement set is formed by taking
3 – 2k = 9 which gives
the members of A  B and excluding them
from the universal set) k = –3.
(ii) (A  B) = {g, h, j, m, n} (these are the After that the matrices can be written as,


letters in both sets A and B)
(A  B)’ = {all the letters except g, h, j, m, n}
[  ] [  ] [  ]
3
​    –b ​  ​– (–3)​   
​a  5 2
​6  –4 9  7
  
c ​  ​= ​ ​1  3​  ​

​[    ​ ]​+ 3​[    c ​  ]​= ​[ ​1  3​  ]​


(To find the complement of a set X, the 3
universal set must be used and from which ​a  –b5  
2
​6  –4 9  7
  
the elements of set X are to be excluded. The
remaining elements are the complement of ​[   
3
​a  –b ​ ]​+ ​[    
5  
6
​   –12 ​  
18   3c ]​= ​[   

]
7​  ​
​1  3
set X.)
Adding the corresponding terms,
(iii) A’={a, b, e, i, k, l, q, r, s, t, u, v, w, x, y, z}
–b + (–12) = 7, so b = –19
(A’  B) = {i, k, l} (elements common to
set A’ and B) a + 18 = 1, a = –17
(iv) (A’  B’) can be found by listing out the __
2
5 + 3c = 3, c = – ​ 3  ​
A’ and B’ and then combining the two sets.
However, an easier method will be using 3
the Venn diagram. 19 26 33
Draw two overlapping sets A and B in a 29 40 51
universal set as shown. 39 54 69


34 40
A B 79 94
A is 3 × 2 matrix (3 rows and 2 columns) while
B is a 2 × 3 matrix (2 rows and 3 columns). AB
will be a 3 × 3 matrix,

S61 Success In Mathematics


© SAP Group Pte Ltd

Solution.indd 61 9/7/2011 2:44:05 PM


AB is not defined since number of columns
a b c of A ≠ number of rows of B.
d e f BA is defined since number of columns of
g h i B = number of rows of A.
So C = BA
a is the element (1,1) of AB which the sum of Order of C is 1 × 2.

[  ]
the products of the corresponding elements in
row 1 of A and column 1 of B,   2   3
(ii) C = [2 –4 5] ​  ​ 4  ​  ​
  1 
 ​   ​
a = 3 × 1 + 4 × 4 = 19 –3 2
Similarly, b = 3 × 2 + 4 × 5 = 26 = [ 2 × 2 + (–4) × 4 + 5 × (–3) 2×3
c = 3 × 3 + 4 × 6 = 33 + (–4) × 1 + 5 × 2]
d = 5 × 1 + 6 × 4 = 29 = [–27 12]

[  ]
e = 5 × 2 + 6 × 5 = 40
f  = 5 × 3 + 6 × 6 = 51 –1
6 ​     
​ 8 ​  ​  0 ​   ​
g = 7 × 1 + 8 × 4 = 39 __ __ 5
–​   ​  –​ 3 ​ 
h = 7 × 2 + 8 × 5 = 54 3
i = 7 × 3 + 8 × 6 = 69 2A = 2​ 4  [  ] [  ]
2   0 3  ​= ​ 4   0
8   6  ​
BA will be a 2 × 2 matrix,
a b
2A + 3B = ​ 4 [  ]   0
8   6  ​+ 3B
c d For the operation to be defined, C must be a 2 × 2
matrix which should be an identity matrix
a is the element (1,1) of BA which the sum of
[  ]
1   0  ​
the products of the corresponding elements in ∴ C = ​ 0  1


row 1 of B and column 1 of A,
a = 1 × 3 + 2 × 5 + 3 × 7 = 34

[  ]
​ 8   6 0
[  ]
    ​+ 3B = ​ 1   0
0   1  ​


Similarly, b = 1 × 4 + 2 × 6 + 3 × 8 = 40
c = 4 × 3 + 5 × 5 + 6 × 7 = 79
[  ] [  ]
3B = ​ 01   1  0  ​– ​ 4    0  ​
8  6
d = 4 × 4 + 5 × 6 + 6 × 8 = 94 = ​  –3

​–8​    ​ –5 ]
0​   ​


4 h = 2, k = –1
[  ]
Matrix I must be a 2 × 2 and should be ​ 1  0
     ​
0  1

__
1 –3  0
B = ​ 3 ​ ​    [  ]
​–8​  ​ –5​   ​= ​ –1  

8   __
__ ]
    0   ​
5
 –​ 3 ​    – ​ 3 ​ 
since it is an identity matrix.
[  ] [  ] [  ]
h​ 2  3
     ​+ k​ 3  6
0  1
     ​= ​ 1  0
0  1
     ​gives
0  1
7 ​ –9
   [ 
  ​   
​–10 ​ –1 ] [ 
4​   ​, ​    16 
​ –39​  39
  ] [  ]
​ 18​   –6
–11
​55​   ​, ​      
​–23
  ​  ​
2h + 3k = 1 – Eq 1 A = [​  1
2      ]​​[ 1   –2
 –2
5  –3     ]​(not the individual
5  –3
3h + 6k = 0 – Eq 2  elements squared)


h + k = 1 – Eq 3


From Eq 2: 3h + 6k = 0
h + 2k = 0
1 × 1 + (–2) × 5   1  × (–2) + (–2) × (–3)
​ 5 × 1 + (–3) ×   5   5 × (–2) + (–3)   
× (–3)  ​ ]


h = –2k
Sub in Eq 3: –2k + k = 1
–9
= ​   
​–10 [ 
  ​    4​   ​
​ –1 ]


–k = 1
k = –1
1
AB = ​   [  ][ 
​–3​   ​​ 3
–2
​5​           –1​   ​
​4​   ​ 1 ]


h=2
Substituting in Eq 1, the values for h and k, it
[ 1 × 3 + (–2) × 4   1  × (–1) + (–2) × 1
= ​ 5 × 3 + (–3)      
× 4   5 × (–1) + (–3) × 1  ​ ]

tallies.
∴ h = 2, k = –1

= ​ –5    –3
  3   –8   ​ ]
5 3 × 2, 1 × 3, 1 × 2, [–27 12] –5
(AB)2 = ​     
​ 3​  ​ [ 
–3​   ​​  
–5   
–8 ​ 3​   ​ –8][ 
–3​   ​
]
(i) Order of A: 3 × 2
Order of B: 1 × 3
Since product of A and B is either AB or
​[ 
(–5) × (–5) + (–3) × 3
= ​            
  ​
3 × (–5) + (–8) × 3
(–3)

× (–5) + (–3) × (–8)
            
3 × (–3) + (–8) × (–8)
 ​  ​]
BA. [ 
= ​   
​ –39 39
16​   
​55​   ​ ]

S62 Success In Mathematics


© SAP Group Pte Ltd

Solution.indd 62 9/7/2011 2:44:05 PM


[  ] [  ] [  ]
​–3​   ​+ ​ 3 ​4​   
A + B = ​  1​5​   
–2 –1 –3
​ 1​   ​= ​  4​9​   
​–2​   ​ 10 (i) ​  [ 
11 23
​23 ​   20
​25​    ] [  995
]
      ​
​18​   ​, (ii)[​ 11  23  20 ]​, (iii) ​ 1265

(A + B) = [​   4​9​   
​–2​  ]​​[  4​9​   
​–2​  ]​
–3 –3 (iv) [2260]
2


(iii) ​ 11
​  23
  20   ]
[  ]
23 ​ ​25​  ​18​   ​C = D
4 × 4 + (–3) × 9   4 × (–3) + (–3) × (–2)
= ​              
​      ​  ​
9 × 4 + (–2) × 9   9 × (–3) + (–2) × (–2) Since A is a 2 × 3 matrix and D is a 2 × 1


–11
= ​    –6
​ 18  ​   
​   ​  ​
–23 ] matrix C must be 3 × 1. If D is the total
sales value, C must be the price of each

8
30
___ 3
__
x = ​  7 ​  , y = – ​ 7 ​  item. Hence, C = ​ 25  [  ]
20
 
​ ​   ​
 
10

[  ] [  ] [  ] ][  ]
 x + 3y
​ –23 ​  ​​  x​y​   ​ = ​4x –  2y   ​
AB = ​  1​4​   

20

D = ​ 11
​  23
  20    
23 ​ ​25​  ​18​   ​​ 25 
​ ​   ​
 

[  ] [  ]
x + 3y 10
If ​ 4x –  2y   ​= ​ 18 3     ​


11 × 20 +  
= ​ 23 23 ×  25 + 20 × 10  ​
× 20 + 25 × 25 + 18   
× 10 ]
We have a pair of simultaneous equations.
x + 3y = 3 --- = ​[ 1265
     ]​
995
4x – 2y = 18 ---
 × 4, 4x + 12y = 12 --- (iv) Since B is the sales of each item at branch
 – , 14y = –6 X, C is the price of each item and F is the
___
6 total sales value, E must be the sales of
y = – ​ 14  ​  each item at branch Y.
__
3 B + E = [11 23 20] + [23 25 18]
= – ​ 7 ​  = [34 48 38]
(  )
[  ]
__
3 F = (B + E)C
Sub in , x + 3​ – ​ 7 ​   ​= 3 20
 
__
9 = [34 48 38]​ 25 
​ ​   ​
 
 x = 3 + ​ 7  ​
10
___
30 = [2260]
= ​  7 ​ 
___
30 __
3
 x = ​  7 ​ , y = – ​ 7 ​ 
21 CIRCLE PROPERTIES
1
9 ​   0
​0​   [  ] [  ] [  ] [  ]
​1​   ​, ​  1​2​    ​ –10​   ​, ​  1​0​   0​1​   ​
​ –10​   ​, ​  1​2​    1 24 cm
A = ​[   –1​  ]​
1 0 Let XB = y cm
​2​  ​   OB = 13 cm
A = [​   1​2​   
​ –10​  ]​​[  1​2​   
​ –10​  ]​
Using Pythagoras’ Theorem, 132 = y2 + 52
2
y2 = 169 – 25
[  1 × 1 + 0 × 2   1  × 0 + 0 × (–1)
= ​ 2 × 1 + (–1)   
× 2   2 × 0 + (–1)  × (–1)  ​ ]

= 144
y = 12 cm
AB = 2y (radius perpendicular to chord)
[  ]

= ​ 1  0
0   1  ​(identity matrix) = 24 cm
A = A2 A
3
2 32 cm
[  ][ 
= ​ 10   0  1  ​​ 1 ​2​  ​ 
   0
] [  ]
1   0
–1​   ​= ​ ​2​  ​ –1​   ​(since IA = A)
Since chord BC is bisected by the diameter, the
angles at X are 90º.
If multiplication is continued A4 = A3A
Let radius of the circle be r cm.
= AA
OX = r – 2 cm
= A2
OC = r
=I
Using Pythagoras’ Theorem, r2 = (r – 2)2 + 82
∴ An = I, where n is even
r2 = r2 – 4r + 4 + 64
[  ]
An = ​  1​2​   ​ –10​   ​or An = A when n is odd.

4r = 68
r = 17 cm
Hence A11 = ​ 1 ​2​    [  ]
​ –10​   ​and A12 = ​ 1 ​0​  0 ​1​   ​. [  ]

S63 Success In Mathematics


© SAP Group Pte Ltd

Solution.indd 63 9/7/2011 2:44:06 PM


AX = 2(17) – 2 X
= 32 cm
3 1.15 cm, 7.79 cm
Let the distance of AB from centre be x cm and O
the distance of CD from centre be y cm.
A xº
Using Pythagoras’ Theorem to find x, C
62 = x2 + 52 110º
x = 3.32 cm (3 s.f.) B
Using Pythagoras’ Theorem to find y,
62 = y2 + 42 ∠AXC = 180º – 110º (opposite angles in a
y = 4.47 cm (3 s.f.)  cyclic quadrilateral)
Distance between the lines = 4.47 – 3.32 = 70º
= 1.15 cm ∠AOC = 140º (angle at centre is double angle
If the lines are located on both sides of the at circumference)
centre of the circle, the distance between them __________
180º – 140º
∠OCA = ​  2 ​   
would be the sum of the individual distances
instead of the difference. = 20º
ie. distance between the lines = 4.47 + 3.32 7 38º
= 7.79 cm Draw a perpendicular line from C through the
4 20º, 75º centre to touch circumference at D. Join A and
D.
∠XYC = 90º (angle subtended from the D
 diameter)
∠XCY = 180º – 70º – 90º
= 20º (Ðx)
∠XYA = 180º – 90º
B
= 90º 38º
∠AXY = 180º – 35º – 90º A
= 55º

∠BXC = 180º – 55º – 70º
C
= 55º
∠BYC = 55º (angle subtended from the same ∠ADC = 38º (angle subtended from the same
chord BC) chord AC)
y = 55º + 20º ∠DAC = 90º (angle subtended from the
diameter)
= 75º
∠ACD = 180º – 38º – 90º
5 57º = 52º
Draw a chord BD ∠x = 90º – 52
∠CBD = 90º (angle subtended from the = 38º
diameter)
8 7 cm, 13.4 cm (3 sig. fig.)
∠BDC = 180º – 33º – 90º
∠OCX = 90º (angle between radius and tangent)
= 57º Let radius of circle be r cm which forms a right-
∠CAB = 180º – 57º angled triangle as shown from the centre of the
= 123º circle.
x = 180º – 123º
= 57º
18 + r
r
6 20º
Draw two lines from points A and C to meet at
24
X.

S64 Success In Mathematics


© SAP Group Pte Ltd

Solution.indd 64 9/7/2011 2:44:06 PM


Using Pythagoras’ Theorem to find r, the chord BD)
(18 + r)2 = r2 + 242 ∠x = 42º + 42º
324 + 36r + r2 = r2 + 576 = 84º
36r = 252 ∠y = 180º – 42º (angles in a cyclic quad)
r = 7 cm = 138º
AX = CX (tangent lines from the same point X )
11 74º
OC = OA (radii)
Since ∠AED = 90º AD must be the diameter
OX is common.
(angle subtended from the diameter)
Therefore, ∆OCX ≡ ∆OAX and AC is
ABCD becomes 3 isosceles triangles AOB,
perpendicular to OX (as OX is the line of
BOC and COD.
symmetry for the kite figure OAXC)
AC = 2 × height of ∆OCX with OX as the base ∠COD = 180º – 2(42º)
__
1 = 96º
Area of ∆OCX = ​ 2 ​ (24)(7) ∠BOA = 180º – 96º – 52º
= 84 cm2 = 32º
__
1 _________
180º – 32º
84 = ​ 2  ​ (25)(height)(considering OX as the base) ∠ABO = ​  2 ​   
height = 6.72 cm = 74º (∠x)
AC = 2 × 6.72
12 70º
= 13.4 cm (3 sig. fig.)
If a line OA is drawn, two isosceles triangles
9 15.5 cm (3 s.f.) which are congruent would be obtained.
Let the required length be x cm. ∆ABO ≡ ∆ADO.
Draw the circles and let the centre of the small Therefore ∠ABO = ∠ADO
and the large circles be A and B as shown. As = ∠BAO
the angles between the radii and the tangents = ∠DAO
are 90º, a trapezium is formed. = 20º
∠BAD = 40º
X Y
∠BCD = 40º (angle from the same chord BD)
∆CXD is an isosceles triangle
_________
180º – 40º
∠CXD = ​  2 ​   
= 70º (∠y)
13 93º
X x cm Y
Draw a line BE.
6 cm ∠CBE = 180º – 87º (opposite angles in a cyclic
10 cm quadrilateral)
A = 93º
16 cm ∠ABE = 180º – 93º
B
= 87º
Draw a line parallel to XY from A to YB and a ∠AFE = 180º – 87º (opposite angles in a cyclic
right-angled triangle is obtained. quadrilateral)
x cm = 93º (∠x)
14 1.40 cm (3 s.f.)
4 cm ∠BAD = ∠BCD (angles subtended from the
16 cm
 same chord)
162 = x2 + 42 (Pythagoras’ theorem) ∠ABC = ∠CDA (angles subtended from the
x = 15.5 cm (3 s.f.)  same chord)
Therefore, ∆ABX is similar to ∆CDX.
10 84º, 138º AD = diameter of circle = 10 cm
∠BCD = 42º (alternate angles)
XD = (10 – x) cm
∠BAD = ∠BCD = 42º (angles subtended from

S65 Success In Mathematics


© SAP Group Pte Ltd

Solution.indd 65 9/7/2011 2:44:07 PM


Using similar triangles, ∠BXD = 180º – xº
x ______
__ 3 2 × ∠BXD = ∠BOD
​ 4  ​= ​ 10 – x ​ 
2(180º – xº) = xº
x(10 – x) = 12
360º – 2xº = xº
10x – x2 = 12
x2 – 10x + 12 = 0 xº = 120º (∠BOD)
Using quadratic formula, x = 8.61 cm or 1.40 cm ∠AOD = 60º (angles of an equilateral triangle)
Since length x must be less than the radius as ∠BOA = 120º + 60º
shown on the diagram, = 180º (therefore BOA is a straight line)
x = 1.40 cm (3 s.f.)
17 116º
15 2 cm ∠BOC = 2 × ∠BDC (angle to centre is double
Let the radius be r cm the angle to circumference)
Since 102 = 82 + 62, the triangle is a right-angled ∠BOC = 54º
triangle ABC as shown with a circle drawn in Since ∆BOC is an isosceles triangle,
it. Two radii of the circle could also be drawn _________
180º – 54º
which are perpendicular to the 8 cm and the 6 cm ∠BCO = ​  2 ​   
sides as shown at points P and Q respectively. = 63º
The angle between the radii is also 90º. ∠BCD = 180º – 80º (opposite angles of a cyclic
A quadrilateral)
= 100º
∠OCD = 100º – 63º
= 37º
8 cm 10 cm ∠x = 180º – 37º – 27º
= 116º
R
P
22 TRIGONOMETRY
B Q 6 cm C 3
__
1 (i) ​ 5 ​ 
As all 3 sides of the triangle are tangents to the By Pythagoras’ Theorem, BC = 5 cm
circle, the 8 cm side could be split up into BP = __
3
Using basic trigonometry, sin EBC = ​ 5 ​ 
r cm and AP = (8 – r) cm.
Similarly the 6 cm side could be split up into 3
__
(ii) ​ 5 ​ 
BQ = r cm and QC = (6 – r) cm. __
3
Since tangents to circle from external points Using basic trigonometry, cos ECB = ​ 5 ​ 
must be equal AP = AR and QC = RC. __
4
Hence, 8 – r + 6 – r = 10 (iii) – ​ 5 ​ 
14 – 2r = 10 cos EBA = –cos EBC (supplementary angles)
r = 2 cm __
4
= – ​ 5 ​ 
16 ∠BOD = ∠BCD = xº (opposite angles of a __
4
(iv) – ​   ​ 
rhombus) 3
Draw two lines from B and D to point X. tan ECD = –tan ECB (supplementary angles)
__
4
B = – ​ 3 ​ 

2 (i) 15.1 cm2


__
1
Area of ∆ABC = ​ 2 ​  (5)(9) sin 42º
X C
O
= 15.1 cm2

D
(ii) 74.5º
As there are no known angle-side pair to
A
start with, sine rule cannot be used. Use

S66 Success In Mathematics


© SAP Group Pte Ltd

Solution.indd 66 9/7/2011 2:44:07 PM


cosine rule to find AC. Using basic trigonometry,
Let AC = y cm, ___h
sin θ = ​ 12  ​ 
y2 = 52 + 92 – 2(5)(9) cos 42º
___
h
= 39.12 sin 32.1º = ​ 12  ​ 
y = 6.25
h = 12 sin 32.1º
Using the angle-side pair, ÐABC and length = 6.38 cm (3 sig. fig.)
AC,
____
9 ______
6.25 5 929 m (3 s.f.)
​      ​= ​    ​ 
sin x sin 42º
Start N
9sin 42º = 6.25sin x
_______
9sin 42º
sin x = ​  6.25 ​    •B
= 0.9635
x = 74.5º 1500

3 (i) 13.5 cm (3 s.f.) •A


Let QR = y cm,
Later N
Using cosine rule,
y2 = 72 + 112 – 2(7)(11) cos 95º
= 183.42 •B
y = 13.5 cm (3 sig. fig.) •A
38º
(ii) 31.1º
Using the angle-side pair, ÐQPR and length
QR, A triangle is formed as shown.
____
7 ______
13.5
​       ​= ​    ​ 
sin x sin 95º N
7sin 95º = 13.5sin x
________
7sin 95º B • 122º
x
sin x = ​  13.5 ​   
•A
= 0.5165 1500 m
38º
x = 31.1º

4 (i) 32.1º
Let the distance be x.
Smallest angle should be facing the smallest
By basic geometry, the angle facing the 1500 m
side. Therefore, ∠BAC is the smallest.
length is 84º.
Let ∠BAC = θ.
Using sine rule,
Using cosine rule,
x
______ ______
1500
82 = 122 + 152 – 2(12)(15) cos θ ​     ​  = ​    ​ 
sin 38º sin 84º
2(12)(15) cos θ = 122 + 152 – 82 ______
sin 38º
x = 1500 ​   ​ 
sin 84º
cos θ = 0.8472
= 929 m (3 sig. fig.)
θ = 32.1º
6 (i) 252.1º
(ii) 6.38 cm (3 s.f.)
B N

12 cm 8 cm
h
A• • E
A C 220 m B
15 cm 180 m 140º

Let the height be h, which makes a right


C•
angle with the base AC.

S67 Success In Mathematics


© SAP Group Pte Ltd

Solution.indd 67 9/7/2011 5:26:59 PM


Using basic geometry, ÐBAC is 140º as the _____
26.7º
sinα = 10 sin ​  11 ​ 
 
clockwise angle of AC from the north line
is 230º. A triangle is formed as shown. = 0.4085
A α = 24.1º

140º
8
(i) 121.7º
180 m 220 m Let ÐBDA = α
Using the angle-side pair, ÐABD and length
AD,
C B ______
18 ____
20
x ​     ​ 
= ​    ​ 
sin 50º sinα
x = 180 + 220 – 2(180)(220) cos 140º
2 2 2
________
20sin 50º
= 141471 sin α = ​  18 ​   
x = 376.1 m = 0.8512
Let ÐABC = θ α = 58.3º
____
180 _______
376.1 θ =180º–α
​    ​ = ​    ​ 
sin θ sin 140º = 121.7º
__________
180sin 140º
sin θ = ​  376.1 ​    (ii) 33.6 cm
= 0.3076 Let BC = x
θ = 17.9º ∠BAD = 180º – 50º – 58.3º
Bearing of C from B = 270º – 17.9º (Given = 71.7º
that B is due east of A.) Taking the entire triangle and using cosine
= 252.1º rule,
(ii) 67.7 m (3 s.f.) x2 = 202 + 342 – 2(20)(34) cos 71.7º
The shortest distance of A from the side CB = 1129
is the height of the ∆ABC with BC as the x = 33.6 cm
base. Let the height be h and BC be x.
9 69.3º
__
1 Let the length of the line which divides the
Area of ∆ABC = ​ 2 ​  (180)(220) sin 140º
quadrilateral be x.
= 12727.2 m2
Using cosine rule,
__
1
Area of ∆ABC = ​ 2 ​  xh x2 = 82 + 122 – 2(8)(12) cos 100º
__
1 = 241.3
12727.2 = ​ 2  ​ (376.1)h
x = 15.5 cm
h = 67.7 m (3 sig. fig.) Using sine rule,
7 24.1º ____
15.5 ______ 7
​   
 ​ = ​     ​ 
sin θ sin 25º
Let ÐADB = θ
Using the angle-side pair, ÐABD and length __________
15.5 sin 25º
sin θ = ​  7 ​   
AD,
= 0.9358
____
10 ______
7
​    ​ = ​     ​  θ = 69.3º
sin θ sin40º
sin θ = 0.9183 10 (i) 104 m (3 s.f.)
θ = 180º – 66.7º
= 113.3º (as θ is an obtuse angle, the angle A • •
X Y
found on the calculator has to be 140 m
subtracted from 180º to obtain θ) 35º 180 m
∠BAD = 180º – 40º – 113.3º
B •
= 26.7º Z
Taking the entire triangle and using the angle-
Let the distance between the two trees be d.
side pair, ÐBAD and length BC,
d2 = 1402 + 1802 – 2(140)(180) cos 35º
Taking ÐBCD = α
= 10715
________
11 ____
10 d = 104 m (3 sig. fig.)
​     ​ = ​    ​ 
sin 26.7º sinα

S68 Success In Mathematics


© SAP Group Pte Ltd

Solution.indd 68 9/7/2011 2:44:08 PM


____
(ii) 139 m (3 s.f.) y2 = (​​  ​√720 ​  )2​​ ​+ 82
Let perpendicular distance between the = 720 + 64
paths be x which is also the height of the
y = 28
triangle XYZ with XY as the base.
__
1 ∆XAY is formed as shown
Area of ∆XYZ = ​ 2  ​(140)(180) sin 35º X
= 7227 m2
__
1 Y
Using Area of ∆XYZ = ​ 2 ​  base × height
__
1
7227 = ​ 2  ​(104) x A
 x = 139 m (3 sig. fig.) AX = 23.8
11 (i) 23.8 cm (3 s.f.) AY = 28
∆AFX is formed as shown with a right XY = 6.3
angle at AFX Taking ÐXAY = θ and using cosine rule,
F X _______________
23.82 + 282 – 6.32
cos θ = ​    
   ​
2(23.8)(28)
= 0.9835
A θ = 10.4º
Using Pythagoras’
________
Theorem, 12 11.3 cm (3 s.f.)
AF = √ ​ 10
____+ 12  ​ 
2 2 Reconstructing the original pyramid and taking the
 = √
​ 244 ​  apex point to be X, ∆AXC is an isosceles triangle.
__
3 X
FX = ​ 4  ​ (24) = 18
Using Pythagoras’ Theorem,
If AX  = x ____
x2 = (​​  √
​ 244 ​  )2​​​+ 182 P R 8 cm
= 568
4 cm
x = 23.8 cm (3 sig. fig.)
(ii) 6.3 cm (1 dec. pt.) A C
________
∆XGY is formed as shown with a right
AC = √
​ 102 + 102 ​   (Pythagoras’ Theorem)
angle at XGY ____
= ​√200 ​ 
X G
= 14.14 cm
Taking Y to be the mid-point of AC, ∆XYC is
Y
formed with right angle at XYC.
__
1
XG = ​ 4  ​× 24 = 6 cm X
__
1
GY = ​ 5  ​(10) = 2 cm
______
XY = √
​ 62 + 22 ​ = 6.3 cm (1 dec. pt.)
8 cm
(iii) 10.4º
YC = 8 cm
∆ACY is formed as shown with a right Y C
ACY
angle at________
_____
14.14
AC = √ ​ ____
242 + 122 ​  YC = ​  2 ​    = 7.07 cm
= ​√720 ​  Taking ÐXCY as θ,
Let AY = y Y ____8
tan θ = ​ 7.07
   ​ 
= 1.132
A C θ = 48.5º

S69 Success In Mathematics


© SAP Group Pte Ltd

Solution.indd 69 9/7/2011 2:44:08 PM


________
XC = √
​ 7.072 + 82 ​  (iii) 25.6 km (3 s.f.)
__
1
= 10.68 cm Area of ∆APB = ​ 2 ​  (30)(70) sin 100º
__
1
By similar triangles, RC = ​ 2 ​  × 10.68 = 1034 km2
= 5.34 cm Taking the shortest distance from P to line
AB as the height of the triangle with AB as
∆ARC could then be formed as shown.
the base,
R __
1
Area of ∆APB = ​ 2 ​  bh
__
1
5.34 cm 1034 = ​ 2 ​  (80.8)h
48.5º h = 25.6 km (3 sig. fig.)
A C
14.14 cm
14 (i) 36.9º
Let AR = x
Let the height of X from A be h.
x2 = 14.142 + 5.342 – 2(14.14)(5.34) cos 48.5º
____
h
= 128.39 tan 42º = ​ 250    ​ 
x = 11.3 cm (3 sig. fig.) h = 250 tan 42º
= 225.1 m
13 (i) 80.8 km (3 s.f.) Let angle of elevation of X from C be θ.
N ____
h
tan θ = ​ 300   ​ 
_____
225.1
= ​  300 ​ 
P•
70 km = 0.7503
30 km
B θ = 36.9º
A X (ii) 65.5º
The angle of elevation of X at B is 42º
Let X be the point on AB such that X is due
which increases as a person walks towards
south of P. C. At a point which is the shortest distance
∠APB = 200º – 100º from A, it is the maximum and then the
= 100º angle decreases to 36.9º at C. So find the
Let the distance AB = d shortest distance of A from BC.
d2 = 302 + 702 – 2(30)(70) cos 100º Let ÐABC be α.
= 6529 _____
300 ______ 250
​    ​ = ​    ​ 
sin α sin 20º
d = 80.8 km (3 sig. fig.)
 sin α = 0.4104
(ii) 26.1 km (3 s.f.)  α = 24.2º
Let ÐPAB = θ Let point Y on BC be the closest point to A.
Length AY becomes the height h of ∆ABC
____
70 _______
80.8
​    ​ = ​    ​  with BC as the base.
sin θ sin 100º
__________ A
70 sin 100º
sin θ = ​  80.8 ​   
250 m
= 0.8532
24.2º
θ = 58.6º B C
Y
∠AXP = 180º – 20º – 58.6º ____
h
sin 24.2º = ​ 250   ​ 
= 101.4º
Let PX = x h = 250 sin 24.2º
_________
30 x
________ = 102.5 m
​     ​ 
= ​     ​  Let angle of elevation of X at the point Y be
sin 101.4º sin 58.6º
β.
x = 26.1 km (3 sig. fig.)

S70 Success In Mathematics


© SAP Group Pte Ltd

Solution.indd 70 9/7/2011 2:44:09 PM


_____
225.1 Angle of depression = 32º – 18º
tan β = ​ 102.5 ​ 
= 14º
= 2.196
β = 65.5º 16 13.5 cm (3 s.f.)
CD = DE = 5 cm
15 (i) 23.9 m (3 s.f.) 180(6 – 2)
_________
Taking ∆FYX ∠CDE = ​  6 ​   
Y = 120º
Taking isosceles ∆CDE to find CE,
D

X 5 60º 5

32º C E
F CE = 2(5sin 60º)
__
1 = 8.660 cm
FX = ​ 2  ​ (150) = 75 m
__________
180º – 120º
YX = h ∠DCE = ​  2 ​   
∠FXY = 90º + 32º (Since YX is vertical) = 30º
= 122º
180(5 – 2)
_________
∠YFX = 45º – 32º ∠BCH = ​  5 ​   
= 13º
= 108º
∠FYX = 180º – 13º – 122º
= 45º ∠BCE = 360º – 120º – 108º + 30º
______
h ______
75 = 162º
​     ​  = ​     ​ 
sin 13º sin 45º Forming ∆BCE,
______
sin 13º
h = 75 ​   ​  B
sin 45º
= 23.9 m (3 sig. fig.) 5 cm 162º
E
C 8.66 cm
(ii) 14º
Taking ∆YXT Let BE = x
T x2 = 8.662 + 52 – 2(8.66)(5) cos 162º
Y = 182.4
x = 13.5 cm (3 sig. fig.)
23.9 m
75 m
23 MENSURATION
32º
X
1 343 cm3
XT = 75 m One face of a cube is a square. Since the area
∠YXT = 180º – 122º of the face is 49 cm2, it can be square rooted to
= 58º find the length of the side of the cube.
___
Let YT = x Length of the side = √​ 49 ​ 
x2 = 23.92 + 752 – 2(23.9)(75) cos 58º = 7 cm
= 4296.4 Volume of cube = 73
x = 65.5 m
= 343 cm3
Let ∠YTX be θ
20.25
_____
____
23.9 ______ 65.5 2 ​  π ​   cm2
​   
 ​ = ​    ​  C ___
___ 9
sin θ sin 58º Using the formula C = 2πr, r = ​ 2π  ​ = ​ 2π  ​ 
__________
23.9 sin 58º
sin θ = ​  65.5 ​   
___
4.5
= ​  π ​ 
= 0.3094
θ = 18.0º

S71 Success In Mathematics


© SAP Group Pte Ltd

Solution.indd 71 9/7/2011 2:44:09 PM


Area of the circle = πr2 __
1
Area of the isosceles triangle = ​ 2 ​  r2 sin 50.9º
(  )
___
4.5 2
= π​​ ​  π ​   ​​ ​(the π values can be

__
1
= ​ 2 ​  (6)2 sin 50.9º
reduced)
_____
20.25 = 13.97 cm2
= ​  π ​   cm2
Area of segment
Note that it is usually better to work out = area of sector – area of triangle
the problem as far as possible algebraically = 16 – 13.97
and finally substitute values at the last step. = 2.03 cm2 (2 dec. pl.)
Students should develop this skill as it gives a
better understanding of the concepts involved 4 (i) 172 = 82 + 152
At the same time, the question could be solved The sides of the triangular cross section are
more efficiently without having to reach for 8 cm, 15 cm and 17 cm, where the numbers
calculator every now and then. form a Pythagorean triple such that 172 = 82
+ 152. Hence, the triangle is a right-angled
3 17.33 cm, 2.03 cm2 (2 dec. pl.) triangle.
__
1
Using the formula for area of sector, A = ​ 2 ​  r2θ
(where θ is the angle of the sector in radians)
___
2A
θ = ​  2 ​ (re-arranging the formula)
r 15 cm 17 cm
The perimeter of a sector consists of an arc and
2 radii.
Using the formula for length of arc,
L = rθ 8 cm

(  )
___
2A
= r​ ​  2 ​   ​
r


(ii) 40 cm
__
1
Area of the cross section = ​ 2 ​  bh
___
2A
= ​  r   ​
__
1
(The θ does not feature anymore in the = ​ 2 ​  (8)(15)
calculation for this part of the question - an = 60 cm2
advantage of working out algebraically where _______________
volume
possible and hence, easier to work out the Height of prism = ​    
   ​
cross section area
answer and with minimal use of a calculator.) _____
2400
= ​  60 ​ 
 
The perimeter of the sector = L + 2r
___
2A = 40 cm
= ​  r   ​+ 2r (iii) 1720 cm2
2(16)
_____ Perimeter of the cross section = 8 + 15 + 17
= ​  6 ​   + 2(6)
= 40 cm
= 17.33 cm (2 dec. pl.) Surface area of a solid prism
The sector consists of an isosceles triangle and = perimeter of cross-sectional area × height
a segment. To find the segment area, subtract + 2 × area of cross-section.
the area of the isosceles triangle. Here we need = (40)(40) + 2(60)
the angle of the sector to find the area of the = 1720 cm2
triangle.
5 1.08 radians, 61.9º
___
2A In this case, we have to first work backwards to
θ = ​  2 ​ 
r
find the cross section area of the prism from the
2(16)
_____
= ​  2 ​  volume and height given.
(6)
_______
volume
= 0.889 radians Cross section area = ​   ​ 
height
____
180 ____
810
= 50.9º (use angle in deg = angle in rad × ​  π ​  ) = ​  15 ​  
__
1 θ
Using the trigonometric formula A = ​ 2 ​ ab sin C = 54 cm2 10 cm
for the triangle.

S72 Success In Mathematics


© SAP Group Pte Ltd

Solution.indd 72 9/7/2011 2:44:10 PM


__
1 radius of the sector)
The area of the cross-sectional sector = ​ 2 ​  r2θ
____
150
= ​ 360 ​ × π(12)2
(where θ = angle in radians between the two
radii in the sector) = 60π cm2 (better to leave in terms of π first)
__
1 The area of the sector becomes the curved area
54 = ​ 2  ​(10) 2θ
of the cone which is given by the formula,
θ = 1.08 radians curved area of a cone = πrL (where r = radius
= 61.9º of base and L = slant height of cone)
The slant height is the original sector radius.
__
1
6 (i) = ​   ​  r2θ Therefore, 60π = πr(12) (note that the π cancels
2
Using radian measure for angles, off neatly)
__
1 r = 5 cm
area of sector = ​ 2 ​  r2θ Base area of the cone = πr2
(ii) 6.24 cm (3 s.f.) = 25π cm2
__
1 9 217 cm3 (3 s.f.)
area of sector = ​ 2 ​  r2θ
First, find the area of the regular octagon
__
1 forming the base. The octagon is divided into 8
35 = ​ 2 ​  r2(1.8)
congruent isosceles triangles with all the apex
___
70 angles at the centre of the octagon. One of the
r2 = ​ 1.8  ​ 
isosceles triangles is shown below.
r = 6.24 cm (3 sig. fig.)
(iii) 23.7 cm (3 s.f.)
perimeter of sector = rθ + 2r h
h
= r(θ + 2)
= (6.24)(1.8 + 2) 67.5º 67.5º
= 23.7 cm (3 sig. fig.)
3 cm 1.5 cm
____
180º
7 187 cm (3 s.f.)
2
Each interior angle of an octagon = (n – 2)​  n ​   
Area of the remaining portion of the disc = area = 135º
of original disc – area of cut-off piece ABCD. Each interior angle of the octagon is divided into
Note the ABCD is not a usual figure where a two by the sides of the triangle giving the equal
straight forward application of any standard ____
135º
method would apply. However, it could be seen angles of an isosceles triangle ​  2 ​   = 67.5º. The
that: isosceles triangle itself could be divided into
area ABCD = sector AOD – sector BOC two right-angled triangles by drawing a vertical
____
40º ____
40º from the base. The right-angled triangle with
= ​ 360º  ​ × π(8)2 – ​ 360º  ​ × π(5)2
height formed in the process is shown above.
__
1 __
1 ___
h
= ​ 9 ​  × π(8)2 – ​ 9 ​  × π(5)2 Using trigonometry, tan 67.5º = ​ 1.5   ​ 
__
π h = 1.5 tan 67.5º
= ​ 9 ​  (64 – 25)
____
39π = 3.621 cm
= ​  9 ​   Area of right-angled triangle
__
1
Area of the remaining portion of the disc = ​ 2 ​  bh
____
39π __
1
= π(8)2 – ​  9 ​   = ​ 2 ​  (1.5)(1.5 tan 67.5º)
____
39π Since there are 8 isosceles triangles in the base,
= 64π – ​  9 ​  
_____
179π there will be 16 right-angled triangles as shown
= ​  3 ​ 
  forming the base.
= 187 cm2 (3 sig. fig.) Therefore, area of base
8 25π cm2 (  ) __
1
= 16​ ​ 2 ​   ​(1.5)(1.5 tan 67.5º)
____
150
Area of the sector = ​ 360 ​ × πR2 (where R is the = 43.456 cm2

S73 Success In Mathematics


© SAP Group Pte Ltd

Solution.indd 73 9/7/2011 2:44:10 PM


__
1 __
1
Volume of pyramid = ​ 3 ​ (base area)(height) Volume of cone = ​ 3 ​  πr2 h
__
1 __
1
= ​ 3 ​ (43.456)(15) = ​ 3  ​ πx3 (taking the r = h = x)
= 217 cm3 (3 sig. fig.) __
1
Hence ​ 3 ​  πx3 = 729
10 523 cm3, 192 cm3 (3 s.f.)  πx3 = 2187 _____
_____
√2187
3
(i) The diameter of a sphere inscribed in a  x = ​  ​  π ​ ​    
cube of sides 10 cm will also be 10 cm as  = 8.863 cm (take 3 decimal places
shown in the diagram. for intermediate values
Radius of the sphere = 5 cm as this not the final
__
4
(  )
Volume = ​ ​ 3 ​   ​πr3 answer)
Surface area of cone = πrL + πr2 (where L is the

__
4
(  )
= ​ ​ 3 ​   ​π(5)3 slant height of the cone)
To find L, use Pythagoras’ Theorem for the
= 523 cm3 (3 sig. fig.)
right-angled triangle formed by L and x.

L L x
x

x
x

(ii) A cube inscribed in a sphere of radius 10 L2 = x2 + x2


cm will have the line AB drawn in a cube = 2x__2
as shown being equal to the diameter. This L = x​√2 ​ cm __
is due to the symmetry of the sphere and = 8.863 × √​ 2 ​ 
the cube which causes the line AB to pass = 12.534 cm
through the centre of the sphere. Surface area of cone
The points A and B will be on the sphere. = πrL + πr2
Take the sides of the cube to be x and find = π(12.534x + x2)
the value of x and the diagonal on one face of = π(12.534 × 8.863 + 8.8632)
the cube as y. = 596 cm2 (3 sig. fig.)
Using Pythagoras’ Theorem, In this question, there is not much of an
y2 = x2 + x2 advantage to use formula as the powers and
y2 = 2x2__ roots are quite cumbersome. The intermediate
y = x​√2 ​  values have to be calculated with a calculator.
Again using Pythagoras’ Theorem,
102 = x2 + y2 12 4.90 cm (2 dec. pl.)
B Surface area of cone = πrL + πr2
100 = x2 + 2x2
= 3x 2 Surface area of sphere = 4πR2
____
100 (where L is the slant height of the cone, r is the
x = ​  3 ​  
2
d base radius of the cone and R is the radius of
(  )
____
100 __​ 12 ​ 
 x = ​​ ​  3 ​    ​​ ​

the sphere)
To find L, use Pythagoras’ Theorem for the
A
Volume of cube = x3 right-angled triangle formed by L, h and r.
(  ) ____
100 __​ 3 ​ 
= ​​ ​  3 ​    ​​2​

= 192 cm3 (3 sig. fig.)


11 596 cm2 (3 s.f.) h L L h
Since the cone was cast from the cube, the volume
of cone must be equal to the volume of the cube.
Volume of cube = 93 r
= 729 cm3 r

S74 Success In Mathematics


© SAP Group Pte Ltd

Solution.indd 74 9/7/2011 2:44:10 PM


L2 = h2 + r2 Base area of tank = 8 × 9
= 82 + 62 = 72 cm2
= 100 Volume of water = 5 × 8 × 9
 L = 10 cm = 360 cm3
Base area of cone = πr2
πrL + πr2 = 4πR2,
π(rL + r2) = 4πR2
__
(  )
4 2
= π​​ ​ 2 ​   ​​ ​
rL + r2 = 4R2 = 4π cm2
Substituting for r, L and R, (It is better to leave value in terms of π and
6(10) + 36 = 4 R2 work the steps using formula.)
R2 = 24___ __
1
Volume of cone = ​ 3 ​   (base area) × height
R=√ ​ 24 ​  __
1
= 4.90 cm (2 dec. pl.). = ​ 3 ​   (4π)h
____
4πh
13 40.8 cm3 (3 s.f.) = ​  3 ​  
Since both the figures have matching circular ____
4πh
Total volume of water and cone = 360 + ​  3 ​  cm3
surfaces,
base radius of cone = radius of hemisphere The total volume of water and cone could also
Let the radius be r cm and height of cone be be (base area) × height according to the second
h cm. diagram or 72 h cm3.
__
1 ____
4πh
Volume of cone = ​ 3  ​ (base area) × height Therefore, 360 + ​  3 ​  = 72 h
__
1 Solve to find h,
= ​ 3 ​   πr2h
216h = 1080 + 4πh

__
1 __ 4

Volume of hemisphere = ​ 2  ​ ​ ​ 3  ​ πr3  ​ ) 54h = 270 + πh
__
2 (54 – π)h = 270
= ​ 3  ​ πr3 ______
270
h = ​ 54 – π
  ​ 
ratio of volume of the hemisphere : volume of
__
2 __
1 = 5.31 cm (3 sig. fig.)
cone = ​ 3 ​   πr3 : ​ 3 ​   πr2h
= 2r : h
Since the ratio 2r : h = 4 : 15, 24 VECTORS
__ ___
2r 4
​   ​ = ​ 15  ​ 
h 1 (i) ​ 5.5 [  ] [  ]–10 
10     ​, ​   5     ​
30r = 4h ​___›
4(12) Since a is ​PQ​   which starts at P(2, 5) and
_____ ​___›


r = ​  30 ​ (Since
   the height
of cone is 12 cm) ends at Q(7.5,
​___›
15), ​ [ 
PQ​  = ​ 7.5 ] [  ]
  –2   ​= ​ 5.5
15 –5
 
10     ​
= 1.6 cm Since b is ​RS​   which starts at R(20, 5) and
_​ __›

__
2
Volume of the figure = ​ 3  ​ πr3 + ​ 3 ​   πr2h
__
1 ends at S(10, 10), ​RS​  = ​ 10 [  – 20
10 – 5 ] [  ]
     ​= ​ –10  
5     ​
__
1 (ii) –2a
= ​ 3 ​   πr2(2r + h) ​___›
__
1 ​AB​ starts    at A(–2, 15) and ends at
= ​ 3  ​ π(1.6)2(2 × 1.6 + 12)

​___› –13 – (–2)
= 40.8 cm3 (3 sig. fig.) B(–13, –5), ​AB​  = ​ –5 – 15 ] [  ]
      ​= ​    –11
​ ​  ​
–20
14 5.31 cm (3 s.f.)
= –2​   [  ]
5.5
​ 10 ​  ​= –2a
(iii) k___
=2
​ ›
water 5 cm h cm ​XY​ starts   at X (27.5, –10) and ends at


​___›
13 – 27.5
      ​= ​ –14.5
Y(13, 10), ​XY​  = ​ 10 – (–10)  
20      ​ ] [  ]
4 cm

S75 Success In Mathematics


© SAP Group Pte Ltd

Solution.indd 75 9/7/2011 4:30:53 PM


Since ​ –14.5
  [ 
 20      ​= a + kb ] (ii) a – b
_​ __› ​___› ​___›

= ​ 5.5
  [  ] –10
 
10     ​+ k​  5     ​ [  ] ​AD​  = ​AB​  + ​BD​  
= a + b + (–2b)
k​ –10  [  ] [  ]
–14.5
  5.5 
 5     ​= ​ 20      ​– ​ 10     ​ [  ]

=a–b
(Note that the answers could also be
k​ –10   [  ] [  ]
–20
 5     ​= ​ 10     ​ obtained by finding BC and then using
k=2 the properties of the rhombus where the
opposite sides are equal in length.)
[  ]
2 (i) ​   4
 
–9    ​, B and C refer to the same point. (iii) 24 units2
​[   9
–7   ]​+ [​  –2   ]​= [​  –9   ]​
  –5
    4
  Since the parallelogram is a rhombus, it
consists of 4 right-angled triangles of equal
Since [​    9
–7
    ]​= [​    4
    ]​+ [​  –5
–2 –9
    ]​ area. To find the area of one such triangle
___ ___ ___
​ › ​ › ​ › with base as |a| and height as |b|.
​AB​  + ​CD​  = ​AD​  
B and C refer to the same point.
(ii) ​ –21[  ]
  80
     ​ |b|
5a – 7b = 5​   9
  [  ] [  ]
–5
 
–7    ​– 7 ​ –2    ​
= [​    80
    ]​
–21 |a|

(iii) 20.2 (3 s.f.) |a| = 3 and |b| = 4


__
1
2a + 3b = 2 ​   9
  [  ] [  ]
–5
 
–7    ​ + 3 ​ –2    ​
area of triangle = ​ 2  ​(3)(4) = 6 units2

= [​      3
area of rhombus = 6 × 4 = 24 units2
    ]​
–20
_________ 4 as shown
|2a + 3b| = ​√____
32 + (–20)2 ​  B
=√ ​ 409 ​  P
= 20.2 (3 s.f.)
(iv) as
_​ __›shown A
​AE​  = ​ –13.5 [ 
6      ​
]
= 1.5​[ –9 ]
4     ​
_​ __

= 1.5​ D C
_​ __›AD​   _​ __› O
Since AE​
​   and AD​ ​   are parallel and starts Note that components that sum up to form a
from the same point A, this is only possible
vector need not be perpendicular to each other
when A, D and E are collinear.
or of equal units as in the x-y coordinate system.
3
(i) –2b The components must be non-parallel and must
Let X be the mid-point of AC. have units of consistent
_​ __›
unit lengths.
​___› _​ __
1 ›
__ (i) For the vector AB​​   from the starting point
​AX​  = ​ 2  ​AC​ ​   
A to the ending point B, 2 steps have to
__
1
= ​ 2 ​  (2a) be moved along the horizontal lines and 3
=a steps along the oblique lines. Hence, one
​___› _​ __› ​___› ​___› step to the right along the horizontal line is
Since
_​ __› AB​
​   = a + b and ​   = ​AX​  + ​XB​  
AB​ a while one step higher along the oblique
​XB​  _must be the height of the triangle lines is b.
​ __› _​ __›
and XB​​   = b For ​BC​  = 3a – 7b, starts from B. Move 3
A parallelogram formed would be a rhombus units to the right and then move 7 units
​___› ​___› down (since the coefficient of b is negative)
with diagonal ​BD​  = 2​BX​   along the oblique lines. The point obtained
​___›
Hence BD​ ​   = –2b would be point C.

S76 Success In Mathematics


© SAP Group Pte Ltd

Solution.indd 76 9/7/2011 5:31:58 PM


_​ __› _​ __› ​___›
__
1 __
7 __
1
For ​ 2  ​ a – ​ 2  ​ b, starts from A. Move ​ 2  ​ a unit ​CE​  = CD​
​   + ​DE​  
to the right and then move 3.5 units down = q – p + (–p) (Since side DE is parallel
along the oblique lines. The point obtained and equal in length to the side AB.)
would be point D. = q – 2p
​___› ​___› _​ __
1 ›
__ (iv) 2.60 (3 s.f.)
(ii) ​OP​  = ​AB​  – ​ 2  ​ ​BC​   The hexagon consists of 6 equilateral
__
1 triangles of sides |p| and |q|. Also |p| = |q|
= 2a + 3b – ​ 2  ​ (3a – 7b)
__
1
__
1 ___
13 area of each triangle = ​ 2  ​|p|2 sin 60º
= ​   ​  a + ​  2 ​  b
_​ __› 2 __
1
= ​ 2 ​  |p|2 (0.866)
OP​
​   is a position vector which starts from
the origin O. Similarly, as in (i) the vector __
1
area of hexagon = 6 × ​ 2 ​  |p|2 (0.866)
__
1 ___
13 = 2.60 |p|2 units2 (3 sig. fig.)
​ 2 ​  a + ​  2 ​  b could be drawn starting from O.
The point obtained would be P. 7 (i) ​ –3[  ]
 
4    ​
_​ __›
5 (i) 5p – q
​___› ​___› ​___› ​OA​  = ​ –3 [  ]
 
4    ​
​AC​  = ​AB​  + ​BC​   OAED___ is a parallelogram.
​ › _​ __› ​___›


= 2p + q + 3p – 2q
= 5p – q
Since DE​ ​   = OA​ [  ]
​  , ​DE​  = ​ –3  
4    ​.

3
__
(ii) – ​   ​  p + q
2 _​ __
(  29
___
)
(ii) ​ 0, ​  5 ​   ​
_​ __›
Since DA​
› __1 ›
​___ _​ __›
​   = ​ 2  ​CB​
​   , DA​
__
1
​   = ​ 2  ​​[ –(3p – 2q) ]​ ​AB​  = ​ 2 [  ] [  ]
  –3 
7   ​– ​ 4    ​


__
3
= – ​ 2 ​  p + q = ​5[  ]
 
3   ​ ​___› _​ __›
__6 3
__ Since point E
_​ __› is located on ​AB​ ,  AE​
​    is
(iii) ​ 7 ​  p + ​ 7 ​  q
_​ __› ​___› ​___› parallel___ to AB​
​  _. __
​ › ​ ›
[  ]
__
3 __
3
Since AX​ ​  ,  ​AX​  = ​ 7  ​(2p + q)
​   is ​ 7  ​of AB​ Hence AE​ ​   = k​AB​  = k​ 5  
3   ​
__
6 __
3 _​ __› _​ __› ​___›
= ​ 7  ​ p + ​ 7  ​ q ​OE​  = OA​ ​   + ​AE​  
__3
(iv) ​   ​  p
_​2__› ​___› ​___› ​___› [  ] [  ]
= ​ –3  5 
4    ​+ k​ 3   ​
​DC​  = ​DA​  + ​AB​  + ​BC​   Since the point E is on the y-axis, the x co-
__
3 ordinate of the point E must be 0.
= – ​ 2 ​  p + q + 2p + q + 3p – 2q
–3 + 5k = 0
__
3 __
3
= ​ 2  ​ p k = ​ 5 ​ 
​___›
6
(i) 2q ​OE​  = ​ –3 [  ] [  ]
  __
3 5 

[  ]
4   ​+ ​ 5 ​​  3   ​
In a regular hexagon, there are three pairs
= ​ ___ 0 
of equal and parallel sides as the opposite 29     ​
sides of a regular hexagon are equal and ​  5 ​ 
parallel. In terms of vectors, ___
29
​___› ​___› ​___› _​ __› _​ __› ​___› co-ordinates of E: (0, ​  5 ​ ).
​AB​  = ​ED​ ,  BC​ ​   = FE​
​   and CD​
​   = ​AF​ .  35
___
The line joining the points A and D is (iii) ​   ​ 
3
parallel to the sides BC and FE while it is _​ __› _​ __›
double _​the
__› length of BC. ​OC​  = k​AB​  
Hence AD​
​   = 2q. ​m [  ]
  [ 
7    ​= k​ 7 – 4
2 – (–3)
]
      ​
(ii) _q__– p ___ ___ ___
​ › ​ › ​ › ​ › = k​ 5 [  ]
 
3   ​
​CD​  = ​CB​  + BA​ ​   + AD​
​    7 = 3k
= (–q ) + (–p) + 2q __
7
=q–p k = ​ 3 ​ 
(iii) q – 2 p
7
(  )
__ ___
m = ​ ​ 3 ​   ​(5) = ​  3 ​ 
35

S77 Success In Mathematics


© SAP Group Pte Ltd

Solution.indd 77 9/7/2011 5:35:59 PM


35
___ ​___› ​___›
(iv) ​   ​  = –​AB​ + ​    AP​  
9 __
1
Taking OC as the base of ∆OEC and OD = a – b + b – ​ 3  ​ a
as the base of ∆OED, both triangles have __
2
= ​ 3 ​  a
the same height from their bases to point E.
The ratio of the area then becomes the ratio Since ∆OAB and ∆OPB both have the
of the base lengths. same base length, the area ratio must be the
​___› ___
​OC​  = ​ 7      ​
35
[  ]
​  3 ​ 

ratio of the heights.
As OA and BP are parallel and are of length

[  ]
_​ __› ​___› _​ __› __
2 __
2
[  ]
0  3      ​ ratio 1: ​ 3 ​ (a and ​ 3 ​  a) or 3 : 2, the heights of
​OD​ = ​    OE​ + ​
   ED​ =
   ​ ___ 29     ​ + ​ –4
​  5 ​  the triangle are of ratio 3 : 2. (taking OB as

[  ]
the base).
= ​  3
​ __ ____________
area of ∆OAB __ 3
9 ​   ​ ​ 
area of ∆OPB 2
   ​= ​   ​ 
​ 5 ​ 

[  ]
​___› ​___› __
2 3
__
If ​OD​ =    k​OC​  = ​  
3​  ​  ​ 9 (i) – ​ 5  ​ p + ​ 5 ​  q
__
9
​ 5 ​  ​___› ​___› ​___›
​OB​  = ​OA​ + ​ _​ __›   AB​   

[  ]
___
35 ​___›
​   ​  = OA​ + ​
​    OC​  
= k​ 37      ​ = p + q
_​ __› ​___› ​___›
___9
 k = ​ 35  ​  ​AD​ = ​    _AO​ + ​    OD​   
​ __› ​___
3 ›
__
Hence OD : OC is 9 : 35. = ​AO​ + ​ 
     ​​OB​  
5
____________
area of ∆OEC ___ 35 __
3
​     ​= ​   ​  . = –p + ​ 5  ​ (p + q)
area of ∆OED 9
__
2 __
3
__2 = – ​ 5 ​  p + ​ 5  ​ q
8 (i) ​    ​ b
3
∠OAB is common to both ∆OAB and ∆XAZ. __
2
(ii) – ​   ​  p
∠AXZ = ∠AOB (XZ//OB), 3
_​ __› _​ __›
Hence ∆OAB and ∆XAZ are similar. ​AE​ =    k​AD​ (since
   AE is a vector parallel to
__
2 AD)
XA = ​ 3 ​   OA (using the ratio of OX : XA is

____
XA
1 : 2)
___
XZ __ 2

__
2
 = k​ – ​ 5  ​ p + ​ 5  ​ q  ​
_​ __› _​ __› _​ __›
__
3
)
Ratio ​ OA ​ = ratio ​ OB  ​= ​ 3  ​
​BE​ = ​    BA​ + ​
   AE​  
_​ __› _​ __›
_​ __› _​ __
Therefore XZ​ = ​ 
__2 ›
​    3 ​  OB​ (as​    XZ and OB are ( 
__
2 __
3
)
= –q + k​ – ​ 5  ​p + ​ 5  ​q ​(since BA​
​   = –​OC​ ) 


parallel)
__
2
= ​ 3  ​ b ( 
__
3
) __
2
= ​ ​ 5 ​  k – 1  ​q – ​ 5  ​ kp
_​ __› _​ __›
(ii) 4 Since ​BE​   is parallel to ​OA​ , the q component
∠OAB = ∠YZB (YZ//OA) must be 0.
∠AZX = ∠ABO (XZ//OB) __
3
​ 5  ​ k – 1 = 0
_Hence
​ __›
∆AXZ and ∆ZYB are similar.
__
5
​AB​   = b – a k = ​ 3 ​ 
​___› ​___ ​___›
​AZ​ = ​ 
2 › __
__ 2
    ​  ​AB​  = ​    ​(b – a)
_​ __› 3 _​ __› 3
Hence BE​ =
__
2 __ 5
(  ) __
2
​    – ​ 5 ​ ​  ​ 3  ​  ​ p = – ​ 3  ​p
__
1 __
1 3
__
(iii) ​    ​
​ZB​ = ​  3 ​   = ​ 3  ​(b – a)
    ​  AB​
2
AZ : ZB = 2 : 1 Taking ∆AOB and ∆AEB to have the same
____________
area of ∆AXZ __ 22 base AB and the area ratio will be the ratio
​     ​= ​  2 ​ = 4
area of ∆ZYB 1 of their heights.
_​ __› ​___›
__2 __ 3 __
2
(iii) ​   ​  a, ​   ​  As |​BE​  | = ​ 3 ​  |​OA​ | the height ratio will be 3 : 2.
_​3__› 2_​ __› _​ __›
____________
area of ∆AOB __ 3
​BP​  = BA​ + ​ ​    AP​   ​     ​= ​   ​  .
area of ∆AEB 2

S78 Success In Mathematics


© SAP Group Pte Ltd

Solution.indd 78 9/7/2011 5:38:02 PM


(ii) 36
3
__
10 (i) ​ 7  ​(q – p) There are 36 possible outcomes as given in
_​ __›
the table (6 × 6 = 36).
​   = q – p
AC​
_​ __›
__
3 (iii) Out of the 36 possible outcomes, 6 have the
​AD​  = ​ 7  ​ (q – p)
score on A as 2. Therefore,
__
4 3
__ n(favourable outcomes)
____________________
(ii) ​ 7  ​ p + ​ 7 ​  q P (score on A = 2) = ​    
    ​
​___› ​___› ​___› n(total outcomes)
​OD​  = ​OA​  + ​AD​ R   ___
6
= ​ 36  ​ 
__
3
= p + ​ 7  ​(q – p) __
1
= ​ 6  ​
__
4 __
3
= ​ 7 ​  p + ​ 7  ​ q
___
11
__
4 (iv) ​    ​
(iii) ​   ​  p 36
3
OB is a scalar multiple of OD as O, D and There are a total of 11 outcomes which
B are collinear. has a score of 2 on A or a score of 2 on B
(  __
4
Therefore OB = k ​ ​ 7  ​ p + ​ 7  ​ q  ​
__
3
) (obtained by counting the possibilities on
the sample space, taking care not to double
​___› _​ __› ​___›
​CB​  = CO​ ​   + ​OB​   count the outcome where both A and B

(  __
4 __
3
= –q + k ​ ​ 7  ​ p + ​ 7  ​ q  ​ )
scores are 2)
P (score on either A or B = 2)

___
4k
(  ___
3k
= ​ 7 ​  p + ​ ​ 7 ​ – 1  ​q)
n(favourable outcomes)
____________________
= ​    
  
n(total outcomes)
 ​
​___› ​___›
Since ​CB​  is parallel to OA​ ( 
___
3k
)
​  ,  the ​ ​ 7 ​ – 1  ​q
___
11
= ​ 36  ​
must be 0.
___
3k Getting a score of 2 on A and getting a
​ 7 ​ – 1 = 0 score of 2 on B are not mutually exclusive
__
7 events as the occurrence of one event does
 k = ​ 3 ​ 
​___› not mean the other cannot occur. Therefore,
(  ) (  )
__
4 __ 7 __
4
​CB​  = ​ ​ 7  ​  ​​ ​ 3  ​  ​p = ​ 3  ​ p P (score of 2 on either A or B) ≠ P (score on
A = 2) + P (score on B = 2).
3
__
(iv) ​   ​ 
4 ___
4
∆OAC and ∆OBC have the same base OC. 2 (i) ​    ​ 
11
Ratio of their area is the ratio of their heights.
__
4 No. of vowels = 4 (O, A and 2 I’s)
Since OA is parallel to CB and CB = ​ 3  ​ p, No. of letters = 11
__
4 n(favourable outcomes)
____________________
OA : BC = 1: ​ 3  ​= 3 : 4 P (picking a vowel) = ​    
    ​
n(total outcomes)
____________
area of ∆OAC ______________ height of ∆OAC __ 3 ___
4
​     ​= ​     ​= ​    ​ = ​ 11  ​ 
area of ∆OBC height of ∆OBC 4
___
2
(ii) ​    ​ 
11
25 PROBABILITY No. of B’s = 2
n(favourable outcomes)
____________________
1 (i) The possible combinations of the scores are P (picking a B) = ​    
    ​
n(total outcomes)
given below: score of A, score of B. ___
2
= ​ 11  ​ 
B 1 2 3 4 5 6
A 6
___
(iii) ​    ​ 
1 1,1 2,1 3,1 4,1 5,1 6,1 11
2 1,2 2,2 3,2 4,2 5,2 6,2 As B is not a vowel, picking a B and picking
3 1,3 2,3 3,3 4,3 5,3 6,3 a vowel are mutually exclusive events (if
4 1,4 2,4 3,4 4,4 5,4 6,4 one event happens, it is definite the other
5 1,5 2,5 3,5 4,5 5,5 6,5 will not happen).
6 1,6 2,6 3,6 4,6 5,6 6,6

S79 Success In Mathematics


© SAP Group Pte Ltd

Solution.indd 79 9/7/2011 4:42:30 PM


P (picking a vowel or the letter B) GG. The nine possible outcomes are all
= P (picking a vowel) + P (a letter B) mutually exclusive from one another.
___
4 ___ 2 Of these combinations to get one blue and
= ​ 11  ​ + ​ 11  ​ 
one red the outcomes must be either RB or
___
6 BR.
= ​ 11  ​ 
P (picking a blue cube and a red cube in
10 either order)
___
3 (i) ​   ​  = P(RB or BR)
23
No. of cubes = 23 (5 + 10 + 8 = 23) = P(RB) + P(BR) (as they are mutually
No. of blue cube = 10 exclusive)
___5 ___ 10 ___ 10 ___ 5
P (picking a blue cube) = ​ 23  ​ × ​ 22 ​ + ​ 23 ​ × ​ 22  ​ 
n(favourable outcomes)
____________________ ____
50
= ​        ​ = ​ 253  ​ 
n(total outcomes)
___
10
= ​ 23 ​  102
_____
4 (i) ​    ​ 
1235
(ii) 1st pick 2nd pick
___
4 Total number of students = 40
​    ​  R → RR
22 Treat this as a ‘without replacement’
__10 situation after the first student is chosen. If
___5 ​    ​ 
​ 23   ​  22 B → RB
R __
the first chosen is a boy, there are another
8
​    ​ 39 to choose, out of whom 17 are boys. If
22
G → RG the first and second chosen are boys, there
___
5 are another 38 to choose out of whom 16
​    ​ 
22 R → BR
__ __ are boys.
5R ​ 10
23
 ​  9
​ 22   ​  P (all boys are chosen)
10B B B → BB
8G __ 8 = P (first choice is a boy) × P (second choice
​    ​
22 is a boy) × P (third choice is a boy)
G → BG ___
18 ___ 17 ___ 16
___
5 = ​ 40 ​ × ​ 39 ​ × ​ 38 ​ 
__ 8
​ 23   ​ 
​    ​ 
22 R → GR _____
102
__10 = ​ 1235  ​ 
​    ​ 
G 22 B → GB
__ 7 99
____
​    ​ (ii) ​    ​ 
22 130
G → GG For there to be at least 1 boy and 1 girl,
Treat this as a ‘without replacement’ it simply has to be a situation where there
situation after the first pick and draw a should be no ‘all boys’ or ‘all girls’ outcome
probability tree. for a ‘mixed’ outcome.
If the first is a blue, there will be one less The three situations, ‘all boys’, ‘all girls’ and
cube and one less blue cube giving a colour ‘mixed’ outcomes are mutually exclusive
combination of 5 red, 9 blue and 8 green as one happening will mean that the other
with a total of 22 cubes. two would not happen. Further, the total
Similarly, if the first is a red, there will 4 probabilities of the three should add up to 1
red, 10 blue and 8 green with a total of 22 as no other outcomes are possible.
cubes. If the first is a green, there will 5 Therefore,
red, 10 blue and 7 green with a total of 22 P (all boys) + P (all girls) + P (mixed) = 1
cubes. _____
102
P (all boys) = ​ 1235  ​ 
50
____
(iii) ​    ​  ___
22 ___ 21 ___ 20
253 Similarly, P(all girls) = ​ 40 ​ × ​ 39 ​ × ​ 38 ​ 
find P (picking a blue cube and a red cube
in either order) ____
77
= ​ 494  ​ 
It can be seen from the probability tree that
there are nine possible outcomes: _____
102 ____ 77 ____ 99
P (mixed) = 1 – ​ 1235  ​ – ​ 494  ​ = ​ 130  ​ 
RR, RB, RG, BR, BB, BG, GR, GB and

S80 Success In Mathematics


© SAP Group Pte Ltd

Solution.indd 80 9/7/2011 2:44:14 PM


__
1 7 and more than 16 are mutually exclusive
5 (i) ​    ​
2 events. There are 4 numbers here which are
For a standard pack of 52 cards, there are more than 16 (17, 18, 19 and 20).
26 red and 26 black. P(picking a number more than 16)
___
26 n(favourable outcomes)
P(red) = ​ 52 ​  ____________________
= ​    
    ​
n(total outcomes)
__
1
= ​ 2  ​ ___4
= ​ 20  ​ 
3
___
(ii) ​    ​  P (either a multiple of 7 or more than 16)
52
There are 4 kings (one each from diamonds, = P (picking a multiple of 7) + P (picking a
clubs, spades and hearts). number more than 16)
Excluding king of clubs, there will be 3 ___1 ___ 4
= ​ 10  ​ + ​ 20  ​ 
kings.
___3
P (a king, which is not a king of clubs, is = ​ 10  ​ 
chosen)
___
3 9
___
= ​ 52  ​  (iii) ​    ​ 
20
4
___ For positive integers up to 20, picking a
(iii) ​    ​  multiple of 3 and picking a multiple of 5 are
13
The event of choosing an ace and the not mutually exclusive events (the number
event of choosing a club are not mutually could be 15 which is a multiple of both 3
exclusive because there is a card which is and 5). Here we have to count the possible
an ace of clubs. numbers.
In this situation, we have to physically Multiples of 3: 3, 6, 9, 12, 15, 18.
count the possibilities carefully avoiding Multiples of 5: 5, 10, 15, 20.
double counting. Total up the possible numbers without
Aces: 4 (including the ace of clubs) double counting: 9.
Clubs: 13 (including the ace of clubs) (This is actually the n (the number of
No. of cards which are either an ace or a elements of union set of the two sets of
club = 13 + 4 – 1 multiples) if stated in set language).
= 16 P (either a multiple of 3 or multiple of 5)
(1 has to be subtracted to avoid double ___
9
= ​ 20  ​ 
counting the ace of clubs)
___
16 __
1
P(ace or a club is chosen) = ​ 52 ​  7 (i) ​   
6
___4  ​ The fraction of the area sector A occupies
= ​ 13  ​ 

6
___
1
(i) ​    ​ 
__
1 ____
(  )
60º __ 1
is ​ 6 ​  ​ ​ 360º  ​ = ​ 6  ​  ​of the circle.
10 __
1
P (dart lands on sector A) = ​ 6  ​
There are two discs which are numbered
multiples of 7 (ie. 7 and 14). 3
__
(ii) ​   ​ 
P (picking a multiple of 7) 4
n(favourable outcomes) P (dart does not land on sector C) = 1 – P
____________________
= ​    
    ​ (dart lands on sector C)
n(total outcomes)
___
2 ____
90º
= ​ 20  ​  = 1 – ​ 360º  ​ 

___
1 __
1
= ​ 10  ​  = 1 – ​ 4 ​ 
__
3
3
___ = ​ 4  ​
(ii) ​    ​ 
10
For positive integers up to 20, multiples of 19
___
(iii) ​    ​
36

S81 Success In Mathematics


© SAP Group Pte Ltd

Solution.indd 81 9/7/2011 5:41:23 PM


P (dart lands on sector A or sector D) ___1 ___ 4
P(all same colour) = ​ 35  ​ + ​ 35  ​ 
= P(dart lands on sector A) + P (dart lands
on sector D) __
1
= ​ 7 ​ 
____
60º ____ 130º
= ​ 360º  ​ + ​ 360º ​  6
__
(ii) ​ 7 ​ 
__
1 ___ 13 P (there is at least 1 red and 1 blue) = P (not
= ​ 6 ​  + ​ 36 ​ 
all the balls are of same colour)
___
19 = 1 – P (all same colour)
= ​ 36 ​ 
__
1
175
____ = 1 – ​ 7 ​ 
(iv) ​   ​  __
6
648 = ​ 7 ​ 
The probability of the second throw landing
anywhere is independent of the outcome of __
2
(iii) ​ 7 ​ 
the first throw. The probability of the first
and second throw landing on the same P (first of a particular colour and the next
sector is found by multiplying itself, two of different colour)
P (two throws of the dart lands on same = P (first red and rest blue) + P (first blue
sector A) and rest red)
= P (first throw lands on sector A) × (the probabilities of this two events could
P (second throw lands on sector A) be added as they are mutually exclusive)
= [P(first throw lands on sector A)]2 __
3 __ 4 __ 3
P(first red and rest blue) = ​ 7 ​  × ​ 6 ​  × ​ 5 ​ 
Since the four events where both the throws
land on the same sector are mutually ___6
= ​ 35  ​ 
exclusive,
__
4 __ 3 __ 2
P(two throws of the dart lands on the same P(first blue and rest red) = ​ 7 ​  × ​ 6 ​  × ​ 5 ​ 
sector) ___4
= P (both on sector A) + P (both on sector = ​ 35  ​ 
B) + P (both on sector C) + P (both on P (first of a particular colour and the next
sector D) two of different colour)
(  ) (  ) (  ) (  )
____
60 2 ____ 80 2 ____ 90 2 ____ 130 2
= ​​ ​ 360  ​  ​​ ​+ ​​ ​ 360  ​  ​​ ​+ ​​ ​ 360  ​  ​​ ​+ ​​ ​ 360 ​  ​​ ​
___
6 ___ 4
= ​ 35  ​ + ​ 35  ​ 

(  ) (  ) (  ) (  )
__
1 2 __ 2 2 __ 1 2 ___ 13 2
= ​​ ​ 6 ​   ​​ ​+ ​​ ​ 9 ​   ​​ ​+ ​​ ​ 4 ​   ​​ ​+ ​​ ​ 36 ​   ​​ ​
__
2
= ​ 7 ​ 

____
175 9 (i) 0.096
= ​ 648 ​ 
Taking the probabilities of each being late
__
1 for school is independent.
8 (i) ​ 7 ​ 
P (Charles not late) = 1 – P (Charles is late)
Each ball is taken out and is not replaced = 1 – 0.04
and hence the it will reduce the number of = 0.96
balls of the chosen colour by one and will P (Bernard will be late and Charles will not
also reduce the total number by one. be late on a day)
Since the events all red and all blue are
= P (Bernard will be late) × (Charles will
mutually exclusive,
not be late on a day)
P (all same colour) = P (all red) + P (all blue)
= 0.1 × 0.96
__
3 __ 2 __ 1
P (all red) = ​ 7 ​  × ​ 6  ​× ​ 5 ​  = 0.096
___1
= ​ 35  ​  (ii) 0.9998
__
4 __ 3 __ 2 P (not all three will be late on a day)
P (all blue) = ​ 7 ​  × ​ 6 ​  × ​ 5  ​ = 1 – P (all will be late on a day)
___4 = 1 – 0.05 × 0.1 × 0.04
= ​ 35  ​ 
= 0.9998

S82 Success In Mathematics


© SAP Group Pte Ltd

Solution.indd 82 9/7/2011 2:44:14 PM


(iii) 0.0025 Getting an odd total and a total of 6 are
P (Alan will be late for the next two days) mutually exclusive events.
= P (Alan will be late for next day and the P (either have a total which is odd or a total
following day) of 6)
= P (Alan will be late for next day) × P = P (have a total which is odd) + P (total of 6)
(Alan will be late the following day) __
1 ___ 5
= ​ 2 ​  + ​ 16  ​ 
= 0.05 × 0.05
= 0.0025 ___
13
= ​ 16 ​ 
(the probabilities could be multiplied as
they are independent ie. being early or late
one day does not influence the next day).
26 BASIC STATISTICS
__
1
10 (i) ​   ​  2 → 1, 1, 2
2
2 → 1, 1, 2 1 (i) as shown
1
3 → 1, 1, 3 Number of chin-ups Frequency
4 → 1, 1, 4 0 2
1
2 → 1, 2, 2 1 3
2 → 1, 2, 2 2 3
2
3 → 1, 2, 3 3 4
4 → 1, 2, 4 4 5
2 → 2, 1, 2 5 3
2 → 2, 1, 2 6 2
1
3 → 2, 1, 3
7 2
4 → 2, 1, 4
2 8 1
2 → 2, 2, 2
2 → 2, 2, 2 (ii) 6
2
3 → 2, 2, 3 Fittest 20% is the top 5 (20% of 25).
4 → 2, 2, 4 The top five could do 6, 7 and 8 chin-ups
From the tree diagram, the 16 possible with the least number being 6.
score outcomes for coin and die are:
(1, 1, 2), (1, 1, 2), (1, 1, 3), (1, 1, 4), (1, 2, 2), (iii) 3.6, 4, 4
(1, 2, 2), (1, 2, 3), (1, 2, 4), (2, 1, 2), (2, 1, 2), Mean = ____________________________________________
​ 0 × 2 + 1 × 3 + 2 ×     
3+3×4+4×5+5×3+6×2+7×2+8×1
25     ​
(2, 1, 3), (2, 1, 4), (2, 2, 2), (2, 2, 2), (2, 2, 3) = 3.6
and (2, 2, 4). Mode = 4 (most frequent)
By adding the scores, Median = 4 (since middle position is 13, the
___
8 __ 1 13th ranking person could do 4
P(total is even) = ​ 16  ​ = ​ 2 ​ 
___
1 chin-ups)
(ii) ​    ​ 
16
___
1 2 (i) 7
P (product is odd) = ​ 16  ​  Counting from the list, number of bottles = 7
5
___ (ii) as shown
(iii) ​    ​ 
16 Since the class interval in (i) is 490 < m
___
5
P (total is 6) = ​ 16  ​  ≤ 510, we have to take the following
13
___ additional class intervals to cover the data
(iv) ​   ​  set given:
16
Since the total must be odd or even, 430 < m ≤ 450
P (have a total which is odd) 450 < m ≤ 470
= 1 – P (total is even) 470 < m ≤ 490
__
1 510 < m ≤ 530
= 1 – ​ 2 ​ 
__
1 530 < m ≤ 550
= ​ 2 ​ 

S83 Success In Mathematics


© SAP Group Pte Ltd

Solution.indd 83 9/7/2011 2:44:15 PM


Mass of contents (g) Number of bottles _____
5800
Mean number of units sold = ​  5 ​ 
 
430 < m ≤ 450 2
450 < m ≤ 470 5 = 1160
470 < m ≤ 490 8 (iii) 1400 units
490 < m ≤ 510 7 If the mean for six years is 1200, total
510 < m ≤ 530 6 = 6 × 1200
530 < m ≤ 550 2 = 7200
(iii) 23.3 % Number sold in 2009 = 7200 – 5800
The bottles 470 g or less of sauce are in the = 1400 units
class intervals 430 < m ≤ 450 and 450 < m
5 (i) US$ 81.83
≤ 470 with a total of 7 bottles.
Percentage of bottles 470 grams or less of _______________________
77 + 82 + 79 + 84 + 86 + 83
Mean price = ​     
6 ​   
___
7
sauce = ​ 30  ​ × 100% = US$ 81.83
= 23.3 %
(ii) 7.79%
3 (i) $6200 Price in January = US$ 77
Total income from the bar chart Price in June = US$ 83
= 7200 + 4700 + 6400 + 6600 + 5800 + 6500 _______
83 – 77
= 37 200 Percentage change = ​  77 ​  × 100%
______
37 200 = 7.79%
Mean = ​  6 ​   = $6200 per month
(ii) $46 500 (iii) US$ 89.46
$6200 is 20% less than the mean of the January to June is a 5-month gap, the same
previous 6 months, so $6200 is 80% of the as June to November.
previous mean. Assuming same rate of increase in price of
_____
6200 oil = 83 (1 + 0.0779)
previous mean = ​  80 ​ 
 × 100 = 7750
= US$ 89.46
income for the previous 6 months = 7750 × 6
= $46 500 6 (i) 9
(iii) $6820 Counting from the list, number of bulbs =
5% more than $6200 means monthly 9.
income = $6200(1 + 0.05)
= $6510 (ii) as shown
income for the year = $6510 × 12 The class intervals to be plotted are:
= $78 120 950 < x ≤ 970
$78 120 – $37 200 = $40 920 970 < x ≤ 990
mean income for next 6 months 990 < x ≤ 1010
$40 920
_______ 1010 < x ≤ 1030
= ​  6 ​   
(Refer to Graph 14, page S85)
= $6820
75
___
4 (i) 1600 units (iii) ​ 77 ​ 
Number of units sold in 2006 = 900 Probability of one bulb lasting for more
Number represented by one figure = 900 ÷ 4.5 ___
18
= 200 than 970 hours = ​ 22 ​ 
Number of units sold in 2007 = 8 × 200 Probability of two bulbs not lasting for
= 1600 ___
4 ___ 3
more than 970 hours = ​ 22  ​ × ​ 21  ​ 
(ii) 1160 units
___2
Total number of units sold = ​ 77  ​ 
= (5 + 6.5 + 4.5 + 8 + 5) × 200
Probability of at least one bulb lasting more
= 29 × 200
= 5800 ___
2 ___ 75
than 970 hours = 1 – ​ 77  ​ = ​ 77 ​ 

S84 Success In Mathematics


© SAP Group Pte Ltd

Solution.indd 84 9/7/2011 2:44:15 PM


Graph 14
Topic 26, question 6(ii).

6
No. of bulbs

0
950 < x ≤ 970 970 < x ≤ 990 990 < x ≤ 1010 1010 < x ≤ 1030 Life time of bulbs

S85 Success In Mathematics


© SAP Group Pte Ltd

Solution.indd 85 9/7/2011 2:44:15 PM


7
____
7 (i) 120 (iii) ​     ​ 
590
If 135º corresponds to 45 students, 360º When two students are selected at random,
corresponds to the total number of students the first person to have the score 20 or
surveyed. ___
7
____
45 below, the probability is ​ 60  ​  (total of 7
Total number surveyed = ​ 135  ​ × 360
obtained from the class intervals 10 < x ≤
= 120 15 and 15 < x ≤ 20).
23
___ The next person to have the score, the
(ii) ​   ​ 
40 ___
6
Proportion of students engaged in running probability is ​ 59  ​  (one person with the
________
135 + 72 score had been taken out earlier and not
or swimming = ​  360 ​   
‘replaced’ and the numbers of students is
___
23 also 1 less)
= ​ 40 ​ 
probability (both will have score 20 or
Probability of a student engaged in running
___
23 ___7 ___ 6
or swimming = ​ 40 ​  below) = ​ 60  ​ × ​ 59  ​ 

15
____ ____
7
(iii) ​    ​  = ​ 590
   ​ 
119
Number of students who indicated running 9 (i)
= 45
Number of students who indicated weight
____
60 0 1 2 3 4 5 6 7 8 9 10
training = ​ 360  ​ × 120
(ii) 5.41 days, 7 days, 6 days
= 20 The mean number of days absent is the
Let the two students be A and B ____
total
Probability (one indicated running and the ​  17 ​ days
other weight training) Total = 6 + 5 + 2 + 8 + 7 + 3 + 1 + 7 + 2 + 6 +
= Probability (A indicated running and B 7+9+4+7+6+7+5
indicated weight training) + Probability = 92
(B indicated running and A indicated ___
92
weight training) Mean = ​ 17 ​ 
____
45 ____ 20 ____
20 ____ 45 = 5.41 days
= (​ 120  ​ × ​ 119  ​ 
) + (​ 120  ​ × ​ 119  ​ 
)
Mode is the most frequent in the list of days
____
15 absent, which is 7 days.
= ​ 119  ​ 
The median would be the 9th position in
8 (i) 30.25 the 17 member team. Counting from left
Take the middle scores of each class as the for a member who was absent once, the
average score of those in a class interval 9th position will be at 6 days of absence.
and sum up the estimated score. Hence, median is 6 days of absence.
Estimated sum = 12.5 × 2 + 17.5 × 5 + 22.5 ×
(iii) 7 days
9 + 27.5 × 11 + 32.5 × 18 + 37.5 × 7 + 42.5 ×
The upper quartile will be 13th position
6 + 47.5 × 2 = 1815
which is between the 9th and the 17th
_____
1815
Mean = ​  60 ​    positions in the middle upper quartile is 7.
= 30.25
10 (i) 2 6, 7
(ii) min: 2, max: 8 3 4, 4, 6, 7, 7, 9
Those in the 40 < x ≤ 45 class interval may
4 2, 5, 5, 7, 8
all meet the requirements for ‘A’ grade they
may have all miss the grade also. Those on 5 1, 3, 7, 8
the 45 < x ≤ 50 class interval would have 6 0, 2, 3
all made the grade. Key: 5/1 means 51.
Hence, minimum = 2, maximum = 8

S86 Success In Mathematics


© SAP Group Pte Ltd

Solution.indd 86 9/7/2011 2:44:16 PM


(ii) $45.05, $45 3 (i) as shown
Sum = 901 (adding the numbers) Number of correct answers (x) Frequency
____
901
Mean = ​  20 ​  x≤5 3
= $45.05 x ≤ 10 17
Taking the middle two values at position x ≤ 15 42
10 and 11(as 20 is even), both values are 45 x ≤ 20 85
and hence median = $45 x ≤ 25 112
(iii) 75% x ≤ 30 120
From the diagram 15 out of 20 workers met (Refer to Graph 16, page S89)
the criteria, (ii) 17.5, 13, 20.5, 7.5
___
15 For the median, locate the 50% (50% of
so ​ 20 ​ × 100% = 75%
120 = 60 workers) mark in the cumulative
frequency axis and draw a horizontal line
to the curve and draw a vertical line down.
27 CUMULATIVE FREQUENCY Median = 17.5
For the lower quartile, locate the 25% (25%
1 Height of students (h) Cumulative Frequency of 120 = 30 workers) mark in the cumulative
h ≤ 1.60 8 frequency axis and draw a horizontal line
h ≤ 1.65 20 to the curve and draw a vertical line down.
h ≤ 1.70 37 Lower quartile = 13
h ≤ 1.75 68 For the upper quartile, locate the 75% (75%
h ≤ 1.80 89 of 120 = 90 workers) mark in the cumulative
h ≤ 1.85 100 frequency axis and draw a horizontal line
to the curve and draw a vertical line down.
(ii) as shown
Upper quartile = 20.5
(Refer to Graph 15, page S88)
Inter-quartile range = 20.5 – 13
(iii) 1.72 cm
= 7.5
For the median, locate the 50% (50% of
(iii) 23
100 = 50 students) mark in the cumulative
frequency and draw a horizontal line to the
(____
18
Top 18 workers is the top 15% ​ 120  ​ × 100%
curve and draw a vertical line to the ‘height )
= 15% . The cut-off for the top 15% is the
of students’ axis. Median = 1.72 cm 85 (100 – 15 = 85) percentile.
th

For the 85th percentile, locate the 85% mark


2
30, 19, 25, 23.5, 28.5, 11, 5
in the cumulative frequency axis and draw
Maximum = 30, minimum = 19
a horizontal line to the curve and draw a
Data arranged in ascending order: 19, 23, 24,
24, 26, 28, 29, 30 vertical line down. 85th percentile = 23
Median = 25 (average of middle values 24 and (nearest integer).
26) 4 (i) 9.4 km
Lower quartile = 23.5 (average of 2nd and 3rd From the curve, median distance
values as there are 4 values lesser than the = 9.4 km (approx.)
median) (Refer to Graph 17, page S90)
Upper quartile = 28.5 (average of 6th and 7th
(ii) 5 km
values as there are 4 values higher than the
Lower quartile take 25% of 150 = 37.5,
median)
distance = 6.5 km
Range = 30 – 19
= 11 Upper quartile take 75% of 150 = 112.5,
Inter-quartile range = 28.5 – 23.5 distance = 11.5 km
=5 Inter-quartile range = 11.5 – 6.5 = 5 km
23.5 25 28.5
(iii) 12 km
19 • • 30 Furthest 20% is demarcated by the 80th
percentile (distances longer than the 80th
15 Values of data 35 percentile distances). 80th percentile distance
Box-and-whisker diagram = 12 km

S87 Success In Mathematics


© SAP Group Pte Ltd

Solution.indd 87 9/7/2011 2:44:16 PM


Graph 15
Topic 27, question 1(ii).

100
Cumulative Frequency

80

60

40

20

0
1.50 1.60 1.70 1.80
Height of students

S88 Success In Mathematics


© SAP Group Pte Ltd

Solution.indd 88 9/7/2011 2:44:17 PM


Topic 27, question 3(i).
Graph 16

120

100

80
Cumulative Frequency

60

40

20

0
5 10 15 20 25 30
Number of correct answers

S89 Success In Mathematics


© SAP Group Pte Ltd

Solution.indd 89 9/7/2011 2:44:17 PM


Graph 17
Topic 27, question 4(i).

125 P80
P75

100
Cumulative Frequency

P50
75

50

P25

25

0
5 10 15 20
Distance (km)

S90 Success In Mathematics


© SAP Group Pte Ltd

Solution.indd 90 9/7/2011 2:44:17 PM


The shortest distance the transport service of median 1.5, there will be a total of 13
will cover is for a worker staying 12 km away. persons on the higher side (6 + 4 + 3 + 1).
Total on the lower side must also be 13 since
5 (i) School A : 21.5 min, School B : 13 min the median is an average between 1 and 2.
School A Number of newcomers = 13 – 3 – 4
Maximum time – 42.5 min =6
Minimum time – 21 min
Range = 42.5 – 21 7 (i) 36.5, 29, 42
= 21.5 min median is determined by the the 9th and
School B the 10th positions which are 35 and 38
Maximum time – 36 min respectively.
Minimum time – 23 min _______
35 + 38
Median = ​  2 ​   = 36.5
Range = 36 – 23
= 13 min Among the 9 of those on the lower side
of the median, lower quartile will be
(ii) 31 min, 29.8 min determined by the 5th position, ie. 29.
For the medians draw lines at the 50% mark Among the 9 of those on the upper side
or at 10 students. of the median, upper quartile will be
School A : 31 min, School B : 29.75 min determined by the 14th position, ie. 42.
(approximate to 29.8 min)
(ii) 27, 13
(iii) Since School B had narrower range of range = 53 – 26
timings obtained by their students, they = 27
must have been generally trained to be able Inter-quartile range = 42 – 29
to run better. Even though the top runners = 13
were from School A, there is a wider Box-and-whisker diagram:
disparity in the timings by the runners from 29 36.5 42
School A. 26 • • 53

6 (i) 2.05, 2, 2
∑fx
____ 20 Ages of technicians 60
Using mean = ​   ​ 
∑f
8 (i) 495.8 g, 494.6 g
___________________________________
3×0+4×1+6×2+4×3+2×4+1×5
= ​  3 +    
4 + 6 + 4 +   
2 + 1 ​ Mean mass of line A product
___
41
= ​ 20 ​  = _____________________________________________
​ 502 + 487 + 488     
+ 490 + 507 + 500 + 498 + 491 + 505 + 490
10     ​
= 2.05 drinks = 495.8 g
Mode = 2 (most frequent). Mean mass of line B product
Median = 2
(ii) 1.36 = _________________________________________________
​ 510 + 501 + 482     
+ 489 + 496 + 506 + 478 + 489 + 503 + 492
10     ​
____________

(  )
Using, = 494.6 g

∑fx2 ____
____ ∑fx 2
Standard deviation = ​ ​       ​  ​​ ​ ​
​– ​​ ​    
∑f ∑f (ii) 7.07, 9.92
∑fx2 3 × 02 + 4 × 12 + 6 × 22 + 4 × 32 + 2 × 42 + 1 × 52
____ Standard deviation of line A product
​   ​ = _______________________________________
​       20    ​
= 7.07
∑f
_____________________
0 + 4 + 24 + 36 + 32 + 25 Standard deviation of line B product
= ​  20 ​       = 9.92
____
121 The lighter loaf is very likely to come from
= ​  20 ​ 
B where the mean is lower. The mass of
= 6.05 __________ line B’s products are also more varied
Standard deviation = √
​ 6.05 – 2.052 ​  from their mean value and hence, a higher
= 1.36 chance of being lighter.
(iii) 6 9 (i) 25%
Since the new median is 1.5 and all the (from the diagram)
newcomers were added to the lesser side Median increase = 25%

S91 Success In Mathematics


© SAP Group Pte Ltd

Solution.indd 91 9/7/2011 2:44:18 PM


(ii) 41%, 19% removed and mixed.
(from the diagram) For batch A, the heights would range from
Maximum = 36% 31 cm (90th percentile of the batch) to 35
Minimum = –5% cm (maximum of the batch).
Range = maximum – minimum For batch B the heights would range from
= 36 – (–5) 40 cm (90th percentile of the batch) to 44
= 41% cm (maximum of the batch).
Upper quartile = 29% Hence the maximum of the combined batch
Lower quartile = 10% would be 44 cm (maximum of batch B) and
Inter-quartile range = 29 – 10 the minimum is 31 cm (90th percentile of
= 19% batch A) give a difference of 13 cm.
(iii) (Refer to Graph 18, page S93)
Draw a cumulative frequency diagram
and fit in the information given in the 28 NUMBER PATTERNS AND
box-and-whisker diagram. The horizontal PROBLEM SOLVING
axis represents the increase in value of
individual investments. The vertical axis 1 (i) 1 + 3 + 5 + 7 + 9 = 25
represents the number of investments As matter of patterning it can be seen that
made. Since the number of investments is every line adds on the next odd number.
not given, express it in percentage. So, next line is 1 + 3 + 5 + 7 + 9 = 25
Mark the maximum increase and minimum (ii) 64
increase at 100% and 0% at the number of Again as matter of patterning, it can be seen
investments. The median will be at the 50% that the sum is n2 where n is the number of
while the lower and upper quartiles will be terms added up. There are eight terms to be
at the 25% and 75% respectively. Then join summed up in the addition 1 + 3 + 5 + 7 +
the points with a smooth curve as shown. 9 + 11 + 13 + 15.
10 (i) 24 cm, 31 cm Therefore, 1 + 3 + 5 + 7 + 9 + 11 + 13 + 15
(from the diagram) = 82
Find the median heights by drawing a = 64
horizontal line from the 50% mark (25 (iii) –50
plants). To evaluate 1 – 2 + 3 – 4 + 5 – 6 +  + 97
Batch A median : 24 cm – 98 + 99 – 100, split up the sequence as
Batch B median : 31 cm (1 + 3 + 5 +  + 97 + 99) – (2 + 4 + 6 + 
(ii) 8 cm, 10.5 cm + 98 + 100).
(from the diagram) There are 50 terms in the first bracket (all
For both batches, find the lower and upper the odd numbers less than 100) and hence 1
quartile heights by drawing horizontal lines + 3 + 5  + 97 + 99 = 2500 (502 = 2500).
from the 25% (12.5 plants) and 75% (37.5 Using the formula,
n(n + 1)
_______
plants) marks respectively. 1 + 2 + 3 + 4 +  + n = ​  2 ​   
Batch A: lower quartile – 20 cm
The second bracket is
upper quartile – 28 cm
2 + 4 + 6 +  + 98 + 100
inter-quartile ranges = 28 – 20
= 2(1 + 2 + 3 +  + 49 + 50)
= 8 cm


Batch B: lower quartile – 25 cm
upper quartile – 35.5 cm
[  50(50 + 1)
_________
= 2​ ​  2 ​  ] ​
inter-quartile ranges = 35.5 – 25 = 50(50 + 1)
= 10.5 cm = 2550
(iii) 13 cm Therefore, 1 – 2 + 3 – 4 + 5 – 6 +  + 97
For the tallest 10% of both batches draw a – 98 + 99 – 100
line at the 90th percentile. The plants with = 2500 – 2550
heights above the 90th percentile would be = –50

S92 Success In Mathematics


© SAP Group Pte Ltd

Solution.indd 92 9/7/2011 2:44:18 PM


Graph 18
Topic 27, question 9(iii).

–20 0 20 40 60
Percentage increase in value of investment

100%

75%
Number of investments

50%

25%

0%
–10 0 10 20 30 40
Percentage increase in value of investment

Inter-quartile range

S93 Success In Mathematics


© SAP Group Pte Ltd

Solution.indd 93 9/7/2011 2:44:18 PM


Alternatively, (iv) 22
1 – 2 + 3 – 4 + 5 – 6 +  + 97 – 98 + 99 Let first number be x
– 100 Sum of 6 diagonal numbers
= (1 – 2) + (3 – 4) + (5 – 6) +  + (97 – 98) = x + (x + 7) + (x + 14) +  + (x + 35)
+ (99 – 100) = 6x + 7(1 + 2 + 3 + 4 + 5)
(bracketing the terms into pairs where no = 6x + 105
sign change is required as signs before the 6x + 105 = 867
open brackets are all positive) x = 127
As there are 50 pairs of bracketed terms Since x is the first number in its row, last
each evaluating to a value of –1, number in the same row
(1 – 2) + (3 – 4) + (5 – 6) +  + (97 – 98) = 127 + 5
+ (99 – 100) = –50 = 132 (a multiple of 6)
Row number = 132 ÷ 6
2 (i) row 84, column 2 = 22
The last number of each row is a multiple
of 6. Since 500 is not a multiple of 6, it will 3 (i) 100, 180
appear in the next row after the row where 53 – 52 = 100
the highest multiple of 6 which is less than 63 – 62 = 180
500 appears. (ii) 7600
500 ÷ 6 = 83 remainder 2 203 – 202 = 7600
500 – 2 = 498
(iii) as shown
498 (multiple of 6) will appear as the last
Continuing the pattern in (i) 73 – 72 = 294, 83
number in the 83rd row (498 ÷ 6 = 83) 500
– 82 = 448, 93 – 92 = 648 and 103 – 102 = 900.
will be in the 84th row, 2nd column.
The last digit of the number on the right of
(ii) 28 the equal sign is determined solely by the
last digit of the number k and it follows the
a b repeating pattern 0, 4, 8, 8, 0, 0, 4, 8, 8, 0 as
for lines 1 to 10.
The last digit of 10 164 is 4.
c d
Therefore, last digit of k must be 2 or 7.
(iv) 22
From (iii), the possible the values of k have
If the 4 numbers in a square are a, b, c and
to be 12, 17, 22, 27, etc. Narrowing further
d as shown,
it must be more than 20 according to the
b=a+1 line 203 – 202 = 7600. Trying 223 – 222 =
c=a+6 10  164 gives the required k to be 22.
d=a+7
Sum of the 4 numbers 4 (i) 55
= a + (a + 1) + (a + 6) + (a + 7) There is 1 number in the first row, 2 in the
= 4a + 14 second row, etc.
If 4a + 14 = 126 In 10 rows, there would be a total of (1 + 2
a = 28 + 3 +  + 10) numbers.
10(10 + 1)
_________
(iii) 67, 74, 81, 88, 95, 102 1 + 2 + 3 +  + 10 = ​  2 ​   
First number of twelveth row = 55
= Last number of eleventh row + 1 (ii) 105
= 11 × 6 + 1 Since the total position of the number is the
= 67 number itself.
For a diagonal the next number has to The last number in the 14th row
advanced by 1 row (+6) and 1 position to = 1 + 2 + 3 +  + 14
the right (+1). Next number is 67 + 7 = 74. 14(14 + 1)
_________
The sequence is 67, 74, 81, 88, 95, 102. = ​  2 ​   
= 105

S94 Success In Mathematics


© SAP Group Pte Ltd

Solution.indd 94 9/7/2011 2:44:19 PM


(iii) 870 n(n+1)
______
If the bottom layer has 120 cubes, ​  2 ​  =
From (i) and (ii), the last number in the 10th
120 where n is the layer number.
row is 55 and the last number in the 14th
n(n + 1)
_______
row is 105. Therefore, the row with number ​  2 ​   = 120
70 can be located by trial-and-error. n(n + 1) = 240
Last number on the 11th row n2 + n – 240 = 0
= 1 + 2 + 3+  + 11 Solving, n = –16 (rej) or n = 15
= 66 There will be 15 layers.
Last number on the 12th row
6 (i) 4 cm, 5 cm, n, n + 1
= 1 + 2 + 3+  + 12
The length exceeds the breadth by 1 cm for
= 78 every subsequent rectangle. 4th rectangle
Hence, 70 is in row 12. would be 4 cm by 5 cm.
The numbers in row 12 are 67, 68, 69, 70, Hence breadth = n cm and length
, 78 = (n + 1) cm
11(11 + 1)
_________
67 + 68 + 69 +  + 78 = 67(12) + ​  2 ​    (ii) n(n + 1) cm2, (4n + 2) cm
= 870 Area = n(n + 1) cm2
Perimeter = 2n + 2(n + 1) cm
5 (i) 10 = (4n + 2) cm
If the top 3 layers are removed the top view (iii) 18
of the 4th layer will be as shown: 4n + 2 = 74
n = 18
(iv) 23
n(n + 1) = 552
n2 + n – 552 = 0
Solving, n = –24 (rej) or n = 23
The number of cubes = 1 + 2 + 3 + 4 3 __
__ 5 ___ 7
= 10 7 (i) ​    ​ , ​    ​ , ​    ​ 
2 6 12
(ii) 20 __
1 __ 1 __ 3
​ 1 ​  + ​ 2 ​  = ​ 2 ​ 
Number of cubes in the 3rd layer = 1 + 2 + 3
=6 __
1 __ 1 __ 5
​ 2 ​  + ​ 3 ​  = ​ 6 ​ 
The second layer has 3 and the first layer has
__
1 __ 1 ___ 7
one respectively. Hence the total number of ​ 3 ​  + ​ 4 ​  = ​ 12  ​ 
cubes = 10 + 6 + 3 + 1
2m + 1
________
= 20 (ii) ​    ​ 
m(m + 1)
(iii) 56 For the fraction on the right of the equal
Taking the topmost layer to be the first sign, it can be seen that the numerator is the
layer, number of cubes = 1 sum of the denominators of the fractions
Number of cubes in the second layer = 1 + 2 on the left while the denominator is the
=3 product.
Number of cubes in the third layer = 1 + 2 + 3 __
1 _____ 1 m+m+1
_________
=6 Hence ​ m  ​+ ​ m + 1    ​ = ​   ​ 
m(m + 1)
For subsequent layers, keep adding the ________
2m + 1
= ​     ​ 
n (n+1)
______ m(m + 1)
numbers according to the formula ​  2 ​    63
____
(iii) ​    ​ 
where n is the layer number. 992
n(n + 1)
_______ Take m as the position number of each
For n = 6, ​  2 ​   = 21 fraction.
Numbers of cubes = 1 + 3 + 6 + 10 + 15 + 21 __
3 __ 5 ___ 7 ___ 9 __x
For ​ 2 ​  – ​ 6 ​  + ​ 12  ​ – ​ 20  ​ +  ​ y ​, the numerator
 = 56
is 2m + 1, while the denominator is
(iv) 15 m(m + 1) and the sign alternates.

S95 Success In Mathematics


© SAP Group Pte Ltd

Solution.indd 95 9/7/2011 2:44:19 PM


For the 31st term, numerator = 2(31) + 1 3 × 3, 4 × 4, 5 × 5, 6 × 6 and 7 × 7.
= 63 Number of different sizes = 7
denominator = 31(32) (ii) 4
= 992 By joining the points as indicated it can be
As 31 is an odd number, the alternating seen that 4 squares of 6 × 6 grid could be
sign would make the sign for the 31st term drawn.
____
63
positive. The term is ​ 992  ​  .
33
___
(iv) ​   ​ 
32
__
3 __ 5 ___ 7 ___ 9 __x
​ 2 ​  – ​ 6 ​  + ​ 12  ​ – ​ 20  ​ +  ​ y ​

__
3 __ 5 ___ 7 ___ 9 ____
63
= ​ 2  ​– ​ 6 ​  + ​ 12  ​ – ​ 20  ​ +  + ​ 992  ​ 


__
1 __
1 __
) ( 
1 __
1 __ 1
) (  ) ( 
__
1 __ 1
)
= ​ 1 + ​ 2 ​   ​– ​ ​ 2 ​ + ​ 3 ​   ​+ ​ ​ 3 ​ + ​ 4 ​   ​– ​ ​ 4  ​+ ​ 5 ​   ​+ 
___
1 ___
(  1
+ ​ ​ 31  ​ + ​ 32  ​   ​ )
__
1
From the 1st and 2nd sets of brackets, ​ 2 ​ 
cancels off.
__
1
From the 2nd and 3rd sets of brackets, ​ 3 ​ 
cancels off.
Continuing the process, the only terms left
___
1 ___ 33
would be 1 + ​ 32  ​ = ​ 32 ​  .

8 (i) as shown
N Sum of Sum of
Series
value bases (A) series (B)
13 1 1 1
1 +2 3 3
2 3 9
(iii) 140
Number of 7 × 7 squares = 1
13 + 23 + 33 3 6 36
Number of 6 × 6 squares = 4
1 +2 +3 +4
3 3 3 3
4 10 100 Taking the possible locations of the top
1 + 2 +  + 10
3 3 3
10 55 3025 left corner of a 5 × 5 square, they could be
13 + 23 +  + 113 11 66 4356 located at the positions indicated. A 5 × 5
square could not be drawn from any other
(ii) 25 intersections at the top left corner. Hence,
13 + 23 +  + k3 = 105 625 the number of 5 × 5 squares that could be
According to the table, Sum of series = drawn = 9.
(Sum of bases)2 At the other extreme, the number of 1 × 1
Since, Sum of series = 105 625
_______ squares that could be draw = 49.
Sum of bases = ​√105 625 ​  These suggests that the number of possible
= 325 squares of a particular grid are all perfect
Hence 1 + 2 + 3+  + k = 325 square numbers.
k(k + 1)
_______ Hence 12 + 22 + 32 + 42 + 52 + 62 + 72 =
​  2 ​   = 325 140
k(k + 1) = 650
k2 + k – 650 = 0 (possible locations of
Solving k = –26 (rej) or k = 25 the top left corner are
marked with )
9 (i) 7
The squares can be of grid sizes 1 × 1, 2 × 2,

S96 Success In Mathematics


© SAP Group Pte Ltd

Solution.indd 96 9/7/2011 2:44:19 PM


(iv) 14
According to (iii) 12 + 22 + 32 +  + n2 =
1015. By trial-and-error, if n = 10, 12 + 22
+ 32 +  + n2 = 385 and if n = 15, 12 + 22 +
32 +  + n2 = 1240. Hence n must be such
that 10 < n < 15. By narrowing down using
trial-and-error, the n value can be found to
be 14.

10 (i) 15
The 5 numbers are 6, 14, 15, 16 and 24.
__________________
6 + 14 + 15 + 16 + 24
Average = ​  5 ​     
= 15
(It has to be noted that the average could
also be obtained by simply taking the
number in the centre as 6 = 15 – 9, 14 = 15
– 1, 16 = 15 +1 and 24 = 15 + 9.)

(ii) 119
If the smallest of the 5 numbers in the
diamond shape is 110, the other numbers
will be 118, 119, 120 and 128. The average
will be 119. It will not be necessary to total
up and divide to find the average.

(iii) 87
If the number in the centre is x, the total of
a set of 5 such numbers would be 5x since
x would be the average.
5x = 390
x = 78
Largest number (located below the average)
= 78 + 9
= 87

(iv) 206
Largest number in the sequence is 216
(a multiple of nine located at the end of
a line). But 216 cannot be included in a
diamond shape as it would be at the right
most end of the last row. However, 215 can
be included with 197, 205, 206 and 207
being the other numbers. The average of
these 5 numbers is 206.

S97 Success In Mathematics


© SAP Group Pte Ltd

Solution.indd 97 9/7/2011 2:44:20 PM

Das könnte Ihnen auch gefallen